EM EOR (RR)
A 67-year-old woman presents with generalized fatigue and malaise for the past week. She also notes melenic stool for the past few months. Her vital signs include HR of 112 bpm, BP of 105/37 mm Hg, and RR of 15/minute. Laboratory studies show a hemoglobin level of 4.0 g/dL and platelet count of 110,000/µL. How many units of packed red blood cells should be given to raise her hemoglobin level to 8 g/dL? 1 unit 2 units 3 units 4 units
4 units For most patients, a hemoglobin level of 7 g/dL is considered the threshold below which transfusion should be initiated. A single PRBCs unit will raise hemoglobin by 1 g/dL and hematocrit by 3%, therefore, a patient with a hemoglobin of 4 g/dL would require 4 units of PRBCs to achieve a hemoglobin of 8 g/dL. In general, any patient with a platelet count of < 5,000/µL should receive a platelet transfusion as well as those subject to massive transfusion protocols. A single donor unit of platelets will increase a patient's platelet count by approximately 50,000/µL. Platelets are relatively contraindicated in those with immune thrombocytopenia, thrombotic thrombocytopenic purpura, and heparin-induced thrombocytopenia
Of the following, what is the most common cause of acute arterial occlusion? Atrial fibrillation Endocarditis Vasospasm Ventricular thrombus
Atrial fibrillation Acute arterial occlusion can occur secondary to thrombus formation in situ or emboli. The heart is the predominant source of peripheral emboli, with atrial fibrillation being responsible for most cases of embolic occlusion. Atrial fibrillation causes poor cardiac wall motion and stagnant blood flow that promotes clot formation. Treatment is based on the location of the occlusion.
In which of the following scenarios is immediate transcutaneous pacing indicated? A 33-year-old marathon runner with sinus bradycardia at a rate of 39 beats/minute and a blood pressure of 90/60 mm Hg who is asymptomatic A 42-year old woman with sinus bradycardia at a rate of 45 beats/minute and a blood pressure of 111/76 mm Hg who swallowed a bottle of metoprolol pills in the waiting room A 72-year-old man with sinus bradycardia at a rate of 41 beats/minute and a blood pressure of 89/50 mm Hg with chest pain, diaphoresis, and confusion A 75-year-old man with sinus bradycardia at a rate of 50 beats/minute and a blood pressure of 145/90 mm Hg with exertional dyspnea
A 72-year-old man with sinus bradycardia at a rate of 41 beats/minute and a blood pressure of 89/50 mm Hg with chest pain, diaphoresis, and confusion
Which of the following patients may obtain the most benefit from treatment with oseltamivir twice a day for 5 days during influenza season? A 50-year-old man with a history of hypertension and high cholesterol diagnosed with influenza A after 3 days of symptoms A 73-year-old woman with a history of arthritis diagnosed with influenza A after 3 days of symptoms A 9-year-old healthy boy diagnosed with influenza A after 3 days of symptoms An asymptomatic 35-year-old woman with two children at home who are ill with influenza A for 1 day
A 73-year-old woman with a history of arthritis diagnosed with influenza A after 3 days of symptoms Influenza is a highly contagious orthomyxovirus that is transmitted through aerosolized respiratory secretions and large droplets. Patients complain of fever and respiratory symptoms, which usually resolve within about 1 week. Complications from the illness can include pneumonia, worsening chronic respiratory disease, myocarditis, meningitis, and bacterial superinfection. Individuals at greatest risk for these complications include the very young, the very old, and those with chronic medical conditions such as chronic obstructive lung disease, congestive heart failure, chronic renal insufficiency, and an immunocompromised state. Symptomatic treatment is the mainstay with antipyretics, decongestants, and oral hydration. However, antiviral medications are available to help prevent disease complications and shorten the duration of symptoms. The use of a neuraminidase inhibitor, such as oseltamivir, is the recommended first-line therapy for patients requiring treatment with antivirals. The Infectious Diseases Society of America also recommends treating high-risk outpatients with oseltamivir. hese populations include children under age 5 but especially under age 2, adults over age 65, pregnant women up to 2 weeks postpartum, residents of nursing homes and long-term care facilities, Americans Indians, and people with chronic asthma, chronic lung disease, coronary artery disease, congenital heart disease, diabetes, renal insufficiency, chronic liver disease, extreme obesity with a body mass index greater than 40 kg/m2, and immunocompromised individuals.
A 65-year-old asymptomatic man is noted to have a murmur on cardiac auscultation. Which of the following findings is suggestive of chronic mitral regurgitation? A continuous machine-like murmur throughout the cardiac cycle A harsh crescendo-decrescendo systolic murmur A holosystolic murmur with radiation to the axilla A mid-systolic "click" followed by mid- to late-systolic murmur over the mitral area
A holosystolic murmur with radiation to the axilla The characteristic murmur of chronic mitral regurgitation is a holosystolic murmur at the apex which radiates to the axilla. A continuous machine-like murmur throughout the cardiac cycle (A) is the characteristic murmur of a patent ductus arteriosus (PDA). A harsh crescendo-decrescendo systolic murmur (B) is the characteristic murmur of aortic stenosis, where calcific degeneration leads to stenosis and narrowing of the aortic valve. A mid-systolic "click" followed by a mid to late-systolic murmur over the mitral area (D) is characteristic of mitral valve prolapse.
Which of the following patient history elements is most indicative of cardiac syncope? Absence of a postdrome Postevent confusion Prodrome with dizziness and nausea Provocation with prolonged standing
Absence of a postdrome Syncope is defined as a transient loss of consciousness due to cerebral hypoperfusion with loss of postural tone, characterized by a rapid onset and spontaneous recovery. The delineation between benign and lethal etiologies during initial evaluation is vital to ensure appropriate assessment of patients and concomitant use of resources. The overwhelming cases of syncope presenting to the emergency department are related to orthostatic causes or neurally mediated causes, such as vasovagal, situational, and carotid sinus etiologies. a detailed history from the patient and bystanders is key to the assessment of cardiac etiologies of syncope. Historical elements indicating cardiac causes of syncope, as opposed to orthostatic or neurocardiogenic episodes, include syncope during exertion, palpitations at the time of syncope, ECG findings of dysrhythmogenesis (e.g., bifascicular block, intraventricular conduction delay, QT interval abnormalities, Brugada pattern findings), family history of sudden cardiac death, and lack of postdrome.Prolonged prodrome and postdrome symptoms are common with vasovagal etiologies, with fatigue and nausea lingering well beyond the event itself, whereas true cardiac etiologies typically have no harbingers or postepisode symptomatology.
A 60-year-old man presents to the ED with severe left lower extremity pain and paresthesias after being diagnosed with a myocardial infarction one week ago. He states his pain started suddenly while at home. On physical exam, his left leg is cool and cyanotic. Which of the following is the most likely diagnosis? Acute arterial occlusion Necrotizing soft tissue infection Phlegmasia alba dolens Phlegmasia cerulea dolens
Acute arterial occlusion Arterial Embolism History of recent MI or atrial fibrillation Sudden onset of pain, pallor, paresthesias, pulselessness, paralysis, and pain out of proportion to exam (6 Ps) Most common source is left heart Most common site is femoral artery bifurcation Treatment: anticoagulation (heparin), restore flow (embolectomy, thrombolysis, bypass, angioplasty)
A 45-year-old man presents to the urgent care clinic with abdominal pain, jaundice, anorexia, and fatigue. He recently returned from a humanitarian trip to Honduras. His temperature is 39.2°C (102.5°F). Physical examination reveals hepatomegaly, right upper quadrant abdominal tenderness, and pruritus. Which of the following is the most likely diagnosis? Acute appendicitis Acute cholecystitis Acute pancreatitis Acute viral hepatitis
Acute viral hepatitis Right upper quadrant abdominal pain, jaundice, anorexia, fatigue, pruritus, and fever are strongly suggestive of acute viral hepatitis. This patient most likely acquired hepatitis A virus infection from his recent trip to Honduras, likely from contaminated food as the virus is transmitted by the fecal-oral route. ever, anorexia, and fatigue appear first, followed by dark urine and pale stools. Jaundice and pruritus appear next, usually within two weeks of symptom onset. Laboratory studies typically show elevations in serum aminotransferases, serum bilirubin, and alkaline phosphatase. Diagnosis is made by the detection of serum IgM anti-HAV antibodies. Acute viral hepatitis is self-limited and resolves without treatment in most patients. However, a small percentage of cases can progress to fulminant hepatic failure.
A 21-year-old woman presents to the ED with racing heart rate. Her ECG is shown above. Vital signs are BP 120/80 mm Hg, HR 180 beats per minute, RR 18 breaths per minute, and T 98.4°F. She has already attempted bearing down with no improvement. Which of the following is the most appropriate next step in treatment? Adenosine Cardioversion Diltiazem Procainamide
Adenosine This patient is exhibiting signs and symptoms of supraventricular tachycardia (SVT); however, more specifically this is paroxysmal reentry or preexcitation tachycardia. Reentry SVT has two potential routes - AV nodal reentry tachycardia (AVNRT) or atrioventricular reentry tachycardia (AVRT). Reentry circuits require at least two different conduction pathways with different refractory time. The ECG will show a regular, fast, narrow complex tachycardia with absent P waves. Management depends upon the stability of the patient. Unstable patients should undergo immediate synchronized cardioversion. Stable patients should first try vagal maneuvers. If these are unsuccessful, adenosine given as a rapid bolus is the medication of choice. It is first given at a dose of 6 mg, followed by 12 mg if the first dose is unsuccessful.
A 22-year-old man who was pitching a baseball game presents to the emergency department by ambulance after being struck in the head by a batted baseball thirty minutes prior to arrival. The paramedics report that the patient was conscious and alert on their arrival but that his teammates noted he was unconscious for a short period of time on the pitcher's mound after being struck. One of his teammates also informed the paramedics that the patient has a history of Christmas disease. On arrival, the patient's vital signs include a heart rate of 108 bpm, respiratory rate of 18 breaths/minute, a blood pressure of 144/96 mm Hg, and he is estimated to weigh 70 kg. His GCS is 11; he responds to loud verbal stimuli, but is slow to answer questions posed to him. Physical examination is notable for a large, tender, ecchymotic region along his left temporal bone. What is the appropriate course of management? Administer 3,500 units of factor VIII, followed by a noncontrast CT of his head Administer 7,000 units of factor IX, followed by a noncontrast CT of his head Obtain a noncontrast CT of his head, and administer 3,500 units of factor IX if there is evidence of intracranial hemorrhage Obtain a noncontrast CT of his head, and administer 7,000 units of factor VIII if there is evidence of intracranial hemorrhage
Administer 7,000 units of factor IX, followed by a noncontrast CT of his head Hemophilia B, or Christmas disease. Treatment in all cases regardless of severity is repletion of the deficient factor, with the goal of increasing circulating factor levels to anywhere from 20% (mild oropharyngeal bleeding) to 50% (definitive hemarthrosis) and 100% in cases of severe, life-threatening hemorrhage (intracranial, intra-abdominal). In any case of presumed central nervous system hemorrhage in a patient with hemophilia, treatment involves immediate 100% factor repletion prior to any diagnostic studies which would delay factor administration. For this patient with a history and examination concerning for a traumatic epidural hematoma, this is the appropriate course of management. Calculation of factor repletion in hemophilia B is obtained by multiplying the patient's weight in kilograms by the percent of factor repletion desired, and then by the volume of distribution of the product, which for factor IX is 1.
A 45-year-old man presents to the emergency department for a headache. He has a history of diabetes mellitus type 1, hypertension, and alcohol use disorder. He states he started feeling unwell 3 days ago, and the headache started this morning. He also reports having blurred vision and nausea without vomiting and no chest pain or shortness of breath. Vital signs are remarkable for a temperature of 39°C, HR 115 bpm, BP 100/60 mm Hg, RR 23 breaths/min, and SpO2 of 97%. Physical examination is remarkable for a man who appears lethargic. Upon passively flexing the patient's neck, he flexes at the hips and knees. Which of the following sequences is the best next step in this patient's management? Administer ceftriaxone and vancomycin; CT head; lumbar puncture Administer ceftriaxone, ampicillin, and vancomycin; CT head; lumbar puncture CT head; lumbar puncture; administer ceftriaxone, ampicillin, and vancomycin Lumbar puncture; administer ceftriaxone, ampicillin, and vancomycin
Administer ceftriaxone, ampicillin, and vancomycin; CT head; lumbar puncture Meningitis is due to meningeal infection and inflammation, and the diagnosis should be considered in patients with headache who present with an associated triad of fever, altered mentation, and neck stiffness. A lumbar puncture (LP) for analysis of the cerebrospinal fluid should be obtained in patients with features concerning for meningitis. Bacterial meningitis will demonstrate a low glucose, high protein, and a significant elevation of WBCs in the CSF that is often > 100 WBCs/µL. In patients presenting with features concerning for elevated ICP, such as altered mental status, neurological deficits, seizures, or papilledema, a CT scan of the head should be obtained prior to LP due to the risk of herniation. If a CT scan is required, antibiotics should be administered first, as a delay worsens the prognosis and mortality risk. The recommended antibiotics include ceftriaxone and vancomycin in patients beyond the neonatal period and up to the age of 50.
A 66-year-old man presents with chest pain. He has a history of hypertension and coronary artery disease. His symptoms started acutely 30 minutes prior to arrival and have been constant and worsening since. He rates the pain a 10/10 and describes it as sharp and severe. He is also now complaining of numbness in his left hand. Vital signs are significant for HR 120 bpm, BP 210/120 mm Hg, RR 20/min, and oxygen saturation of 99%. The examination is remarkable for a man who is in severe distress and grossly diaphoretic. He has clear breath sounds bilaterally and is tachycardic. A neurologic exam reveals 3/5 strength in the left upper extremity compared to 5/5 strength in the right upper extremity with sensory deficits in the left upper extremity compared to the right. The exam is otherwise unremarkable. ECG is performed and is shown above. What is the next step in management? Activate catheterization lab Administer esmolol CT angiography of the chest Noncontrast CT of the head
Administer esmolol Aortic Dissection Risk factors: advancing age, male sex, HTN, Marfan syndrome Sx: acute onset of "ripping" or "tearing" chest pain or back pain PE: asymmetric pulses or SBP difference of > 20 mmHg CXR: widened mediastinum Dx: CT angiography or transesophageal echocardiogram (TEE) Treatment: reduce BP and HR (beta-blockers), pain control, emergency surgery (Type A dissection)Type A: involves ascending aortaType B: involves only descending aorta
A 14-year-old boy presents to the ED with severe abdominal pain. You notice a painful papule on his right arm where he said he was bit by something when he was working in the garage. On physical exam, his abdomen is rigid. Vital signs are BP 140/90 mm Hg, HR 120 bpm, RR 18/min, and T 98.4°F (36.7°C). Which of the following is the most appropriate next step in management? Administer intravenous antivenin Administer intravenous lorazepam Consult general surgery Obtain computed tomography of the abdomen
Administer intravenous lorazepam This patient is exhibiting signs and symptoms consistent with black widow spider envenomation. The black widow spider can be identified by the red hourglass shape on its ventral abdomen. Signs and symptoms of envenomation include autonomic instability with hypertension and tachycardia, nausea, vomiting, muscle cramps, severe abdominal pain with rigidity mimicking a surgical abdomen, fasciculations, ptosis, and headache. Management of black widow spider envenomation is almost universally supportive with analgesia and benzodiazepines such as lorazepam for cramping. Antivenin should be reserved only for the most severe symptoms due to the risk of anaphylaxis and serum sickness as it is derived from equine serum
A 45-year-old man with no significant past medical history presents to the emergency department after being sent in by his primary care physician. He was undergoing a routine work physical when the above rhythm strip was obtained. The patient is asymptomatic. His vital signs include T 37.0°C, HR 50 bpm, BP 120/80 mm Hg, and oxygen saturation 100% on room air. Laboratory work is within normal limits. What is the most appropriate next step in management? Administer atropine Admit the patient for continuous cardiac monitoring Discharge with repeat ECG in 2-4 weeks Initiate transcutaneous pacing
Admit the patient for continuous cardiac monitoring The patient's rhythm strip shows that he has a Mobitz type II second-degree atrioventricular heart block and will require admission for continuous cardiac monitoring (with transcutaneous pacing pads). Patients with Mobitz type II second-degree AV block are at high risk for progressing into complete heart block, requiring continuous cardiac monitoring and possible permanent pacemaker placement.
A 48-year-old man presents to the clinic with intermittent chest pain and pressure over the past two months. The pain is of an aching quality and does not radiate. Nothing seems to aggravate or alleviate the pain. He can also feel his heart beating rapidly during these episodes, which he does not experience at other times. He notes feeling a bit short of breath at random times of the day and has a mild cough. He denies fever, nausea or vomiting, dizziness, or loss of consciousness. He has no history of blood clots and he has not been recently hospitalized. Vital signs include BP 143/90 mm Hg, HR 90 bpm, RR 18/minute, and O2 saturation 95%. There is jugular venous distention of 4 cm bilaterally. Cardiac auscultation reveals a distant S1 and S2 without murmurs, rubs, or gallops. Auscultation of the lungs reveal mild crackles at the lung bases. An electrocardiogram is performed which reveals diffuse low QRS voltage. No ST changes or Q waves are seen. A chest X-ray shows enlargement of the cardiac outline in a water bottle-like shape and blunting of the costophrenic angles. The patient's troponin and D-dimer levels are negative. Which pattern on the electrocardiogram suggests that the patient's condition is progressing to cardiac tamponade? Alternating high and low QRS complex amplitudes Complete absence of P waves and an irregularly irregular rhythm Polymorphic ventricular tachycardia with gradual increases and decreases of QRS complex amplitudes around the isoelectric line ST segment elevations and PR depressions in almost all leads
Alternating high and low QRS complex amplitudes The patient's physical examination demonstrates that he is most likely suffering from worsening congestive heart failure, as indicated by bilateral jugular venous distention, evidence of pulmonary edema, and his symptoms of chest pain. The chest pain is most likely a result of pericardial effusion, which can occur as a complication of right-sided heart failure. Heart sounds can be distant on auscultation. The chest X-ray may show an enlarged cardiac silhouette, and if the volume of effusion is large, the heart may appear to "sag" and take on a "water bottle" shape. The electrocardiogram in patients with a pericardial effusion is often normal or shows sinus tachycardia with low QRS voltage. If the effusion progresses to cardiac tamponade, electrical alternans can occur - alternating high and low QRS complex amplitudes between beats. This pattern appears on the ECG due to the heart's swinging back and forth inside a large fluid-filled pericardium, which causes the ventricular electrical axis to change each beat
A 21-year-old man presents with a cough and fevers. He describes the cough as productive of green sputum that began 3 days ago, and he has had intermittent fevers since then. He reports no myalgias, chest pain, or dyspnea. He has no pertinent past medical history and is up to date on vaccinations. He attends college at the local university. His vital signs include a BP of 110/69 mm Hg, HR of 92 bpm, T of 101.1°F, and RR of 18 breaths/min. Chest X-ray reveals a left lower lobe hazy infiltrate without an effusion. Which of the following is the most appropriate initial antibiotic choice for this patient? Amoxicillin Azithromycin Ceftriaxone Levofloxacin
Amoxicillin Community-acquired pneumonia (CAP). Approximately one-half of CAP cases do not have a causative organism isolated, but among those that do, Streptococcus pneumoniae is the most common, followed by viruses, then atypical bacteria such as Mycoplasma pneumoniae, Chlamydia pneumoniae, and Legionella pneumophila. Symptoms include cough, fever, fatigue, dyspnea, and pleuritic chest pain but can vary widely. Extrapulmonary symptoms are common in atypical causes of pneumonia. Klebsiella pneumoniae is classically associated with those with alcohol use disorder. Haemophilus influenzae has decreased incidence since the introduction of the vaccine but can be seen in patients who are immunocompromised or older and in unvaccinated children. Symptomatic treatment includes antipyretics, volume repletion, and supplemental oxygen. Antibiotic therapy depends on patient risk factors, including previous history of MRSA or Pseudomonas pneumonia, age, comorbidities, and local resistance patterns. In healthy patients with CAP, first-line initial therapy is high-dose amoxicillin. Other options are doxycycline or a macrolide (if local resistance is < 25%). For those with comorbidities, including COPD, heart disease, and chronic kidney disease, combination therapy is recommended with amoxicillin-clavulanic acid or a cephalosporin plus a macrolide or doxycycline. Fluoroquinolones are acceptable as single-coverage agents in these patients but should be avoided due to side effects, such as tendinopathy and QT prolongation, and significant interactions with other drugs.
A 43-year-old woman presents with a report of 11 days of sinus congestion and nasal drainage. The patient reports that she had a sore throat, cough, and runny nose in the preceding week that initially seemed to improve but now seems to be worsening for the last 7 days. She notes that now her right maxillary area is painful, and her upper teeth hurt on the right side. The postnasal drainage has a foul taste. She reports she has a similar infection every year around this time. Temperature is 101.2°F (39°C), blood pressure is 124/69 mm Hg, heart rate is 73 bpm, and oxygen saturation is 99% on room air. Her examination reveals tenderness to percussion over her sinuses. Which of the following is the most appropriate initial therapy? Amoxicillin-clavulanate 875 mg/125 mg orally twice daily Azithromycin 500 mg orally once daily Levofloxacin 500 mg orally once daily Trimethoprim-sulfamethoxazole 160 mg/800 mg orally twice daily
Amoxicillin-clavulanate 875 mg/125 mg orally twice daily Diagnosis of acute sinusitis is based on a history of two or more of the following symptoms: nasal congestion, inability to smell, facial pain or pressure, and mucopurulent nasal discharge. Symptoms may also include foul breath, tooth pain, fever, and worsening facial pain when bending forward. Symptoms commonly present after a recent viral upper respiratory tract infection. Most infections are self-limited and will resolve within 2 weeks, regardless of treatment with antibiotics. Antibiotics are reserved for patients with severe symptoms at onset, worsening of symptoms after initial improvement, or persistent symptoms for > 10 days. Amoxicillin (875 mg orally twice daily or 500 mg orally three times daily) or amoxicillin-clavulanate (875 mg/125 mg orally twice daily or 500 mg/125 mg orally three times daily) are good first-line agents for acute bacterial sinusitis since the common causative organisms are Streptococcus pneumoniae, Haemophilus influenzae, Moraxella catarrhalis, Staphylococcus aureus, and Streptococcus pyogenes.
A 13-year-old girl presents complaining of sudden onset right-sided facial swelling associated with a foul taste in her mouth. Physical exam reveals an underweight teenage girl. She has difficulty opening her mouth and the oropharynx is clear without swelling or exudates. There is swelling and tenderness to the right preauricular space, extending beyond the angle of the mandible. Temperature is 39.7°C, blood pressure is 114/64 mm Hg. Which of the following is the most appropriate treatment for this patient? Amoxicillin/clavulanate Ampicillin/sulbactam Cephalexin Moxifloxacin
Ampicillin/sulbactam This patient's presentation is consistent with acute bacterial parotitis. Risk factors for parotitis include old age, sialoliths, neoplasms, Sjögren's syndrome, HIV, anorexia, and bulimia. This patient's fever is an indicator of severe infection that is unlikely to respond to oral antibiotics. Parotitis Patient will be older History of dehydration or intubation Pain and tenderness with trismus and dysphagia PE will show a firm, erythematous swelling of the pre- and postauricular areas that extends to the angle of the mandible Most commonly caused by Staphylococcus aureus Treatment is intravenous antibiotics
A 30-year-old healthy woman presents to the emergency department for severe perianal pain and difficulty with defecation for one day, which is not associated with fever, bleeding, or decreased oral intake. Examination is significant for a purple-colored mass external to the anal verge that is firm and tender. Which of the following is the best management of this patient's condition? Anesthetize the lesion and evacuate with cruciate incision Anesthetize the lesion and evacuate with elliptical incision Discharge with recommendations for stool softeners, sitz baths, and primary care follow-up Surgery consult for drainage in the operating room
Anesthetize the lesion and evacuate with elliptical incision This patient's clinical presentation is consistent with a thrombosed external hemorrhoid. Constipation, increasing age, liver disease, and pregnancy are all related to increased risk of hemorrhoids. External hemorrhoids occur due to dilation of veins distal to the anal verge and, due to their sensory innervation, can become exquisitely painful and tender. Thrombosis in these veins due to hemostasis can occur, leading to the blue-purplish discoloration, pain, and swelling. cutely thrombosed hemorrhoids (for less than 48 hours) with severe symptoms in healthy patients can be treated by anesthetizing the lesion and evacuating with an elliptical incision.
A 60-year-old man presents to the emergency department after a fall in which he hit his head, causing severe neck flexion. On examination, he has 4/5 strength and normal sensation of his upper extremities but has 0/5 strength of his lower extremities and absent sensation to pain below his clavicles, but he is able to sense vibration and tell which direction his toes are pointed. Which of the following is the most likely cause of this patient's symptoms? Anterior cord syndrome Brown-Séquard syndrome Central cord syndrome Spinal shock
Anterior cord syndrome This patient's presentation is consistent with anterior cord syndrome, an incomplete spinal cord syndrome caused by hyperflexion of the spine or thrombosis of the anterior spinal artery. This presentation leads to damage to the corticospinal and spinothalamic tracts while not damaging the posterior columns. Findings on examination include loss of motor function and pain and temperature sensation below the level of the injury with preservation of position and vibratory sensation.
A 17-year-old football player presents after sustaining a knee injury during his game earlier in the evening. He has no medical problems and is not on any medications. He states he was running with the ball when he felt a "pop." Immediately after he had severe pain in his right knee, was unable to walk initially, and now has a noticeable limp. On exam, the patient has intact strength with knee extension and flexion. He has intact range of motion with exacerbation of symptoms during range of motion. There is no laxity noted with valgus or varus testing, nor is there appreciable joint line tenderness. With the patient's knee flexed to 30 degrees, there is increased translation of the proximal tibial anteriorly compared to posterior translation. What is the most likely diagnosis? Anterior cruciate ligament tear Lateral collateral ligament tear Meniscus tear Patellar tendon rupture
Anterior cruciate ligament tear Anterior Cruciate Ligament (ACL) Tear History of a quick stop of movement and change of direction while running Reports pop and swelling Most accurate clinical tests are Lachman test (most sensitive) and anterior drawer test Imaging modality of choice: MRI (highly sensitive and specific) Treatment is surgical intervention
A 19-year-old man presents to the emergency department with pain and difficulty moving his right shoulder. He was playing football and was tackled when his arm was extended to catch the ball. The other player landed on his right arm. On exam, the patient is holding his arm in the abducted position and externally rotated near his body. He is unable to move his right shoulder. His acromion is prominent and the shoulder has lost its round appearance. What is the diagnosis? Acromioclavicular joint separation Anterior shoulder dislocation Inferior shoulder dislocation Posterior shoulder dislocation
Anterior shoulder dislocation An anterior dislocation is caused by a direct blow to the externally rotated, abducted, and extended arm. Fractures include Bankart lesions, Hill-Sachs deformities, and glenoid tuberosity fractures. Bankart lesions occur when a bony fragment is avulsed from the glenoid labrum during a dislocation. A Hill-Sachs deformity is a cortical depression in the humeral head created by the glenoid rim during dislocation. Computed tomography is not generally indicated to evaluate for shoulder dislocation. Ultrasound is less likely to diagnose fractures associated with dislocations but is useful to confirm the diagnosis and successful reduction of the dislocation.
An 18-year old woman is diagnosed with a spontaneous first trimester complete abortion at 10 weeks. Her blood type is found to be O-negative. She does not know the father's blood type. Which of the following is true? Anti-D immune globulin must be administered within 72 hours to be effective in preventing Rh-isoimmunization Subsequent Rh-positive fetuses are not at risk for complications after Rh-isoimmunization The patient is not at risk for Rh-isoimmunization The patient should be given a minimum of 300 micrograms of anti-D immune globulin in order to prevent Rh-isoimmunization
Anti-D immune globulin must be administered within 72 hours to be effective in preventing Rh-isoimmunization Anti-D immune globulin must be administered within 72 hours to be effective in preventing anti-Rh antibody formation secondary to Rh-isoimmunization. This transplacental hemorrhage can occur during term delivery, threatened miscarriage, spontaneous miscarriage, termination of pregnancy, amniocentesis, following abdominal trauma, or during surgery for treatment of an ectopic pregnancy. To prevent this from occurring, patients are given anti-D immune globulin (Rho-GAM)
A 65-year-old man presents in acute respiratory distress. He has a history of hypertension, hyperlipidemia, and coronary artery disease. Paramedics state they were told his symptoms started abruptly 20 minutes prior to arrival. Vital signs include a BP of 180/60 mm Hg, HR 120 bpm, RR 35/min, and oxygen saturation of 85%. The physical exam is remarkable for a man in severe respiratory distress with rales noted bilaterally. Accessory muscle use, tachypnea, and tachycardia are also noted. A high-pitched blowing murmur occurring after S2 is heard best along the left sternal border in the third intercostal space. Which of the following is the most likely diagnosis? Aortic regurgitation Aortic stenosis Mitral stenosis Tricuspid regurgitation
Aortic regurgitation Chronic Aortic Insufficiency Risk factors: bicuspid aortic valve, rheumatic heart disease, infectious endocarditis, Marfan syndrome Sx: long asymptomatic phase followed by HF symptoms: dyspnea, orthopnea, paroxysmal nocturnal dyspnea, fatigue PE: Widening pulse pressure Bounding water-hammer peripheral pulses Head bobbing with systole (de Musset sign) Prominent nail pulsations (Quincke pulse) Hyperdynamic apical pulse displaced to the left Diastolic blowing murmur best heard along LSB Systolic or diastolic thrill or murmur heard over the femoral arteries (Duroziez sign) In severe AR, a mid-diastolic murmur (Austin-Flint murmur) Diagnosis is made by echo to determine the severity of regurgitation Most commonly caused by abnormal leaflets or proximal aortic root Tx: valve replacement surgery for symptomatic patients and asymptomatic patients with EF < 55% or enlarged LV, or progressive LV changesLimited role of pharmacologic therapy in patients preparing for surgery or non-surgical candidates - diuretics, ACEI, ARB, CCB (amlodipine)
A 72-year-old man presents with worsening dyspnea on exertion over the last two weeks. Today, he became very lightheaded while walking to get the mail and had to sit down because he thought he might pass out. Past medical history includes hypertension, diabetes mellitus and previous coronary artery stenting. On examination, he is in no acute distress. He has a few bibasilar rales on lung auscultation and a harsh crescendo-decrescendo systolic ejection murmur that radiates to the carotid arteries. What is the most likely cause of his symptoms? Acute coronary syndrome Acute mitral regurgitation Aortic stenosis Atrial fibrillation
Aortic stenosis Aortic stenosis is most commonly due to calcific degeneration and is seen in older patients with coronary artery disease. Left ventricular hypertrophy develops as a compensatory mechanism to maintain cardiac output. This can result in ventricular dysfunction and left atrial enlargement. Most patients with aortic stenosis are asymptomatic until the stenosis is severe. The most common presenting complaints include dyspnea with exertion or decreased exercise tolerance, presyncope or syncope, and angina. Physical examination reveals a crescendo-decrescendo systolic murmur that radiates to the carotids and is heard best at the second intercostal space. Surgical valve replacement is the only effective treatment of severe aortic stenosis.
Which finding on chest radiograph is most commonly associated with reactivation of latent pulmonary tuberculosis? Apical-posterior upper lobe infiltrate Diffuse interstitial reticulonodular infiltrates Left lower lobar infiltrate Multiple patchy nonsegmental infiltrates
Apical-posterior upper lobe infiltrate Tuberculosis (TB) Caused by Mycobacterium tuberculosis Risk factors: HIV, immigration from an endemic area, immunosuppression, malnutrition Primary TB Usually asymptomatic and progresses to latent TB with no intervention CXR: often normal, hilar adenopathy, Ghon focus Latent TB Asymptomatic Screening: tuberculin skin test (TST) or interferon-gamma release assay (IGRA) Tx: rifampin for 4 months, INH-rifampin daily for 3 months, or INH-rifapentine weekly for 3 months. Alternative is INH for 9 months or 6 months. Reactivation TB Sx: fever, night sweats, weight loss, productive cough, hemoptysisCXR: upper lobe infiltrates, apical cavitary lesions Dx: sputum smears for acid-fast bacilli (AFB) x3, sputum or tissue culture for AFB (gold standard) Tx: rifampin, INH, pyrazinamide, ethambutol (RIPE) for 6 months - Monitor LFTs, add vitamin B6 (prevent peripheral neuropathy due to INH)
A 66-year-old woman presents for acute headache. She has a history of hypertension and migraines. She states the headache started suddenly after walking into a movie theater. She has associated nausea and vomiting and states she has never had anything like this in the past. Physical examination is remarkable for the finding noted above. What is the best first step in the treatment of the patient above? Apraclonidine Laser iridotomy Mannitol Pilocarpine
Apraclonidine Acute Angle-Closure Glaucoma Sx: acute unilateral pain and vision loss, headache, vomiting, and seeing halos around lights PE: cloudy cornea and fixed mid-dilated pupil Dx: increased IOP Tx: emergent ophthalmology evaluation, topical beta-blockers (timolol), topical alpha-agonists (apraclonidine), miotics (pilocarpine), carbonic anhydrase inhibitors (acetazolamide), iridotomy
A 67-year-old man with a history of coronary artery disease and congestive heart failure presents to the emergency department with 3 days of palpitations and fatigue. A rhythm strip is obtained and shown above. Which of the following is the most likely diagnosis? Atrial fibrillation Atrial flutter Junctional escape rhythm Multifocal atrial tachycardia
Atrial flutter Atrial Flutter Associated with COPD, PE, thyrotoxicosis, mitral valve disease, alcohol Sx: palpitations, dyspnea, presyncope Rapid atrial rate: 250-300 beats/minOften presents with 2:1 AV block and HR 150 Notable ECG feature: sawtooth pattern Treatment: cardioversion if unstable, rate control, ablation of foci
A 4-year-old nonverbal boy with a developmental delay presents to the ED with intermittent lower abdominal pain. He is afebrile but tachycardic and appears to be in significant pain. His abdominal exam is benign. His genital exam reveals an elevated left testicle in a horizontal lie and an absent cremasteric reflex. The on-call urologist is consulted and says he will arrive in 30 minutes. What is the next most appropriate management step? Attempt lateral-to-medial manual detorsion Attempt medial-to-lateral manual detorsion Provide pain control until urology evaluation Transfer to a tertiary care center
Attempt medial-to-lateral manual detorsion Testicular torsion is a urologic emergency because there is a 50% loss rate of the affected testis due to delays in diagnosis, presentation, and treatment. It has a bimodal age distribution around infancy and again during adolescence. Patients with cryptorchidism have a tenfold increase in torsion risk. When the testis rotates along its long axis within the scrotum due to inadequate fixation of the tunica vaginalis to the spermatic cord, the spermatic vessels become twisted. This creates acute unilateral scrotal pain, lower abdominal pain, nausea, vomiting, an elevated testis in a horizontal lie, a swollen and discolored hemiscrotum, and an absent cremasteric reflex. Ideally, a torsed testis should be detorsed (either manually or surgically) within 6 hours of symptom onset. If surgical resources are not available in a timely fashion, providers should attempt medial-to-lateral manual detorsion. This is performed by rotating the affected testis in an outward circular motion as if opening a book or turning a door handle.
A 16-year-old boy presents to the local urgent care with cough and chest pain. He states his symptoms started 14 days ago and have been constant since. He reports no improvement or worsening of symptoms, nor any nausea, vomiting, or diarrhea associated with his cough. His past medical history is unremarkable. The chest pain is described as a sharp retrosternal pain that is worsened with coughing. Vital signs and physical examination are both normal. Chest X-ray is performed and shown above. What is the most likely causative organism? Haemophilus influenzae Legionella Mycoplasma pneumoniae Streptococcus pneumoniae
Atypical pneumonia is classified by the causative organisms, clinical course, and X-ray findings compared to the classic "typical pneumonia." Atypical pneumonia caused by Mycoplasma pneumoniae, also known as walking pneumonia, occurs year-round and presents as a subacute respiratory illness with cough, sore throat, and headache. It is also frequently associated with retrosternal chest pain. The diagnosis is often made clinically based on the constellation of symptoms and signs. The chest X-ray may show patchy infiltrates rather than dense lobar consolidation. Mycoplasma pneumoniae, as well as other atypical organisms, lack a cell wall so they do not respond to beta-lactam antibiotics and require the use of macrolides or respiratory fluoroquinolones.
A 35-year-old woman presents to the emergency department with complaints of fatigue, weakness, and shortness of breath. Symptoms started about a month ago when she began to have trouble keeping her eyes open towards the end of the day. Over the last few days she has noted overall weakness and shortness of breath. Vital signs including a respiratory rate of 16 breaths/minute and 96% SpO2. You note bilateral ptosis and clear lung sounds. Ice bags were applied to the eyes for two minutes. After the ice bags were removed, the distance between her upper and lower eyelids had improved by greater than 2 mm. Which of the following describes the pathophysiology of the most likely underlying diagnosis? Autoimmune destruction of acetylcholine receptors on the postsynaptic membrane Neurodegenerative lesion formation in the anterior horn cells Toxin irreversibly bound to presynaptic membrane preventing acetylcholine release Toxin reversibly bound at the presynaptic membrane preventing acetylcholine release
Autoimmune destruction of acetylcholine receptors on the postsynaptic membrane Myasthenia gravis is a rare disorder known for a bimodal onset, with the first peak among women 20 to 40 years of age and a second peak among men 50 to 70 years old. It involves autoimmune destruction of acetylcholine receptors on the postsynaptic membrane at the neuromuscular junction. This results in complement-mediated destruction of the total number of receptors along with autoantibodies competing with acetylcholine to bind to the remaining receptors. Therefore, with repeated stimulation and fewer sites available, fatigue develops. Fatigue and muscular weakness are the hallmarks of myasthenia gravis. Ocular symptoms are often the first manifestation, with ptosis worsening by the end of the day. Muscular weakness can also affect respiration along with the bulbar muscles causing dysarthria or dysphagia.
A 62-year-old woman with a history of COPD presents with shortness of breath. She states she has a chronic cough, but this week, she has had increased sputum production and worsening dyspnea with activity. She reports no prior hospitalizations for COPD. On examination, she speaks in complete sentences and has diffuse expiratory wheezing with moderate air movement. Her vital signs are within normal limits other than a respiratory rate of 22/min. Chest X-ray is negative for acute changes. She is treated with nebulized albuterol and ipratropium and oral prednisone with significant improvement such that she requests discharge. In addition to a short course of steroids and inhaled bronchodilators, which of the following should be prescribed? Amoxicillin Azithromycin Levofloxacin No antibiotics are indicated
Azithromycin The Global Initiative for Chronic Obstructive Lung Disease (GOLD) defines an exacerbation as an acute change in one or more of the following cardinal symptoms: cough (frequency or severity), sputum production (volume or character), and dyspnea. Infection, either viral or bacterial, accounts for the majority of acute exacerbations. Inhaled beta-adrenergic agonists and anticholinergic agents (e.g., ipratropium) are first-line medications given either through the use of a metered-dose inhaler or nebulized treatment. Systemic steroids are also recommended. Antibiotics are recommended for patients with moderate to severe exacerbations, defined as having acute worsening of at least two cardinal symptoms or requiring mechanical ventilation. In this case, the patient has both increased sputum production and worsening dyspnea. Either a macrolide (e.g., azithromycin) or doxycycline would be an appropriate choice of antibiotic for this patient upon discharge.
A 65-year-old man with hypertension presents to the emergency department for a progressive skin nodule on his face for several months. Examination reveals a 1 cm skin-colored lesion with raised pearly margins and visible vessels on his left cheek. He reports no other associated symptoms or lesions. Which of the following is the most likely diagnosis? Basal cell carcinoma Kerion Melanoma Squamous cell carcinoma
Basal cell carcinoma Basal cell carcinoma is the most common of the nonmelanotic skin cancers, present in older patients on sun-exposed areas such as the head, neck, and scalp. It will typically begin as a small raised lesion with smooth pearly edges that will progress in size and may show telangiectatic vessels
A 41-year-old man has been weight lifting to compete in an upcoming bodybuilding competition. A weight lifting partner provided him with an injection to help maximize building muscle mass. During a recent intense weight lifting session, the patient felt a sudden pop in his right arm. It was immediately difficult for him to flex and supinate his arm at the level of the elbow. He decided to see a doctor after no improvement with several days of rest. On physical exam, he has significant bruising of the elbow and there is a bulge in his right upper arm. Which of the following is most likely involved in the injury presented? Biceps tendon Brachioradialis tendon Supinator tendon Triceps tendon
Biceps tendon The biceps tendon is commonly injured in middle aged body builders who also use anabolic steroids. It is more common to tear the proximal biceps tendon, however, both distal and proximal bicep tendon ruptures appear clinically similar. The clinical finding appears as a deformity in which the biceps tendon retracts and coils, giving the appearance of a palpable mass. The patient will also exhibit weak flexion and supination of the injured arm.
A 70-year-old woman with a history of polymyalgia rheumatica presents with a left-sided headache, fatigue, and jaw claudication. She is febrile to 100.7°F. When shining a light in the patient's right eye, both eyes constrict, but when the light is moved to the left eye, both eyes dilate. Her blood work shows ESR 83 mm/h and CRP 22 mg/dL. Which of the following is most likely to confirm the suspected diagnosis? Biopsy Computed tomography angiography Funduscopy Ocular ultrasound
Biopsy Giant cell arteritis, or temporal arteritis, is a systemic vasculitis that affects medium- to large-sized arteries, with the most common vessel involved being the temporal artery. he disease usually affects women over the age of 50 and is associated with polymyalgia rheumatica. Symptoms include myalgias, fatigue, anorexia, headache, jaw claudication, and fever. If disease has progressed to involve the optic nerve circulation, an afferent pupillary defect may be present, as seen in this patient. The gold standard for diagnosis is temporal artery biopsy. In the acute setting, patients with temporal arteritis may have an elevated CRP and ESR. Treatment is corticosteroids, with the route and dose varying depending on extent of the disease. Importantly, patients should initiate corticosteroid treatment as soon as there is suspicion for temporal arteritis, prior to obtaining a biopsy, given the time-sensitive nature of the disease.
A 60-year-old woman presents to the emergency department for progressive, bilateral hand and finger pain. While evaluating her, which of the following would be most suggestive of a diagnosis of osteoarthritis? Bony, hard swelling of the distal interphalangeal joints Pain and swelling localized to the metacarpophalangeal joints Periarticular bone loss on X-ray Stiffness that is worse in the morning and improves with movement
Bony, hard swelling of the distal interphalangeal joints Osteoarthritis is a chronic, progressive disease of joint destruction leading to worsening pain and limitation of use. It most commonly presents as symmetric, polyarticular, mild joint pain, and stiffness that worsens with use and ultimately leads to joint deformity and loss of locomotion in advanced disease. It most commonly affects the distal interphalangeal (DIP) joints, the thumb, knees, and hips. Heberden nodes, or bony, hard swelling of the distal interphalangeal joints, are nearly pathognomonic for this disease process. Diagnosis may be made clinically by the location and timing of symptoms, however, imaging can also be used to assist in equivocal cases. Plain radiographs will show narrowing of the joint spaces (due to cartilage loss) and osteophytic lesions. Treatment of osteoarthritis consists of weight loss and exercise as well as nonsteroidal anti-inflammatory drugs (NSAIDs) to alleviate symptoms.
A 6-year-old girl presents to the emergency department with a cough that has been worsening over the last 8 days. Her cough is dry, intermittent, and comes in bursts. She has had some post-tussive vomiting but no fever, nasal congestion, rhinorrhea, sore throat, or wheezing. Her exam is only remarkable for subconjunctival hemorrhages and a patient who appears exhausted. Which of the following is the most likely diagnosis? Bordetella pertussis Respiratory syncytial virus Seasonal allergies Viral-induced bronchospasm
Bordetella pertussis Bordetella pertussis is a gram-negative coccobacillus that causes severe coughing. The vaccine for pertussis is protective but not completely, and it also wanes after several years. The illness starts insidiously in the first 2 weeks, known as the catarrhal stage, with sneezing, lacrimation, and low-grade fevers. Coughing marks the onset of the paroxysmal stage, during which the cough worsens for 2 to 6 weeks. The classic cough is a "machine-gun burst" of uninterrupted coughing between breaths followed by a loud whoop as they inhale across a still partially closed glottis. Many patients have post-tussive vomiting and evidence of severe coughing (e.g., subconjunctival hemorrhages) but lack fevers, nasal congestion, rhinorrhea, sore throat, or any other common upper respiratory signs. Exhaustion is universal in pertussis. Treatment is with azithromycin, which prevents further spread but does not reduce the severity of illness in symptomatic patients unless given in the catarrhal stage.
Injury to which vessel or vessels is likely responsible for this patient's abnormal computed tomography scan? Bridging veins Intracerebral arterioles Middle meningeal artery Subarachnoid veins
Bridging veins The mechanism of injury causing an acute subdural hematoma is a sudden acceleration-deceleration of brain parenchyma relative to the dural structures, which causes tearing of vessels called the bridging veins, connecting veins from the cortical surface to the dural sinus. The blood dissects the plane between the dura mater and the arachnoid. Acute subdural hematomas are less than 24-72 hours old, creating hyperdense (white), crescent-shaped lesions on computed tomography (CT) scan that can cross the suture lines. Subacute subdural hematomas are generally between 24-72 hours and two weeks old and present a challenge to identify on CT scan since the lesions are isodense.
Causes of Hemoptysis BATTLE CAMP
Bronchitis/Bronchiectasis Aspergilloma Tumor Tuberculosis Lung Abscess Emboli (pulmonary) Coagulopathy Autoimmune disorders/AV malformation/Alveolar hemorrhage Mitral stenosis Pneumonia
A 23-year-old woman with a history of type 1 diabetes mellitus, anxiety, anorexia nervosa, and nicotine dependence is diagnosed with major depressive disorder. Her body mass index is 18 kg/m2. Which of the following antidepressants should be avoided in this patient? Bupropion Buspirone Mirtazapine Trazodone
Bupropion Major depressive disorder is diagnosed by the presence of five or more of the following symptoms for at least 2 weeks: depressed mood, sleep disturbance, anhedonia, feelings of guilt or worthlessness, decreased energy, difficulty concentrating, weight changes, loss of appetite, psychomotor agitation or depression, or suicidal ideations. The first-line treatment for major depressive disorder is cognitive behavior therapy and selective serotonin reuptake inhibitors (SSRI). Other pharmacotherapy that may be used include serotonin and norepinephrine reuptake inhibitors (SNRI), tricyclic antidepressants, monoamine oxidase inhibitors, and atypical antidepressants such as mirtazapine, trazodone, and bupropion. Bupropion should be avoided in patients with anorexia or bulimia nervosa due to the risk of seizures. Other adverse effects include tachycardia, insomnia, and headache. Bupropion is also used for smoking cessation.
A 48-year-old woman presents with progressive neck pain over the last six weeks. The pain radiates down her posterior arm to the dorsum of her forearm. She reports numbness and tingling of the forearm and third digit. On examination, she has mild weakness with pronation of the forearm and decreased triceps reflex. What is her likely diagnosis? C5 radiculopathy C6 radiculopathy C7 radiculopathy C8 radiculopathy
C7 radiculopathy Cervical radiculopathy is often the result of compression of a cervical nerve root, either related to a disk herniation or degenerative changes in the spine. Posterolateral disk herniation is most commonly seen and results in neck pain and headache. The pattern of radiation of pain, dysesthesias, and weakness corresponds to the particular nerve root that is involved. C7 is the most commonly affected nerve root and results in pain that radiates down the posterior arm to the dorsum of the forearm and third digit. Sensory abnormalities involve the forearm and third digit. There may be weakness with forearm extension and pronation and a decreased triceps reflex. MRI is the test of choice for diagnosis.
A 60-year-old woman is diagnosed with a small bowel obstruction. She does not report any prior surgeries. Which of the following diagnoses is the most likely cause of her small bowel obstruction? Cancer Intussusception Stricture Volvulus
Cancer Small Bowel Obstruction History of prior abdominal or pelvic surgery. The most common risk factor associated with SBO is adhesions from previous surgery, followed by cancer. Bilious vomiting PE may show high-pitched bowel sounds X-ray will show dilated bowel, air fluid levels, stack of coins or string of pearls sign Diagnosis is made by imaging Treatment is NPO, IVF, NGT, surgery
A 60-year-old man presents to the emergency department for a painful hernia. The hernia has been present for years and has been easily reducible, but today it became red, exquisitely tender to palpation, and associated with nausea. Examination is significant for a heart rate of 105 bpm and palpation of a 3 cm mass in the right inguinal crease with overlying redness. Which of the following is the most appropriate management of this patient's condition? Admission for pain control and serial abdominal exams Cefoxitin intravenously and emergent surgical consultation Outpatient surgery follow-up Pain control and manual reduction
Cefoxitin intravenously and emergent surgical consultation Inguinal Hernias Bimodal: < 1 and > 40 years old Direct Protrudes directly through Hesselbach triangle and medial to the inferior epigastric artery (IEA)Bulge decreases upon reclining Indirect Most common typeProtrudes through internal ring, lateral to IEA Mnemonic: MDs don't lie Medial to IEA: direct Lateral to IEA: indirect Strangulation risk: indirect > direct Nonreducible hernia: emergent surgery consultation
23-year-old man presents with a syncopal event while running. His ECG is shown above. Physical examination is unremarkable as are a CBC and basic metabolic panel. What management is required for the patient? Activate the cardiac catheterization lab Cardiology consultation and admission Discharge with cardiology follow-up Obtain CT scan of the chest with contrast
Cardiology consultation and admission This patient presents with exertional syncope and an ECG concerning for hypertrophic cardiomyopathy requiring emergent cardiology evaluation. Hypertrophic cardiomyopathy is a genetic disease leading to an asymmetric hypertrophy of the left ventricle. Hypertrophy results in left outflow tract obstruction. The average age at diagnosis is 30-40 years and often presents as syncope or sudden death during exertion. Physical exam may reveal a loud S4 gallop and crescendo-decrescendo midsystolic murmur. The murmur is increased by Valsalva. About 90% of patients with hypertrophic cardiomyopathy will have ECG abnormalities including left ventricular hypertrophy, abnormal Q waves (often described as deep and narrow in the precordial leads), T wave inversion, and left atrial enlargement. Patients with angina, near-syncope, syncope, or dysrhythmias should be hospitalized for further management with a cardiologist
A patient is found to have a serum ionized calcium level of 3.0 mg/dL. Which of the following findings is expected on evaluation? Carpal spasm while checking blood pressure Confusion Shortened QTc on ECG Tall T waves on ECG
Carpal spasm while checking blood pressure Hypocalcemia Sx: seizures, paresthesias PE: Chvostek sign (facial muscle contraction), Trousseau sign (carpopedal spasm), hyperreflexia Labs: serum Ca2+ < 8.5 mg/dL ECG:↑ QT interval Tx: IV replacement for calcium ≤ 7.5 mg/dL (1.9 mmol/L) or tetany, seizure, carpopedal spasm, prolonged QT
A 40-year-old woman presents to the emergency department with fatigue. She also notes increased shortness of breath when she goes on her daily runs which is unusual for her. Physical exam reveals bilateral nonpitting edema and dry skin. Which of the following is a complication of this disease process? Carpal tunnel syndrome Diarrhea Pulmonary edema Pulmonary hypertension
Carpal tunnel syndrome Hypothyroidism is a systemic disease process caused by a decrease in thyroid hormone, which results in a generalized slowing of metabolic processes throughout the body. Skin manifestations result from an accumulation of matrix glycosaminoglycans, resulting in edema of the face and lower extremities, also known as myxedema. Edema in the lower extremities is nonpitting in contrast to the edema seen in congestive heart failure. The cardiopulmonary effects of hypothyroidism can result in a decrease in heart rate and myocardial contractility, respiratory muscle fatigue, and decrease respiratory compensation to hypercapnia and hypoxemia. The decrease in cardiopulmonary metabolism results in decreased exercise tolerance and shortness of breath on exertion. Hypothyroidism can lead to both central nervous system and peripheral nervous system complications. Myxedema coma is a severe form of hypothyroid that leads to central nervous system dysfunction as well as other manifestations of hypothyroidism, including hypothermia, hypercapnia, and hyponatremia. The most common peripheral neuropathy seen in patients with hypothyroidism is carpal tunnel syndrome. Patients may also present with musculoskeletal symptoms such as myalgias and proximal muscle weakness.
A 4-day-old neonate born at 38 weeks gestation following an uncomplicated pregnancy and delivery presents to the emergency department with his father due to bilateral eye redness and discharge. On exam, the patient has bilateral conjunctivitis and copious amounts of purulent discharge from both eyes. What is the treatment for the most likely diagnosis? Cefotaxime Cephalexin Ciprofloxacin Erythromycin
Cefotaxime Neisseria gonorrhoeae Conjunctivitis Newborns ≤ 5 days old, sexually active adults Hyperpurulent discharge Corneal ulceration or perforation Admission Topical + IV antibiotics
A 28-year-old man presents with dysuria. He reports no associated penile discharge or any other symptoms. He also reports having had two new sexual partners in the last 3 weeks. Vital signs and physical examination are unremarkable, with a negative genitourinary exam. Urinalysis shows 2 WBC/hpf with 1+ bacteria. Which of the following antibiotic regimens represents the most appropriate treatment? Ceftriaxone 1 g IV and azithromycin 1 g PO Ceftriaxone 500 mg IM and doxycycline 100 mg BID x 7 days Cephalexin 500 mg PO Ciprofloxacin 500 mg PO
Ceftriaxone 500 mg IM and doxycycline 100 mg BID x 7 days Gonococcal Urethritis Patient presents with purulent urethral discharge and dysuria Labs will show gram-negative diplococci Diagnosis: Gold standard: NAAT PCR test is both sensitive and specific Most commonly caused by Neisseria gonorrhoeae CDC, December 2020: ceftriaxone 500 mg IM only (+ doxycycline 100 mg BID for 7 days if Chlamydia not ruled out)
A 26-year-old woman presented to the ED with pelvic pain. On exam, she had a friable cervix and adnexal tenderness. She was discharged with NSAIDs for pain and several years later she developed infertility. Which of the following treatment regimens may have prevented her infertility? Ceftriaxone IM + azithromycin PO x 1 day Ceftriaxone IM + doxycycline PO x 14 days Clindamycin PO x 7 days Levofloxacin PO x 5 days
Ceftriaxone IM + doxycycline PO x 14 days This patient presented with cardinal symptoms of pelvic inflammatory disease (PID). PID is infection of the upper reproductive tract in woman, typically in an ascending pattern from the vagina or cervix secondary to STD. The cardinal symptoms of PID include lower tract inflammation (cervicitis or friable cervix) and pelvic tenderness or mass. This disease process is also associated with tubal-factor infertility owing to destruction of the ciliated epithelial cells of fallopian tubes. Most recent treatment guidelines for PID per the CDC call for doxycycline BID for 2 weeks (with or without metronidazole PO BID for 2 weeks) with ceftriaxone IM.
A 57-year-old man has polyuria, nocturia, and polydipsia. About three months prior, he was involved in a motor vehicle collision which resulted in severe head trauma. He also has an increased craving for ice water. His 24-hour urine output is 150 mL/kg per day. Which of the following is the most likely diagnosis? Central diabetes insipidus Diabetes mellitus Nephrogenic diabetes insipidus Primary polydipsia
Central diabetes insipidus Diabetes insipidus (DI) results from insufficient antidiuretic hormone (ADH) production by the hypothalamus (central diabetes insipidus) or from impaired ADH action in the kidney (nephrogenic diabetes insipidus). ADH deficiency results in large amounts of dilute urine. In central diabetes insipidus, an insufficient amount of ADH is released in response to physiologic stimuli. Causes of central diabetes insipidus include head trauma (as in this case), congenital, and genetic disorders. This patient most likely suffered traumatic damage to the pituitary stalk resulting in central diabetes insipidus.
An 18-year-old man presents to the emergency department after overdosing on medication. He is somnolent, only arousing to noxious stimuli. His vital signs are normal and his pupils are midposition and briskly reactive. You are unable to elicit nystagmus. He is not tremulous, his reflexes are normal, and he displays no clonus. Toxicity from which of the following agents is most likely responsible for his presentation? Chlordiazepoxide Lithium Methadone Quetiapine
Chlordiazepoxide Coma or central nervous system depression with essentially normal vital signs and midposition pupils are characteristic findings in isolated benzodiazepine toxicity, making chlordiazepoxide the most likely culprit in this case. Most benzodiazepine overdoses are best treated with supportive care, flumazenil in benzodiazepine-naive patients (can precipitate seizures)
A 44-year-old woman presents with right upper quadrant pain. She has a history of obesity and tobacco use. She states her pain started tonight after eating dinner. She has had symptoms similar to this in the past, but tonight the symptoms have persisted and are worse than usual. Vital signs are within normal limits with the exception of an HR of 115 bpm. Physical examination is remarkable for severe right upper quadrant tenderness. Labs are remarkable for WBC of 15,000 cells/microliter and an ALT and AST of 300 U/L and 250 U/L, respectively. Total bilirubin and alkaline phosphatase are normal. Ultrasound is performed and is demonstrated above. What is the most likely diagnosis? Cholangitis Cholecystitis Choledocholithiasis Cholelithiasis
Cholecystitis Cholecystitis Sx: colicky, steadily increasing RUQ or epigastric pain after eating fatty foods PE: Murphy sign, Boas sign (hyperaesthesia, increased or altered sensitivity, below the right scapula) DiagnosisInitial: U/SGold standard: HIDA Most commonly caused by obstruction by a gallstone Treatment is cholecystectomy
A 38-year-old woman presents to the emergency department with nausea and vomiting. She estimates her last menstrual period was about 8 weeks ago and is concerned that she is pregnant. Her bedside transabdominal pelvic ultrasound is shown above. Which of the following can arise as a complication of this patient's condition? Calcium oxalate nephrolithiasis Choriocarcinoma Hypothyroidism Pelvic inflammatory disease
Choriocarcinoma Gestational trophoblastic disease is a spectrum of conditions caused by neoplastic mutation of the trophoblastic cells of the placenta, and it includes complete and partial hydatidiform moles as well as choriocarcinoma. Complete moles are the most common variety of gestational trophoblastic disease. Risk factors include advanced maternal age and prior gestational trophoblastic disease. Symptoms include vaginal bleeding in the first or second trimester, hyperemesis, and pelvic pain or pressure. Gestational hypertension before 20 weeks of gestation is highly suggestive of gestational trophoblastic disease. Laboratory examination may reveal beta-hCG levels significantly above expected levels for gestational age. Physical examination is typically notable for a uterine size greater than dates. The classic ultrasound finding in complete molar disease is the sonographic "snowstorm" or "bag of grapes," which describes the appearance of multiple cystic structures in the uterus. Definitive management is dilation and curettage. Rarely, gestational trophoblastic disease can be complicated by the development of gestational trophoblastic neoplasia, which includes invasive moles, choriocarcinoma, and trophoblastic tumors.
A 29-year-old woman, who is 7 weeks pregnant by dates, presents to the emergency department with severe pelvic pain. Laboratory workup is notable for a serum hCG level of 7,000 IU/L. Ultrasound shows the image above. What is the appropriate next step in management? Administer antibiotics Administer misoprostol Consult obstetrics and gynecology Discharge patient with expectant management
Consult obstetrics and gynecology The patient is presenting with a pseudosac with surrounding free fluid, which is concerning for an ectopic pregnancy Ectopic Pregnancy Risk factors include prior ectopic, PID, tubal surgery, IUD Symptoms include abdominal pain, pelvic pain, amenorrhea, or vaginal bleeding Labs will show positive pregnancy test and lower than expected serum beta-hCG levels Diagnosis is made by ultrasoundDefinitive dx: gestational sac with a yolk sac or embryo outside of the uterine cavityFree fluid with debris is suggestive to ruptured ectopic Most commonly located in a fallopian tube Treatment is methotrexate or surgery
A 65-year-old man with alcohol use disorder presents to the ED with abdominal pain. When asked about the location of the pain, he points to his epigastric region and describes radiation to the back. He reports nausea without vomiting. His vital signs are within normal limits. He has been hospitalized multiple times in the past for similar episodes. An abdominal CT scan with IV contrast is unremarkable. Laboratory results are as follows: White blood cells: 6,600/µL Aspartate aminotransferase: 32 U/L Alanine aminotransferase: 29 U/L Lipase: 143 U/L Ammonia: 26 mmol/L INR: 1.1 Which of the following is the most likely diagnosis? Abdominal aortic aneurysm Alcoholic hepatitis Chronic pancreatitis Small bowel obstruction
Chronic pancreatitis Chronic Pancreatitis Most common cause: chronic excessive alcohol use CT, AXR: calcifications Glucose intolerance Abdominal pain radiating to back Malabsorption Steatorrhea
A 65-year-old man presents with dysuria and urinary urgency. He has a history of hypertension, chronic kidney disease, and hyperlipidemia. He states over the past 6 months, he has had difficulty with urination and wakes up frequently to go to the bathroom. He relates there has been some associated back and perineal pain, and he has not had any penile discharge. He reports having only one sexual partner. Vital signs show a temperature of 37.8°C but are otherwise unremarkable. Exam is remarkable for suprapubic pain and exquisite tenderness to prostate palpation. Which of the following is the most appropriate treatment option for this patient? Admission for IV levofloxacin Ceftriaxone IM once, with azithromycin daily for 5 days Ceftriaxone IM once, with doxycycline bid for 14 days Ciprofloxacin daily for 28 days
Ciprofloxacin daily for 28 days Acute Bacterial Prostatitis Sx: fever, chills, perineal or pelvic pain, and dysuria PE: firm and exquisitely tender prostate Most common causes< 35 years old: N. gonorrhoeae, C. trachomatis> 35 years old: E. coli Treatment< 35 years old: ceftriaxone IM and doxycycline> 35 years old: fluoroquinolone or TMP-SMX for 4 weeks Avoid vigorous prostatic massage, which can lead to septicemia
A 23-year-old woman who wears contacts presents to the emergency department with left eye pain that started while she was cutting wood with a power saw. On examination, her left eye is injected and tearful, and she prefers to keep her eyelid closed. Fluorescein staining reveals the findings seen in the above image. Which of the following is the most appropriate initial treatment? Ciprofloxacin ophthalmic solution Erythromycin ophthalmic ointment Prednisolone ophthalmic solution Trimethoprim-polymyxin B ophthalmic solution
Ciprofloxacin ophthalmic solution Analgesia with topical proparacaine or tetracaine can be used to aid in examination. Management of simple corneal abrasions includes oral pain control, prophylactic topical antibiotics (as needed for larger abrasions), and topical nonsteroidal anti-inflammatory medications for analgesia. In contact lens wearers, antibiotics should be chosen that cover Pseudomonas, including ciprofloxacin ophthalmic solution.
A 25-year-old woman presents to the emergency department with a sore throat and subjective fever for 2 days. Vital signs show HR 104 bpm, BP 126/80 mm Hg, RR 16 breaths/minute, and T 38.7°C. What historical feature of this patient's illness would most suggest viral pharyngitis as a more likely etiology rather than bacterial pharyngitis caused by group A Streptococcus? Conjunctivitis Headache Myalgia Pharyngeal exudate
Conjunctivitis Pharyngitis and tonsillitis are primarily caused by a virus, most often rhinovirus. Symptoms of pharyngitis include oropharyngeal swelling, inflammation of the oropharynx sometimes associated with vesicular or petechiae lesions, and difficulty swallowing. Unfortunately, these symptoms are not specific for viral pharyngitis, and it can be difficult to differentiate viral pharyngitis from bacterial pharyngitis. While viral pharyngitis is more often associated with other respiratory symptoms, those symptoms can also be found in patients with bacterial pharyngitis. The Centor criteria are used to determine the likelihood of having group A streptococcal pharyngitis. Criteria include tonsillar exudates, tender anterior cervical lymphadenopathy, fever, and absence of cough. Unfortunately, even patients with viral pharyngitis can meet these criteria. Studies have shown that if patients have rhinorrhea or conjunctivitis with pharyngitis, they are more likely to have a viral etiology rather than a bacterial etiology
A 60-year-old woman presents to the emergency department with knee pain. She has a history of diabetes, peripheral arterial disease, CAD, and hypertension. She reports no obvious trauma to the affected knee and states this is worse than her typical arthritis pain. Vital signs are remarkable for a temperature of 39°C, HR 120 bpm, BP 100/60 mm Hg, RR 22/min, and SpO2 of 97%. Physical exam is notable for a warm, edematous, and erythematous right knee. The patient has severe pain with range of motion testing and holds the knee in slight flexion. An arthrocentesis is performed. Which group of findings on synovial fluid analysis is most expected in this patient? Cloudy, yellow in appearance, 15,000 WBCs/µL with a predominance of neutrophils, negative birefringent crystals present on analysis Cloudy, yellow in appearance, 24,000 WBCs/µL with a predominance of neutrophils, positive birefringent crystals present on analysis Cloudy, yellow in appearance, 85,000 WBCs/µL with a predominance of neutrophils, no crystals present on analysis Transparent, yellow in appearance, 3,000 WBCs/µL without a predominance of neutrophils, no crystals present on analysis
Cloudy, yellow in appearance, 85,000 WBCs/µL with a predominance of neutrophils, no crystals present on analysis An acutely hot, swollen, and tender joint with restriction of movement is considered to be bacterial septic arthritis until proven otherwise. Although no clinical pattern is diagnostic of septic arthritis, joint pain, joint swelling, and fever are the three most common features, occuring in over 50 percent of patients with bacterial septic arthritis. Classically, a synovial fluid WBC count greater than 50,000 WBCs/µL with a predominance of neutrophils was consistent with septic arthritis. The treatment is IV antibiotics directed at staphylococcal and streptococcal species, including coverage for methicillin-resistant Staphylococcus aureus (MRSA). Orthopedic surgery consultation should be obtained for joint irrigation and further management if the synovial fluid has positive indicators for infection.
A 35-year-old man presents to the urgent care clinic with excruciating right wrist pain. He is an avid soccer player and fell on his outstretched hand during a game. Physical examination shows swelling, deformity, and dorsal displacement of the distal wrist. Palpation of the ulnar and radial pulses is normal. Which of the following is the most likely diagnosis? Colles' fracture Scaphoid fracture Smith fracture Ulnar shaft fracture
Colles' fracture Colles Fracture FOOSH Dinner-fork deformity Dorsal displacement Reduce, splint, orthopedic follow-up
A 32-year-old woman with no significant medical history presents with right-sided pleuritic chest pain. Her only medication is an oral contraceptive. Vital signs are within normal limits, and her physical exam, electrocardiogram, and chest X-ray are unremarkable. Urine pregnancy test is negative. Blood work is significant for a D-dimer of 800 ng/mL. Which of the following is the best next step in management? Computed tomography angiography scan Discharge the patient Lower extremity duplex ultrasound V/Q scan
Computed tomography angiography scan Patients for whom there is a suspicion of pulmonary embolism can be risk-stratified using the Wells criteria for pulmonary embolism. Patients are grouped into either low-, moderate-, or high-risk categories. The pulmonary embolism rule-out criteria (PERC) can be used to effectively rule out a pulmonary embolism in low-risk patients who do not have any of the following: age ≥ 50 years, heart rate ≥ 100 bpm, oxygen saturation < 95% on room air, unilateral leg swelling, hemoptysis, recent surgery or trauma, prior pulmonary embolism or deep venous thrombosis (DVT), or hormone use. Patients who have any of the above criteria, such as this patient on oral contraception, cannot be ruled out for a pulmonary embolism and should have D-dimer testing. Patients with an elevated D-dimer should undergo CT angiography of the chest, as it is the gold standard for diagnosing pulmonary embolism.
A 72-year-old man with no significant past medical history except type 2 diabetes mellitus presents to the ED with a complaint of general illness. Which combination of findings prior to treatment is most likely to indicate the diagnosis of hyperosmolar hyperglycemic state? Confused mental status, normal perfusion, normal skin turgor, blood glucose 400 mg/dL, negative urine ketones, serum sodium of 140 mmol/L Confused mental status, signs of hypoperfusion, poor skin turgor, blood glucose 700 mg/dL, trace urine ketones, serum sodium of 140 mmol/L Normal mental status, normal perfusion, slightly decreased skin turgor, blood glucose 350 mg/dL, negative urine ketones, serum sodium of 148 mmol/L Responsive appropriately to pain only, systolic blood pressure of 70 mm Hg, very poor skin turgor, blood glucose 50 mg/dL, strongly positive urine ketones, serum sodium of 166 mmol/L
Confused mental status, signs of hypoperfusion, poor skin turgor, blood glucose 700 mg/dL, trace urine ketones, serum sodium of 140 mmol/L Hyperosmolar hyperglycemic state (HHS) classically presents with clinical signs of extreme dehydration and decreased level of consciousness. In elderly type 2 diabetes patients, declining renal function causes less renal clearance of glucose than normal. Decreased insulin sensitivity also contributes to hyperglycemia. The resulting increased serum osmolality causes fluid shifts from intracellular to extracellular, but subsequent severe osmotic diuresis creates an often profound state of dehydration and hypoperfusion.
A 35-year-old man presents to the emergency department reporting chest pain for 3 days. He says that the pain is slightly to the left of his sternum and worse with inspiration and while playing basketball. It is a dull pain that can sometimes feel like a stinging sensation. He reports no shortness of breath, cough, nausea, diaphoresis, and fever. His blood pressure is 139/74 mm Hg, heart rate is 82 bpm, temperature is 98.5°F (37°C), and oxygen saturation is 98% on room air. On exam, he has normal heart and lung sounds, but he winces when leaning forward. There is distinct pain noted with palpation over the left anterior medial third rib. His ECG is as shown above. What is the most likely diagnosis? Acute pericarditis Costochondritis Pulmonary embolism Xiphodynia
Costochondritis Costochondritis is caused by inflammation of the costal cartilages or their articulation with the sternum. Pain is often made worse with breathing, movement, and particularly, horizontal arm flexion. The pain may be intermittently sharp or dull. The upper costal cartilages are the most frequent site of inflammation. Diagnosis is made during the physical exam when the pain is reproduced by palpating the specific area over the costochondral junction(s) involved. Findings of at least two of the following are 63% sensitive and 79% specific for a diagnosis of costochondritis: localized muscle tension, stinging pain, lack of cough, and pain reproduced by palpation.
A patient presents to the emergency department after a skiing collision and is diagnosed with a fibular head fracture. Which of the following findings on neurovascular examination would most likely be seen with this injury? Decreased sensation between the first and second toe Decreased sensation to the calf Weakened dorsalis pedis pulse Weakness with plantar flexion of the ankle
Decreased sensation between the first and second toe Common Peroneal Neuropathy Injury at the proximal fibula, ex leg cast or prolonged lying PE: foot drop, numbness in web space between first and second toes Tx: ankle splint to keep foot dorsiflexed
A 48-year-old woman presents to the emergency department with reports of shortness of breath and cough for 3 weeks. She states she initially had fevers, but they stopped 1 week ago. She has had progressive dyspnea on exertion since that time. On examination, she has a temperature of 99.5°F, heart rate of 105 beats per minute, blood pressure of 110/60 mm Hg, respiratory rate of 14 breaths per minute, and oxygen saturation of 95% on 2 liters of oxygen. Lung sounds are diminished at the left base, and there is a normal S1 and S2 but with mild tachycardia. Which of the following additional physical examination findings is expected? Decreased tactile fremitus Distended neck veins Tracheal shift to the right Tympany to percussion
Decreased tactile fremitus Pleural Effusion PE will show ↓ breath sounds + dull percussion + ↓ tactile fremitus CXR will show blunting of the costophrenic angle Can also use CT or US to diagnose Most common causes Transudate: heart failure Exudate: infection > malignancy, PE Management includes treating underlying cause, therapeutic thoracentesis, tube thoracostomy Light criteria are used to differentiate between transudative and exudative effusions
A 45-year-old man presents with diarrhea. He reports he has had diarrhea for the past 2 months. Additionally, he reports bloating and notes that the diarrhea is pale, foul smelling, and voluminous. Chart review reveals a weight loss of 10 pounds since his last visit 6 months ago. He reports no recent fevers, bloody bowel movements, nausea, or vomiting. A tissue transglutaminase antibody test is positive. Which of the following extraintestinal manifestations might be seen in a patient with the most likely diagnosis? Ankylosing spondylitis Decreased prothrombin time Dermatitis herpetiformis Leukocytosis
Dermatitis herpetiformis This patient has celiac disease. This disease is caused by an autoimmune process in reaction to dietary gluten that causes mucosal inflammation, crypt hyperplasia, and villous atrophy. This combination leads to malabsorption, which can cause many symptoms of vitamin deficiencies. Dermatitis herpetiformis is a direct manifestation of celiac disease that causes vesicobullous, pruritic lesions on the elbows, knees, and buttocks.Celiac disease is managed primarily with dietary changes.
A 56-year-old man with von Willebrand disease presents to the emergency department with gross hematuria for 2 days. Vital signs include a heart rate of 90 bpm and blood pressure of 115/75 mm Hg. Laboratory workup is notable for a hemoglobin of 12.0 g/dL, platelets of 150,000/µL, and an international normalized ratio of 1.0. What is the most appropriate treatment to address this patient's bleeding? Cryoprecipitate Desmopressin Platelet transfusion Von Willebrand factor replacement
Desmopressin Desmopressin is the most appropriate first-line treatment for minor bleeding in patients with von Willebrand disease. Von Willebrand disease is the most common inherited bleeding disorder and is characterized by absent or defective von Willebrand factor. Von Willebrand factor is synthesized and released by vascular endothelial cells and plays a role in hemostasis by assisting with platelet adhesion and acting as a carrier for factor VIII. Typically, patients will present with a history of frequent mucosal bleeding such as epistaxis, gingival bleeding, and frequent bruising. GI bleeding and menorrhagia can also be seen. Patients may present with an prolonged activated partial thromboplastin time (seen in patients with low factor VIII levels) but will have normal platelet levels and prothrombin time. First-line treatment for mild bleeding is desmopressin, which acts by promoting the release of von Willebrand factor from endothelial cells. If desmopressin does not work or if the patient presents with life-threatening bleeding, von Willebrand factor (recombinant) or cryoprecipitate can be administered.
Which of the following is more common in ulcerative colitis than in Crohn disease? Development of malignancy Fissures Intestinal obstruction Small intestine involvement
Development of malignancy Ulcerative Colitis Sx: bloody diarrhea, crampy abdominal pain, tenesmus PE: continuous mucosal inflammation, always involving the rectum, absence of perianal involvement (prevalent in Crohn) Extraintestinal findings: uveitis, erythema nodosum, peripheral arthritis, sacroiliitis, ankylosing spondylitis Tx:options depend on severity and location of disease Mild-moderate: mesalamine, topical or oral steroids, 5-ASA Severe: IV steroids +/- topical steroids initially, then anti-TNF or anti-integrin, colectomy for refractory cases (curative) Complications: toxic megacolon, ↑ colon cancer risk
A 21-year-old male college student with no medical history presents to the emergency department for elbow pain. He states that while studying for final exams, he noticed that his elbow has become more swollen over the last week. His vital signs include a temperature of 37°C, a heart rate of 70 bpm, and a blood pressure of 115/75 mm Hg. On exam, you note the swelling above. You are able to fully extend the elbow with only minimal discomfort. What is the most appropriate next step in management? Apply a sugar tong splint Discharge with NSAIDs and changes in study habits Obtain an ultrasound of the elbow Perform an arthrocentesis
Discharge with NSAIDs and changes in study habits Olecranon Bursitis Swelling, pain ↓ active ROM Full passive ROM Aspiration if infection or crystal disease suspected Tx: NSAIDs
A 32-year-old healthy man presents to the emergency department with 10 days of a cough productive of green sputum. He was seen in the emergency department 1 week ago for fever, dyspnea, and a similar cough. At that time, a chest X-ray was negative and he was diagnosed with an upper respiratory infection. Today, his vital signs are unremarkable, a cardiopulmonary exam is benign, and a repeat chest X-ray is negative for acute pulmonary disease. Which of the following is the best management for this patient? Albuterol as needed Dextromethorphan as needed Oral amoxicillin Oral prednisone
Dextromethorphan as needed This patient most likely has acute bronchitis. The most common causes of acute bronchitis are respiratory viruses. The pathophysiology behind acute bronchitis is a bronchial epithelial infection that results in inflammation and thickening of the bronchial and tracheal mucosa. This leads to airflow obstruction and bronchial hyperresponsiveness, manifesting as a cough, dyspnea, and wheezing. Patients may also acutely present with a fever. The cough due to acute bronchitis may be with or without sputum production and last anywhere from 5 days up to 4 weeks, a distinguishing clinical feature that is used to differentiate acute bronchitis from an acute upper respiratory infection. Patients with acute bronchitis may also have abnormal pulmonary function testing, such as a reversible decrease in forced expiratory volume in 1 second, attributed to bronchial hyperresponsiveness. Diagnosis is made clinically, and treatment is focused on symptomatic relief with expectorants, antihistamines, mucolytics, and antitussives such as dextromethorphan.
A 23-year-old woman presents with fever, cough, and shortness of breath. Her vital signs are normal, and her chest X-ray shows nonspecific infiltrative changes. Which of the following is the optimal treatment? Admit with IV ceftriaxone Discharge with PO amoxicillin/clavulanic acid Discharge with PO doxycycline Discharge with PO levofloxacin
Discharge with PO doxycycline Typical organisms causing CAP include Streptococcus pneumoniae, Haemophilus influenzae, Staphylococcus aureus, group A Streptococci, Moraxella catarrhalis, anaerobes, and aerobic gram-negative bacteria. However, community-acquired pneumonia is often caused by atypical organisms which include Legionella spp, Mycoplasma pneumoniae, Chlamydia pneumoniae, and Chlamydia psittaci. Clinical features and imaging results are insufficient at distinguishing typical versus atypical CAP. This patient's mild presentation may suggest atypical CAP. Treatment for CAP should cover both typical and atypical organisms. Mycoplasma pneumoniae is the most common cause of atypical pneumonia in the United States. Doxycycline would be an appropriate treatment for this patient in the outpatient setting as it covers both typical and atypical organisms.
A 75-year-old man with type 2 diabetes mellitus presents with an unwitnessed syncopal episode. He reports he stood up to go to the bathroom, felt dizzy, and subsequently lost consciousness. The patient is currently feeling okay but notes a mild headache and reports a recent "stomach flu" a few days ago that he is recovering from. Physical exam is notable for dry mucous membranes, a small hematoma to the left temple without obvious skull depression, and no focal neurologic findings. ECG shows normal sinus rhythm, and a head CT is negative for acute bleeding. Which of the following findings is most consistent with the most likely cause of this patient's syncope? Diastolic blood pressure decrease of 15 mm Hg when standing Hemoglobin of 5 g/dL Lactate of 4.5 mg/dL Urine sodium concentration of 25 mEq/L
Diastolic blood pressure decrease of 15 mm Hg when standing Orthostatic hypotension secondary to dehydration is often a diagnosis of exclusion in syncope patients. Volume depletion generally results from loss of water and sodium from the gastrointestinal tract, renal system, skin loss, or third spacing. This patient reports a recent gastrointestinal problem, which likely indicates fluid loss via vomiting or diarrhea. In this case, when other causes of syncope such as intracranial hemorrhage and dysrhythmia have been ruled out, dehydration is most likely. Often, laboratory testing is unnecessary in the emergency department to diagnose dehydration, and orthostatic vital signs are a common physical exam finding. A patient is considered orthostatic when they have a decrease in systolic or diastolic blood pressure by greater than 20 mm Hg or 10 mm Hg, respectively, within 2 to 5 minutes of standing
A 30-year-old woman presents with right breast pain. She is 3 weeks postpartum and has been exclusively breastfeeding her child. Her vital signs are within normal limits. On exam, the patient has an area of erythema surrounding her nipple that is tender and warm. No fluctuance is appreciated. An ultrasound confirms the diagnosis. Which of the following is the best treatment for this patient? Ceftriaxone Dicloxacillin Doxycycline Trimethoprim-sulfamethoxazole
Dicloxacillin Mastitis Patient will be a breastfeeding mother Breast erythema, tenderness, fever Most commonly caused by Staph. aureus Management includes cool compresses and analgesics between feedings Antibiotics: dicloxacillin, cephalexin, TMP-SMX (MRSA), clindamycin (PCN allergy) Continue breast feeding to avoid progression to abscess
A 47-year-old man with a history of HIV, who takes his antiretroviral therapies, reports progressively worsening shortness of breath, pedal edema, and orthopnea over the past 2 weeks. Vital signs include an HR of 100 bpm, BP 156/94 mm Hg, RR 24/min, and T 37.6°C. On physical examination, he has mild jugular venous distention and rales on lung auscultation. He has 2+ pitting edema in his lower legs. What will be the most likely findings visualized on echocardiography? Dilation and impaired contraction of both ventricles Increased interventricular septal wall thickness and diastolic dysfunction Nondilated and nonhypertrophied ventricles with impaired ventricular filling Right ventricular free wall replacement with fibrofatty tissue and right ventricle dyskinesis
Dilation and impaired contraction of both ventricles This patient is presenting with signs and symptoms of heart failure. Taking into consideration his young age, lack of previous cardiac history, and history of HIV, his symptoms are most likely due to an underlying cardiomyopathy. The patient in this question has HIV, and the clinician should recall that HIV and antiretroviral therapies, such as zidovudine, didanosine, and zalcitabine, can cause a dilated cardiomyopathy. Additionally, this patient's signs and symptoms are most suggestive of both systolic and diastolic cardiac dysfunction that is leading to congestive heart failure. Therefore, echocardiographic findings will most likely be dilation and impaired contraction of both ventricles. Treatment includes general heart failure therapies, including diuretics, beta-blockers, and angiotensin-converting enzyme inhibitors. Depending on the severity of the disease, patients may also benefit from an implantable defibrillator.
A 10-year-old boy with recurrent nosebleeds due to digital trauma presents to the emergency department for uncontrolled epistaxis. His vital signs are normal, he has no respiratory distress, and he has a steady stream of blood from the right naris that has not stopped with direct nasal pressure. Oxymetazoline spray and continued pressure are not sufficient to stop the bleeding, so right nasal packing is applied with satisfactory hemostasis at 60 minutes. His vital signs are still within normal limits, and his hemoglobin is unchanged from his baseline. Which of the following is the best next step in management? Apply prophylactic packing to left naris and schedule 48-hour outpatient follow-up Discharge with 48-hour follow-up for packing removal Discharge with 48-hour follow-up with prophylactic amoxicillin and ibuprofen for pain Removal of packing with the application of silver nitrate to any residual bleeding vessels
Discharge with 48-hour follow-up for packing removal This patient presents with an anterior nosebleed. Anterior nosebleeds, originating from Kiesselbach plexus, are the cause of approximately 90% of nosebleeds, especially in children and young adults, due to dry mucous membranes, digital trauma, and infection. Patients should be placed in the sniffing position to prevent aspiration of blood products, and initial management should be attempted with vasoconstrictive medications such as oxymetazoline or phenylephrine. Finally, packing may be attempted with commercially available nasal tampons or other prothrombotic oxidized cellulose foams. Once applied, these should be left in place and monitored for one hour. If hemostasis is achieved in one hour and the patient is hemodynamically stable, then the patient may be discharged with 48-hour follow-up for packing removal.
A 50-year-old man with a history of hypertension and tobacco use presents to the emergency department for episodes of chest pain with exertion. He reports several years of stable chest pain that occurs after 10 minutes of walking and resolves with rest. He is currently pain-free. Examination is significant for heart rate of 82 bpm, blood pressure of 145/70 mm Hg, clear lung fields, and no cardiac murmurs. His electrocardiogram is shown above and his initial troponin was 0.0 ng/L. Which of the following is the most appropriate management of this patient's condition? Admission for heparin infusion and urgent cardiology consult Admission for serial troponin testing Discharge with primary care follow-up Emergent cardiac catheterization
Discharge with primary care follow-up Angina Chest pain or pressure with exertion or stress that is relieved by rest Consider noncardiac causes Diagnosis is made by stress test Management lifestyle modifications, beta-blockers, calcium channel blockers, nitrates
A 75-year-old woman with a history of a total hysterectomy presents with abdominal pain. She reports that yesterday she noticed pain in her abdomen that comes and goes along with vomiting and constipation. Additionally, she reports fever and dysuria. She is unsure if she has had any blood in her stool but knows she has had it in the past, and her primary care doctor was not concerned. The physical exam is notable for moderate tenderness to palpation in the left lower quadrant and suprapubic areas without costovertebral angle tenderness, and the rectal exam does not reveal bright red blood or melena. Her urinalysis is positive for many leukocytes without bacteria or nitrites. Her labs are notable for a leukocytosis of 14,000/uL and lactate of 2 mmol/L. Which of the following is the most likely diagnosis? Clostridioides difficile colitis Diverticulitis Ischemic colitis Pyelonephritis
Diverticulitis Diverticulitis Sx: abdominal pain that is localized to the left lower quadrant, fever, nausea, vomiting, and a change in bowel habits PE: localized guarding, rigidity, and rebound tenderness Diagnosis is made by CT with IV contrast: thickened bowel wall, "fat stranding," may show complications - bowel perforation, abscess, fistula, obstruction Consider treatment with supportive care and/or antibiotics based on risk factors and presentation. Antibiotics to cover gram-negative and anaerobic bacteria, bowel rest, and surgery (in severe cases) High-fiber diet can help in prevention
A 55-year-old man presents to the emergency department for rectal bleeding. The patient states he has noticed blood in his stool for the past week, but it has been increasing in volume. This morning, he noticed a large amount of red blood with one of his bowel movements. The patient has a past medical history of obesity, gastroparesis, constipation, and diabetes. He has a 40-pack-year smoking history and drinks three alcoholic beverages every night. His temperature is 99.5°F (37.5°C), blood pressure is 167/108 mm Hg, pulse is 90/min, respiratory rate is 15/min, and oxygen saturation is 99% on room air. Rectal exam reveals bright red blood. There is no tenderness upon palpation of the patient's abdomen. Which of the following is the most likely diagnosis? Colon cancer Dieulafoy lesion Diverticulitis Diverticulosis
Diverticulosis This patient is presenting with a history of constipation and bright red blood per rectum without systemic symptoms or fever, suggesting a diagnosis of diverticulosis. Diverticulosis typically presents with painless bright red blood per rectum. Diverticula typically occur at areas of weakness within the bowel such as where the vasa recta penetrates the circular muscle of the colon and are associated with abnormal colonic motility. Treatment of diverticulosis with bleeding includes keeping the patient NPO ("bowel rest"), administering IV fluids, and monitoring their CBC to track for anemia secondary to bleeding. Most cases resolve spontaneously.
A 38-year-old man with a history of underlying psychiatric disease presents to the emergency department with strange facial and tongue movements. He was recently started on a new medication, but he cannot remember the name. He has difficulty describing his exact underlying psychiatric problem, and the friend with him also does not know the details regarding the patient's chronic medical issues. On exam, the patient is smacking his lips periodically and twisting his tongue in and out of his mouth. He is also holding his head stiffly and tilted to the left. What neurotransmitter imbalance is responsible for the development of this syndrome? Altered calcium-channel gating leading to an excess accumulation of calcium and, thus, hypermetabolism Decreased circulating dopamine and gamma-aminobutyric acid leading to a surplus of acetylcholine and glutamate Dopamine receptor blockade leading to increased cholinergic activity Excess circulating serotonin
Dopamine receptor blockade leading to increased cholinergic activity The clinician should recognize that the patient in this question is presenting with an acute dystonic reaction related to medication use. Dystonic reactions are characterized by involuntary contractions of muscles of the extremities, face, neck, or pelvis that lead to abnormal movements or postures. The most commonly described causative agents include first-generation antipsychotics and antiemetic medications. The abnormal movements are generally attributed to an imbalance in the dopaminergic and cholinergic effects in the basal ganglia. Offending agents are dopamine receptor antagonists. The antagonism leads to a deficit in central dopamine transmission, which leads to excess release of acetylcholine. Acetylcholine has inhibitory effects on movement, leaving patients with twisting, tight dyskinetic movements. Primary treatment is with anticholinergic medications such as diphenhydramine or benztropine.
A 65-year-old man presents with left leg pain. He reports progressive erythema and pain of his left lower leg after cutting it while working on his car. His examination is notable for a 5 x 5 cm area of induration and erythema with yellow purulent drainage. The area is warm to touch and tender to palpation. Which of the following is the most appropriate medication for outpatient management of this patient's condition? Amoxicillin-clavulanate Cephalexin Doxycycline Levofloxacin
Doxycycline Cellulitis is a bacterial infection of the deep dermis and subcutaneous fat tissue, which manifests with localized skin erythema, edema, warmth, and pain. The most common pathogens implicated in cellulitis are beta-hemolytic streptococci and Staphylococcus aureus, including methicillin-resistant S. aureus (MRSA). The treatment of cellulitis is based on whether or not there is associated purulence. Patients with purulent cellulitis should receive empiric coverage for MRSA. Oral antibiotics with MRSA coverage include doxycycline, trimethoprim-sulfamethoxazole, clindamycin, and linezolid. Patients with systemic symptoms requiring admission should receive parenteral antibiotics with coverage for MRSA, which include vancomycin, clindamycin, telavancin, daptomycin, and linezolid.
A 45-year-old previously healthy man presents to the emergency department with a chief complaint of epigastric burning and gnawing pain. While in the waiting room, he has an episode of dark, coffee-ground emesis and continues to vomit while being wheeled to the resuscitation bay. Which of the following is the most likely cause of the patient's symptoms? Duodenal ulcer due to chronic nonsteroidal antiinflammatory drug use Duodenal ulcer due to H. pylori infection Gastric ulcer due to chronic nonsteroidal antiinflammatory drug use Gastric ulcer due to H. pylori infection
Duodenal ulcer due to H. pylori infection Helicobacter pylori Sx: epigastric pain Dx: fecal antigen or urea breath test, can be established on biopsy during EGD Common cause of peptic ulcer disease Tx: quadruple therapy: 14 days of PPI, bismuth, tetracycline, metronidazole. Other regimens possible, depend on local clarithromycin resistance and prior exposure to macrolides. Test of cure in all patients at least 4 weeks after completion of therapy
A 36-year-old man is brought to the emergency department by his family because he has not been sleeping and has been more irritable. The patient says that he feels "like a hundred trillion bucks" and that he wants to be discharged as soon as possible because he is preparing for a meeting with the chief executive officers of five major car brands to discuss his ideas for developing a hovercraft. The family says that the patient recently spent his entire month's paycheck buying gardening tools with the intent of starting an urban farm. What other piece of history is required to make a formal diagnosis? Age of onset of first episode Duration of symptoms of the current episode Medication compliance Presence of previous depressive episodes
Duration of symptoms of the current episode This patient is presenting with symptoms consistent with a manic episode, a component of bipolar disorder. For the diagnosis of bipolar I disorder, patients need a single manic episode, which is characterized by at least one week of profound mood disturbance, with flight of ideas, irrationality, and grandiose thinking being common. A diagnosis of bipolar II disorder requires that patients have had at least one hypomanic episode as well as a major depressive episode (D). Hypomanic episodes can present with similar irritability and flight of ideas, but do not cause significant functional impairment. This patient's delusions and problematic spending are more consistent with manic symptoms.
A 30-year-old man presents to the ED with chest pain for the past 2 days. He describes the pain as sharp and worse with inspiration. He feels most comfortable sitting up and leaning forward. He states that he was recently ill with "flu-like" symptoms and reports intermittent low-grade fevers. Physical examination is significant for a pericardial friction rub on auscultation. Which of the following is most consistent with the suspected diagnosis? Chest X-ray significant for bibasilar infiltrates Chest X-ray significant for cardiomegaly ECG findings of diffuse ST elevation with reciprocal ST depression in leads aVR and V1 ECG findings of ST elevation in leads I, aVL, and V5 -V6 with reciprocal ST depression in leads V1, II, III, and aVF
ECG findings of diffuse ST elevation with reciprocal ST depression in leads aVR and V1 Pericarditis, inflammation of the pericardial sac, is the most common disorder of the pericardium. It may result from viral, bacterial, or fungal infection, malignancy, medications, or systemic illnesses. Symptoms commonly include sharp, pleuritic chest pain that is relieved by sitting up and leaning forward, dyspnea, and low-grade, intermittent fevers. Classically, a pericardial friction rub is appreciated on physical exam. This is a scratchy sound, best heard with the diaphragm of the stethoscope positioned along the lower left sternal border. ECG findings include diffuse, nonanatomic ST segment elevation, ST segment depression in leads aVR and V1, and PR segment depression in multiple leads.
A 28-year-old man presents complaining of intractable pain underneath his cast five days after sustaining a tibia and fibula fracture of his left leg. He reports that he was seen at his orthopedic surgeon's office yesterday for the same pain and was given another prescription for "stronger" pain medicines after an exam. A new cast was applied before discharge, and he is supposed to follow up in six days. However, he insists that the pain medicine is not helping him at all. Physical examination after cast removal reveals paresthesias of the skin between the first two toes, and intense pain with passive flexion of the toe. The anterior aspect of his lower leg feels tense and is tender. What is the most important next step in management? Application of ice and elevation of the extremity Discharge with outpatient orthopedic follow up if delta pressure is ≤ 30 mm Hg Emergent consultation with an orthopedic surgeon Radiograph of the extremity
Emergent consultation with an orthopedic surgeon Acute compartment syndrome is a condition of elevated intramuscular pressures within the confined space of the compartments created by fascias and bones, which results in tissue ischemia and ultimately infarction and irreversible damage of tissue. Normal compartment pressure is between 10-12 mm Hg. It is generally thought that tissue pressures exceeding 20 mm Hg lead to clinically significant reduction of blood flow and oxygenation of the tissues inside the affected compartment. Patients may present with acute compartment syndrome within two hours of injury or up to six days later and classically present with pain out of proportion to physical exam findings. For this reason, increasing analgesic requirements may sometimes be the only clue to developing compartment syndrome. If objective signs are present, tenseness of externally palpable compartments may be the earliest and most objective finding. Passive stretching of involved muscle groups produces intense pain, as does manual compression of an affected compartment. Active movement is likely to be painful as well, while the nerves passing through the affected compartment may be compromised and produce paresthesias. Paresis and muscle weakness are late findings. Pulses are usually maintained unless presentation is particularly late. The key to the management of acute compartment syndrome is early consideration of the diagnosis and emergent consultation with orthopedic surgery as the diagnosis can be made based on the history and physical exam.
A 62-year-old woman with osteoarthritis, diabetes, and obesity presents with acute onset of upper abdominal pain radiating to the back and shoulders. She has had similar pain in the past but it is usually relieved with over-the-counter medications. She is diaphoretic on arrival with HR 115 bpm, BP 95/42 mm Hg, RR 18 bpm, T 38.6°C, and oxygen saturation 98%. She has diffuse upper abdominal tenderness with localized guarding. A portable chest X-ray is shown above. Fluid resuscitation and pain control is ordered. Which of the following is the best next step in management of this patient? Chest CT Emergent surgical consultation Intubation Right upper quadrant abdominal ultrasound
Emergent surgical consultation Peptic ulcer disease (PUD) is most commonly caused by infection with Helicobacter pylori (found in up to 80% of patients with duodenal ulcers) and can result in gastric or duodenal ulcers. The classic symptom of peptic ulcer disease is burning epigastric pain, occasionally with radiation to the back or chest, that occurs a few hours after a meal or wakes a patient from sleep. Associated symptoms include bloating, early satiety, and nausea. Complications of PUD include hemorrhage (most common), perforation, penetration (erosion of an ulcer into an adjacent organ), and gastric outlet obstruction. This patient presents with classic symptoms of perforation, which include epigastric abdominal pain that radiates to the chest, back, or shoulders and is refractory to usual medications. Patients may also present with signs of sepsis such as fever, hypotension, and tachycardia. An upright chest radiograph will show pneumoperitoneum, as with this patient, which should prompt emergent surgical consultation. Aggressive resuscitation and empiric broad-spectrum antibiotics are also indicated.
A 4-year-old girl presents with fever and sore throat. Her symptoms started yesterday evening and became acutely worse today. Her vital signs are HR 165, RR 30, and temp 38.8 °C. She appears anxious and is maintaining a "sniffing" position with her jaw thrusting forward. She has audible stridor without wheezing. What is the most likely diagnosis? Croup Epiglottitis Retropharyngeal abscess Tracheitis
Epiglottitis Sx: rapid onset of fever, muffled voice, dysphagia, respiratory distress PE: stridor, leaning forward, drooling, red swollen epiglottis on direct visualization Imaging: enlarged epiglottis (thumbprint sign) on lateral neck X-ray Most commonly caused by H. influenzae (decrease since Hib vaccine), Strep and Staph species Treatment is airway management and IV antibiotics (third-generation cephalosporin AND antistaphylococcal)
A 59-year-old man with hypertension and coronary artery disease presents to the ED unresponsive. He is placed on a cardiac monitor, which reveals the rhythm shown above. While preparing to defibrillate, which of the following therapies is indicated at this time? Amiodarone 300 mg IV Epinephrine 1 mg IV Magnesium sulfate 4 g IV Procainamide 20 mg/min IV
Epinephrine 1 mg IV Ventricular fibrillation is a disorganized pattern of electrical depolarization that does not generate a pulse or cardiac output. It appears as a fine or coarse irregular pattern with no discernible P waves or QRS complexes. The most common etiology is severe ischemic cardiac disease. Advanced cardiopulmonary life support (ACLS) dictates that the treatment of ventricular fibrillation is early defibrillation. Other ACLS measures include chest compressions and administration of epinephrine 1 mg IV every three to five minutes. Amiodarone 300 mg IV (A) is recommended but is used secondarily to epinephrine in resuscitation.
A 25-year-old woman presents with abdominal pain. She was discharged 3 days ago after undergoing an uncomplicated cesarean section. On the day of discharge, she only had some minor discomfort, but she states that, over the past day and a half, the pain has significantly worsened. Vital signs are remarkable for a temperature of 39°C, HR 120 bpm, BP 95/50 mm Hg, RR 28/min, and SpO2 of 99%. Physical exam is remarkable for severe pelvic discomfort with palpation in the RLQ or LLQ of the abdomen. Her incision is intact without drainage or erythema. Pelvic exam reveals a foul-smelling vaginal discharge and uterine tenderness. Which of the following is the best next step in the patient's management? Administer 500 mg of ceftriaxone and 1 g of azithromycin Consult OB/GYN for incision and drainage Establish IV access and administer clindamycin and gentamicin Prescribe doxycycline with metronidazole and ensure close OB/GYN follow-up
Establish IV access and administer clindamycin and gentamicin Most postpartum infections are identified after hospital discharge. In a postpartum woman with a fever, pelvic infection is presumed until proven otherwise. Risk factors for postpartum endometritis include multiple gestations, cesarean section, younger maternal age, long duration of labor and membrane rupture, internal fetal monitoring, and low socioeconomic level. Clinical features include fever, foul-smelling lochia, leukocytosis, tachycardia, pelvic pain, and uterine tenderness. Admission is needed for those who appear ill, have had a cesarean section, or have underlying comorbidities. Treatment consists of antibiotics and, on some occasions, debridement of necrotic tissue or abscess drainage. The preferred inpatient empiric antibiotics are clindamycin and gentamicin, as this remains the most effective regimen to treat postpartum endometritis.
A 39-year-old woman presents with vaginal bleeding. She reports a history of heavy menstrual periods and has been going through multiple pads an hour for the past one day. Physical exam reveals heavy bleeding with large clots. She is tachycardic with a blood pressure of 92/54 mm Hg. Her pregnancy test is negative, and her hemoglobin is 7.2 g/dL. In addition to fluid resuscitation and transfusion, which of the following medications is most appropriate to help stop the bleeding? Estrogen Misoprostol Nonsteroidal anti-inflammatory Progestin-only contraception
Estrogen Vaginal bleeding in the non-pregnant patient warrants medical attention when there is persistent bleeding necessitating changing pads or tampons every hour for two or more consecutive hours. Menorrhagia occurs when estrogen levels are too high leading to disordered proliferative or hyperplastic endometrium that is prone to abnormal uterine bleeding (AUB). Heavy vaginal bleeding can also result from fibroids, endometriosis, uterine cancer, coagulopathy, birth control, miscarriage, and ectopic pregnancy. on-pregnant patients with acute hemorrhage due to AUB may be given intravenous estrogen to stabilize the endometrium and slow the bleeding. Once bleeding has slowed, an oral taper of combination oral contraceptives (COC) is typically used to ultimately achieve a cyclic pattern of bleeding with estrogen and progesterone.
A 63-year-old man with a history of hypertension presents to the emergency department with dizziness described as a spinning sensation. He also reports feeling off-balance and has been falling to the side. From the following options, what factor found on history or physical examination would make a clinician concerned that this patient has central vertigo? Falling when walking Recurrent episodes lasting minutes to hours Tinnitus in the right ear Unidirectional, horizontal nystagmus
Falling when walking Vertigo is a symptom of vestibular dysfunction and is described as a spinning sensation or a sense of swaying or tilting. Most patients with vertigo will complain of nausea and vomiting, but the vomiting will be less severe in central vertigo. Patients with a central cause usually complain of persistent symptoms rather than episodic symptoms, sometimes lasting for days to weeks. Patients with central vertigo typically have risk factors for cerebral infarction such as hypertension, high cholesterol, and increased age. On physical examination, they will usually have brainstem symptoms, severe gait impairment with an inability to walk, or limb dysmetria. They also typically do not complain of auditory symptoms. If a central cause of vertigo is suspected, then the clinician should begin a cerebral infarction workup to include computed tomography imaging of the head, followed by magnetic resonance imaging of the brain if CT is negative.
A 4-year-old girl presents to the emergency department with vomiting and diarrhea. Which of the following additional symptoms suggests her gastroenteritis is from a bacterial source instead of a viral source? 12- to 48-hour incubation period Fever greater than 40°C Tenesmus Vomiting before diarrhea onset
Fever greater than 40°C Gastroenteritis describes generalized inflammation through the gastrointestinal tract, usually caused by viruses, bacteria, or parasites. The most common symptom is diarrhea, followed by vomiting, sometimes with fevers. Symptoms of the different causes of gastroenteritis overlap significantly, but some historical clues can suggest a likely pathogen. Viral pathogens commonly start with vomiting and are usually short-lived. Rotavirus is the most common cause of gastroenteritis and usually lasts for less than a week. Norovirus usually only lasts three days or less. Clues to a bacterial source of gastroenteritis are a diarrheal onset, high stooling frequency, high fevers greater than 40°C, grossly bloody stools, and severe abdominal pain.
What is the first-line pharmacotherapy for treating a patient with bulimia nervosa? Bupropion Fluoxetine Nortriptyline Sertraline
Fluoxetine (Prozac) Pharmacotherapy has been shown to be efficacious in treating bulimia nervosa and is considered first-line therapy in conjunction with nutritional rehabilitation and psychotherapy. Selective serotonin reuptake inhibitors (SSRIs) have been shown to be most efficacious in treating bulimia nervosa. Fluoxetine is considered the first-line agent because of its efficacy in treating the behavioral and cognitive symptoms of bulimia nervosa, as well as its tolerability. Second-line pharmacotherapy is switching to another SSRI. If the patient fails that as well, third-line pharmacotherapy involves switching the patient to a tricyclic antidepressant (TCA), trazodone, monoamine oxidase inhibitor (MAOI), or topiramate.
A 68-year-old woman is found to have a pulsatile mass on abdominal exam and a CT scan demonstrates a 4.5 cm abdominal aortic aneurysm without signs of rupture. She is asymptomatic with normal vital signs. Which of the following is the most appropriate management of her AAA? Admit to medicine for surgical clearance Emergent operative repair Follow-up for screening ultrasound every year to track progression Reassurance and no further workup
Follow-up for screening ultrasound every year to track progression bdominal Aortic Aneurysm (AAA) Risk factors: male sex, older patients, smoking, HTN Abdominal pain or asymptomatic Physical exam will show pulsatile abdominal massIf ruptured: hypotension Diagnosis is made by US: excellent screening tool ManagementMonitor progression (Society for Vascular Surgery guidelines)4.0-4.9 cm: US annually5.0-5.4 cm: US every 6 months, can also use CT or MRI (MRI is preferred over time due to less radiation) Surgical repair would be indicated if the patient is symptomatic> 5.5 cm or aneurysms with rapid expansion rate: elective surgery The USPSTF recommends one-time screening for AAA by ultrasonography in men aged 65-75 who have ever smoked
A 55-year-old woman with a history of hypertension, diabetes mellitus, and obesity presents to the ED with a productive cough, shortness of breath, and orthopnea. She denies sick contacts, international travel, recent hospitalization or surgery, and estrogen use. Her temperature is 99.4°F, BP is 166/105 mm Hg, HR is 101 beats/minute, RR is 24 breaths/minute, oxygen saturation is 92% on room air. She has normal S1 and S2 heart sounds, with an S3 gallop and crackles in the bilateral lower lung fields. Her chest X-ray is shown above. Her 12-lead ECG shows sinus tachycardia with a rate of 100 bpm without any ST or T wave irregularities. Her WBC count is 12,000 cell/mm3, hemoglobin is 10 g/dL, platelet count is 250 cells/mm3, and troponin 0.04 ng/mL. Which one of the following therapies is indicated at this time? Azithromycin Dobutamine Furosemide Heparin
Furosemide Acute Decompensated Heart Failure Patient presents with exertional dyspnea, orthopnea, paroxysmal nocturnal dyspnea PE will show pitting edema, S3 heart sound Labs will show ↑ BNP Chest X-ray will show cardiomegaly, cephalization, Kerley B lines, effusions Diagnosis is made by echo (most useful study) Treatment: BPAP: ↑ oxygenation, ↓ work of breathing, ↓ preload, afterload Nitroglycerin: ↓ preload, afterload Furosemide: diuresis Hypotension without signs of shock: dobutamine (may worsen hypotension) Severe hypotension with signs of shock: norepinephrine (↑ systemic vascular resistance, ↑ HR, ↑ BP, ↑ myocardial O2 demand)
A 46-year-old woman presents to the emergency department with slight confusion while making breakfast this morning. According to her husband, she was slow to get out of bed and, when moving around the kitchen, she seemed lost. EMS was called to the scene and found her finger stick glucose to be 43. She was given orange juice to drink and some oral glucose paste and is feeling better now. Her husband adds she was recently diagnosed with diabetes type II and started on a new oral medication the name of which he does not recall. Of the following options, which medicine most likely led to this patient's hypoglycemia and would require further monitoring for hypoglycemic events in the hospital? Glipizide Metformin Pioglitazone Sitagliptin
Glipizide Diabetes Mellitus Type 2 Hyperglycemia due to insulin resistance and progressive loss of insulin secretion Risk factors: obesity, abdominal adiposity, family history, lifestyle factors Sx: asymptomatic, polydipsia, polyphagia, polyuria ADA diagnostic criteria: Symptomatic: Random plasma glucose ≥ 200 mg/dL Asymptomatic: Fasting plasma glucose ≥ 126 mg/dL; Glycated hemoglobin (A1C) ≥ 6.5%; Plasma glucose ≥ 200 mg/dL 2 hours after a 75 g glucose load during an OGTT Tx: lifestyle modifications then medication (first-line Rx: metformin) Screen adults aged 35-70 with BMI ≥ 25 kg/m2 every 3 years Those with additional risk factors need annual screening
A 4-year-old girl presents to the emergency department with fatigue. Her history is only remarkable for a resolved cold a few weeks ago. Her heart rate is 160 bpm, blood pressure is 112/76 mm Hg, and oxygen saturation is 99% on room air. She is pale and tachycardic on exam but will wake and interact appropriately. Laboratory work shows a white blood cell count of 18.4 x109/L, hemoglobin of 5.5 g/dL, and platelets of 260 x109/L. She has a positive direct antiglobulin test (i.e., Coombs). Which of the following is the most appropriate next step in management? Bone marrow biopsy Glucocorticoid administration Normal saline bolus Rapid red blood cell transfusion
Glucocorticoid administration Autoimmune hemolytic anemia is when autoantibodies are directed against a patient's own red blood cell membrane antigens. Most patients simply present with fatigue and pallor, but jaundice, fever, and hemoglobinuria are possible. Laboratory tests show profound anemia, usually some leukocytosis, and normal platelet levels. he direct antiglobulin test detects the presence of antibodies or complement adhered to the red blood cell surface and establishes the diagnosis of autoimmune hemolytic anemia. Patients are treated with glucocorticoids (e.g., prednisone).
A 23-year-old woman presents to the ED complaining of severe chest pain. She is pale and diaphoretic. Her vital signs are T 103.2F, HR 124 beats/minute, RR 22 breaths/minute, BP 93/57 mm Hg, and oxygen saturation 96% on room air. She has painful nodules on her fingertips and multiple track marks on her arms. Bedside cardiac ultrasound reveals hyperdynamic activity, no pericardial effusion, and an oscillating right ventricular mass. Blood cultures are most likely to grow which of the following? Gram-negative diplococci Gram-negative rods Gram-positive cocci in clusters Gram-positive cocci in pairs
Gram-positive cocci in clusters S. aureus is seen as gram-positive cocci in clusters on microscopy. Bacterial Endocarditis Risk factors: injection drug use, valvular heart disease Sx: fever, rash, cough, and myalgias PE: fever, Roth spots, Osler nodes, murmur, Janeway lesions, anemia, nailbed hemorrhages, emboli (FROM JANE) Diagnosis is made by echocardiography and Duke criteria Most commonly caused by: IVDA: Staphylococcus aureus, tricuspid Native valve: Staphylococcus aureus, viridans streptococci (most common in previously diseased), mitral Tx: antibiotics GI malignancy: Streptococcus bovis Dental prophylaxis in some cases
An 8-year-old girl presents after falling from the monkey bars onto an outstretched arm at recess. She endorses pain over her distal forearm proximal to the wrist. A plain radiograph of the forearm is shown above. What type of bony injury is most consistent with this patient's X-ray? Bowing fracture Buckle fracture Greenstick fracture Physeal fracture
Greenstick fracture Pediatric Fractures Spiral or Toddler fracture Torsional injury Minimally or nondisplaced Distal tibial metaphysis Casting x 3 weeks Bowing Longitudinal compression Orthopedic consultation Greenstick Axial compression with twisting Cortex fractured on one side Casting and reduction Torus Axial compression Wrinkling or buckling of cortex Distal radius (most common) Casting
A 45-year-old woman presents to the emergency department with bilateral lower extremity weakness. She only has a past medical history of hypertension. She states her symptoms started last week and have progressively worsened. Her symptoms started with paresthesias and have progressed to the inability to walk. Review of systems is positive for an upper respiratory infection two weeks ago that resolved without any complications. Vital signs are unremarkable. Physical examination is remarkable for complete paralysis of the bilateral lower extremities from the knee distally. Patellar and Achilles deep tendon reflexes are absent. Chest exam reveals clear lungs bilaterally without extra heart sounds. Abdominal exam is unremarkable and skin exam does not reveal any new rashes or lesions. What is the most likely diagnosis? Botulism toxicity Guillain-Barré syndrome Miller-Fisher syndrome Tick paralysis
Guillain-Barré syndrome Guillain-Barré syndrome is an acute immune-mediated polyneuropathy that is most often characterized by an acute monophasic paralyzing illness. The classic clinical features include progressive and symmetric muscle weakness that is accompanied by decreased or absent deep tendon reflexes. Weakness generally starts in the lower extremities and ascends. Severe respiratory muscle weakness can occur requiring mechanical ventilation. he initial diagnosis is made on clinical presentation. The diagnosis is then supported if cerebrospinal fluid (CSF) studies and electrodiagnostic studies demonstrate abnormalities. Albuminocytologic dissociation is found on CSF analysis, which is defined as an elevated protein with a normal white count. Treatment focuses on stabilization and supportive measures to ensure intact ABCs and hemodynamic integrity. Patients should also undergo either plasmapheresis or intravenous immune globulin as both of these treatments accelerate the recovery
A 55-year-old man presents to the emergency department with shoulder pain. The patient has a past medical history of hypertension and hyperlipidemia. He states the symptoms started three days ago. He denies any traumatic events leading up to his symptoms. Which positive physical exam finding is most consistent with supraspinatus tendonitis? Hawkins-Kennedy test Homans test O'Brien test Speed Test
Hawkins-Kennedy test Shoulder pain is a common complaint in both the emergency and primary care settings. Rotator cuff pathology and tendinopathy comprise a large portion of patients. Risk factors include repetitive overhead activity, scapular instability or dyskinesis, anatomical variants that cause impingement, and older age. Clinical features include shoulder pain, especially with overhead activity, which is often localized to the lateral deltoid with night pain. On physical examination patient may reveal atrophy of affected muscle if the tendinopathy is longstanding. Range of motion testing, particularly with passive motion above 90 degrees abduction, or pain with internal rotation suggests rotator cuff tendinopathy. The Neer or Hawkins-Kennedy tests are the best tests for impingement, and in combination with the range of motion testing described above, they are highly suggestive of tendinitis. Initial treatment consists of ice, rest, and the use of nonsteroidal anti-inflammatory drugs.
A healthy 30-year-old man presents to the emergency department for a prolonged erection. He reports using cocaine last night while partying, and the erection has been present for 6 hours. Examination reveals tachycardia to 110 bpm and a rigid penile shaft with soft glans penis. Penile blood gas has a pH of 7.10. Which of the following is the most appropriate management of this patient? Aspiration of the glans penis Intracavernosal phenylephrine Intravenous terbutaline Observation for spontaneous resolution
Intracavernosal phenylephrine Priapism Low-flow: venous, painful, emergency Rx: aspiration, intracavernous phenylephrine High-flow: arterial, semierect, painless Rx: observation, arterial embolization Aspiration performed at 2 or 10 o'clock position
Which of the following lab values is most consistent with the diagnosis of disseminated intravascular coagulation? Hemoglobin 13 g/dL, platelets 80,000/µL, D-dimer 300 µg/mL, fibrinogen 150 mg/dL, international normalized ratio 1.3, AST 40 U/L, ALT 35 U/L Hemoglobin 7.5 g/dL, platelets 80,000/µL, D-dimer 2,000 µg/mL, fibrinogen 50 mg/dL, international normalized ratio 4.0, AST 90 U/L, ALT 95 U/L Hemoglobin 8.5 g/dL, platelets 70,000/µL, D-dimer 200 µg/mL, fibrinogen 200 mg/dL, international normalized ratio 1.4, AST 40 U/L, ALT 35 U/L Hemoglobin 8.5 g/dL, platelets 80,000/µL, D-dimer 150 µg/mL, fibrinogen 180 mg/dL, international normalized ratio 1.0, AST 350 U/L, ALT 450 U/L
Hemoglobin 7.5 g/dL, platelets 80,000/µL, D-dimer 2,000 µg/mL, fibrinogen 50 mg/dL, international normalized ratio 4.0, AST 90 U/L, ALT 95 U/L Disseminated intravascular coagulation (DIC) is characterized by inappropriate and widespread activation of the coagulation system, resulting in intravascular thrombin generation, small vessel thrombosis, and consumption of clotting factors and platelets. Concomitant activation of fibrinolysis also occurs, resulting in fibrin breakdown and bleeding. Some conditions associated with DIC include infection, cancer, trauma, liver disease, pancreatitis, pregnancy, envenomation, acute lung injuries, and transfusion reactions. Clinical features vary with the underlying precipitating illness, but classically, oozing from catheter or venipuncture sites is described, with petechiae and ecchymoses also present. Thrombocytopenia, prolonged PT and elevated international normalized ratio (INR), low fibrinogen, elevated fibrin-related markers, and anemia from intravascular hemolysis are classic laboratory markers. Management focuses on treating the underlying cause.
A 51-year-old woman with a history of hypertension, rheumatoid arthritis, migraine headaches, and hypertriglyceridemia presents to the ED for acute, nonradiating chest pain associated with nausea and diaphoresis. Which of the following would contribute to her HEART (history, ECG, age, risk factors, and troponin) score? Cousin with a history of myocardial infarction First-degree relative with peripheral arterial disease History of coronary revascularization procedure Smoker with cessation 6 months ago
History of coronary revascularization procedure The HEART (history, ECG, age, risk factors, and troponin) score was designed as a decision aid for ED clinicians to identify patients presenting with acute, undifferentiated chest pain who are at low risk for major adverse cardiovascular events (MACEs), defined as acute myocardial infarction, need for percutaneous coronary intervention or coronary artery bypass graft, or death within 6 weeks. Patients identified as low risk (HEART score of 0 to 3) are deemed suitable for discharge from the ED without further testing or workup for ACS.
A 22-year-old woman presents to the emergency department due to abdominal pain. On initial evaluation, her abdominal exam is benign, vital signs are within normal limits, and blood work and urinalysis are unremarkable. A rapid response for another patient is called, and you have to exit the room. On re-evaluation, the patient has put lipstick on and is deeply offended that you left her, stating, "I thought I meant something to you," and starts crying loudly. You apologize for abruptly needing to leave, and the patient smiles and asks when you finish your shift. Which of the following personality disorders is the most likely diagnosis? Antisocial Borderline Histrionic Narcissistic
Histrionic Histrionic Personality Disorder Seeking attention, want to be center of attention Dramatic Flamboyant Seductive behavior Sexually inappropriate Treatment: psychotherapy PRAISE ME Provocative behavior Relationships (considered more intimate than they are) Attention (likes to be the center of) Influenced easily Style of Speech (impressionistic) Emotions (rapidly shifting) Made up Emotions exaggerated
An ill-appearing 35-year-old woman with a history of Addison disease presents to the emergency department with 2 days of nausea and vomiting and is unable to tolerate anything by mouth. Her blood pressure is 85/44 mm Hg. Her laboratory workup reveals a glucose of 50 mg/dL, a sodium of 125 mEq/L, and a potassium of 6.0 mEq/L. A bolus of intravenous normal saline with 5% dextrose is initiated. What other agent should be administered concurrently? Dexamethasone Fludrocortisone Hydrocortisone Norepinephrine
Hydrocortisone In primary adrenal insufficiency (Addison disease), cortisol and aldosterone production are impaired. In times of increased metabolic need, such as illness, trauma, or surgery, there is a net deficiency of cortisol (and aldosterone in primary adrenal insufficiency), and adrenal crisis ensues. Hypotension and hypovolemia predominate, while hypoglycemia, abdominal pain, altered mental status, hyperkalemia, and hyponatremia are seen in varying degrees. IV fluid resuscitation with a dextrose-containing solution and corticosteroid supplementation form the basis of treatment. Hydrocortisone (at least a 100 mg IV bolus) is preferred as it exhibits both glucocorticoid and mineralocorticoid effects.
A 26-year-old man presents to urgent care with flank pain. He is extremely uncomfortable, and reports he was fine until a few hours ago when he suddenly began to feel terrible pain in his left lower back. The pain is now accompanied by nausea. He does not report any recent travel or cold-like symptoms. His vital signs are within normal limits, and physical exam shows left costovertebral angle tenderness. You order a computed tomography scan, which confirms the presence of a kidney stone. Which of the following findings necessitates referral to urology? Diameter of stone less than 5 mm Hydronephrosis Obesity Single stone
Hydronephrosis Nephrolithiasis Sx: flank pain radiating to groin PE: patient won't lie still, hematuria Diagnosis: noncontrast helical CT, most common location is the ureterovesical junction (UVJ) Most commonly caused by calcium oxalate Struvite: staghorn calculi, urease-producing bacteria Uric acid: radiolucent on X-ray, gout Cystine: children with metabolic diseases Treatment: < 5 mm: likely to pass spontaneously > 5 mm: medical expulsive therapy (tamsulosin), urology consultation in certain cases > 10 mm: urology consultation, shock wave lithotripsy, ureteroscopy
What is the most common presentation of Hodgkin lymphoma in a 17-year-old boy? Fatigue with splenomegaly Painful axillary lymphadenopathy Painless cervical lymphadenopathy Painless inguinal lymphadenopathy
Painless cervical lymphadenopathy Hodgkin Lymphoma Patient presents with lymphadenopathy (often cervical and painless), pruritus, fever, night sweats, unintentional weight loss, and frequent infections Diagnosis is made by lymph node biopsy showing Reed-Sternberg cells
A 68-year-old man with a history of hypertension and diabetes presents to the emergency department with chest pain. He has a history of tobacco and cocaine use and last used the drug two days ago. He smokes one pack of cigarettes daily. Which one of the historical factors puts him most at risk for having an aortic dissection as a cause for his chest pain? Cocaine abuse Diabetes Hypertension Smoking history
Hypertension Aortic dissection occurs after a tear of the intima allows blood to enter the aortic media, dissecting between the intimal and adventitial layers. The second and most common group of patients who develop aortic dissection include those over age 50 with a history of chronic hypertension. Chronic hypertension is the most common and most important predisposing risk factor found in patients with aortic dissection. Risk factors include chronic hypertension, Advanced age, connective tissue disease, giant cell arteritis, annuloaortic ectasia, family history, cocaine abuse, Iatrogenic (catheterization), and pregnancy
A 36-year-old woman presents to the ED with sudden onset lower abdominal pain and vaginal bleeding. She is 35-weeks pregnant. She reports constant lower abdominal pain and vaginal bleeding for the last 4 hours. On examination, her cervix is closed and she has dark blood coming from the cervical os. Her uterus is firm to the touch. What is the most common risk factor associated with this condition? Advanced maternal age Cocaine use Hypertension Tobacco use
Hypertension The most common risk factor for the development of placental abruption is hypertension. In this scenario, the patient is most likely experiencing a placental abruption, which is the premature separation of a normally implanted placenta. Patients will generally complain of sudden onset, constant abdominal pain and vaginal bleeding that is generally darker in color. They may have uterine firmness on examination. Depending on the degree of abruption, the patient may be hemodynamically unstable. Placental abruption is a life-threatening condition for both the mother and fetus. Large bore IV access should be obtained, IV fluids should be given, and the patient should be typed and crossed for blood. The patient and fetus should have continuous cardiac monitoring. The patient's OB should be emergently consulted. Ultrasound may be helpful in making the diagnosis, however, normal ultrasound does not exclude placental abruption.
Which of the following matches the hypertensive emergency with the correct first-line antihypertensive medication? Acute myocardial infarction - enalapril Aortic dissection - hydralazine Eclampsia - sodium nitroprusside Hypertensive encephalopathy - nicardipine
Hypertensive encephalopathy - nicardipine Hypertensive emergency is characterized by moderate to severe hypertension with evidence of end-organ damage. ypertensive encephalopathy is the result of this failure of autoregulation and is manifest by diffuse vasogenic cerebral edema. Patients typically present with a history of hypertension and severe headache, altered mental status and vomiting. Symptoms may progress to seizures and coma. Once diagnosed, treatment should begin immediately as hypertensive encephalopathy is fully reversible with early, aggressive blood pressure reduction. Reductions in mean arterial pressure of 30% may be necessary although symptom improvement is the best gauge of treatment effectiveness. Nicardipine and labetalol are the medications of choice. Nicardipine is a dihydropyridine calcium channel blocker that is given as a continuous infusion at 5-15 mg/hr. Labetalol is a combined beta- and alpha-adrenergic blocker with a rapid onset of action that can be given as intermittent intravenous boluses or as a continuous infusion.
A 35-year-old man presents to the emergency department after an altercation that occurred at a local bar. He did not lose consciousness and only reports having right eye pain. The patient is noted to have the exam finding in the image above. What is the most likely diagnosis? Acute angle-closure glaucoma Globe rupture Hyphema Hypopyon
Hyphema A hyphema is blood or a blood clot that is located in the anterior chamber and may occur spontaneously or as a result of trauma. Traumatic hyphemas usually result from bleeding from a ruptured iris root vessel. A hyphema may layer out posteriorly if the patient is lying flat and may only become apparent when the patient is sitting upright. Complications of hyphema include increased intraocular pressure, rebleeding, corneal staining, optic atrophy, and accommodative impairment. Treatment consists of prevention of rebleeding and intraocular hypertension. One should elevate the head of the bed to 45 degrees to promote settling of suspended red blood cells and prevent occlusion of the trabecular meshwork. Treatment should be directed by consultation with ophthalmology. Patients deemed stable for discharge should have urgent follow up with ophthamology to reassess intraocular pressure and monitor bleeding.
A 22-year-old man with a known seizure disorder presents after having brief, seizure-like activity that spontaneously resolved shortly prior to arrival. On initial evaluation, he is noted to be protecting his airway with stable vital signs but appears somnolent. Blood glucose is unremarkable, and IV access is established. Family notes no recent illnesses or injuries, but they are concerned that he has been unable to afford his prescriptions recently. Shortly thereafter, he begins to exhibit generalized convulsions that do not spontaneously resolve after several minutes. Which of the following medications should be given next? IV diazepam IV levetiracetam IV lorazepam IV phenytoin
IV lorazepam Status epilepticus is frequently encountered in the emergency department. Features characterizing status epilepticus include seizure-like activity for more than 5 minutes or multiple seizures of any length without a return to a patient's baseline mental status. The best initial choice of medication for status epilepticus is a benzodiazepine, and lorazepam is the best benzodiazepine if IV access is available. IV lorazepam has been demonstrated to be most efficacious compared to a variety of medications. This is likely a result of lorazepam's short (3 minutes) time to peak effect and significantly longer effective duration of action as compared with diazepam (4-24 hours versus 15-60 minutes).
A 17-year-old girl with a history of developmental delay and generalized anxiety disorder presents with crying, intermittent yelling, and retching. Her vital signs are significant for a heart rate of 104 bpm. She is inconsolable and unable to provide significant history due to agitation. She reports no pain but tells you her chest feels tight, and she cannot breathe. Her lungs are clear to auscultation. What is the most effective management? IV lorazepam PO fluoxetine PO gabapentin PO olanzapine
IV lorazepam This patient is presenting with a panic attack, a spontaneous and discrete episode of intense fear or anxiety that begins abruptly. Patients may develop excessive anxiety and maladaptive behavioral changes to avoid future episodes. Emergency management of acute agitation secondary to panic attack is best treated with a benzodiazepine, such as lorazepam. These medications have fast-acting anxiolytic properties that should help improve agitation and facilitate evaluation. Rapid anxiolysis in panic disorder may help improve somatic symptoms, such as chest tightness. Benzodiazepines are not first line for chronic treatment of panic disorder and anxiety due to the risk of abuse and dependence.
A 26-year-old man presents with low back pain after helping his friend move yesterday. He denies any trauma, fever, weakness, numbness, or change in bowel or urinary habits. He has no midline tenderness but has spasm and tenderness noted in the paraspinal muscles. Which of the following medications is most appropriate for this patient's pain? Cyclobenzaprine Ibuprofen Oxycodone Tramadol
Ibuprofen Muscle strains of the back are common after exertion, stretching, and heavy lifting. They classically present with tightness, stiffness, spasms, and pain that is worsened with stretching, twisting, or bending. Exam may demonstrate paraspinal muscle tenderness. Otherwise, the patient's neurological exam should be normal and they should not have any urinary or bowel symptoms, fevers, or chills. Patients who have symptoms of a muscle strain without any other concerning historical or physical exam findings can be treated symptomatically with NSAIDs, such as ibuprofen, acetaminophen,and warm or cool compresses. Patients should be encouraged to engage in activity as tolerated and may benefit from physical therapy if their pain does improve over the next several days.
A healthy 25-year-old man presents to the emergency department complaining of chest pain for two days. He states the pain is moderate in intensity but worse with inspiration and lying down. He also notes he had several days of fever and general malaise last week. Vital signs are significant for temperature of 100°F and his ECG is shown above. Which of the following is the most appropriate treatment? Ibuprofen orally three times daily Levofloxacin orally daily Nitroglycerin sublingual as needed Prednisone orally daily
Ibuprofen orally three times daily This patient's clinical presentation is consistent with pericarditis. Pericarditis is inflammation of the fibrous lining of the heart and can cause chest pain that is positional in nature, a friction rub on auscultation, and diffuse ST elevation as seen above. In the United States and other developed nations, the most likely etiology is viral infection. Other causes can include vasculitis, systemic rheumatic diseases, tuberculosis, and metastatic disease. The initial treatment is ibuprofen or other NSAIDs, with admission indicated for those with fever greater than 101°F, signs of pericardial effusion or tamponade, or those taking anticoagulants.
A 68-year-old woman presents to the emergency department complaining of right upper arm pain after a fall. She was walking down an icy sidewalk when she slipped and landed on her right arm. There is a palpable deformity in her proximal right humerus. She is neurovascularly intact. X-rays confirm nondisplaced right proximal humerus fracture without dislocation. What is the most appropriate clinical intervention? Closed reduction Immobilization in a sling Physical therapy referral Surgical repair
Immobilization in a sling immobilization in a sling is the most appropriate initial treatment for a nondisplaced humerus fracture. The general recommendation is for immobilization after minimally displaced fractures to last one week, with increased range of motion exercises thereafter. Ice can reduce pain and swelling, and pain medications will likely be necessary in the first week following injury. Follow-up physical examination and imaging are performed one week after injury, with subsequent follow-up visits every two weeks after that. The sling may be discontinued two to four weeks after the initial injury. Proximal humerus fractures typically require six to twelve weeks to heal.
A 16-year-old boy presents with left wrist pain after a fall while skateboarding in which he landed on his outstretched hand. He has tenderness and swelling over the dorsum of the wrist on the radial side in between the abductor pollicis longus and extensor pollicis longus tendons. Wrist radiographs are negative for a fracture. Which of the following is the most appropriate management? Discharge home with anti-inflammatories Emergent orthopedic consult Immobilization in a thumb spica splint Perform an incision and drainage
Immobilization in a thumb spica splint Scaphoid Fracture History of fall on an outstretched hand (FOOSH) Dorsal radial wrist pain with decreased range of motion PE will show anatomical snuffbox tenderness Confirm with imaging (X-ray, CT, bone scan, MRI most accurate) Treatment is thumb spica splint
A 9-year-old boy with no medical history presents to the emergency department with facial swelling. He states that his symptoms started 1 day ago. He also notes that his urine has been foamier than it usually is. His parent reports that he recently recovered from a sore throat and fever 2 weeks ago, which was treated with antibiotics. Urinalysis shows red blood cells and proteinuria. What is the pathophysiology of this disease process? IgA deposition Immune complex deposition Myoglobin-mediated toxicity Reduced glomerular filtration rate
Immune complex deposition Poststreptococcal Glomerulonephritis PE: hypertension, hematuria, and periorbital edema Labs: proteinuria and red blood cell casts in the urine, low C3 and CH50 Most commonly caused by group A beta-hemolytic Streptococcus Management includes mainly supportive measures, e.g., salt and water restriction - If edema and hypertension present, concurrent furosemide can help Most common infectious cause of acute glomerulonephritis
A 30-year-old healthy woman presents to the emergency department for a painful mass. Examination is significant for normal vital signs and a 3 cm inflamed abscess in the midline superior natal cleft with no surrounding erythema. The patient states she has had nontender swelling in the same location previously, but now it is very painful for her to lay down. Which of the following is the most appropriate management of this patient's condition? Emergent surgical consultation Incision and drainage and discharge with cephalexin orally Incision and drainage and outpatient surgery follow-up Outpatient surgery follow-up
Incision and drainage and outpatient surgery follow-up Pilonidal Abscess I&D Consider antibiotics (If surrounding cellulitis) Refer to surgery
A 27-year-old gravida 1, para 0 woman currently at 14 weeks gestational age, confirmed via ultrasound the week prior, presents to the emergency deparatment reporting lower abdominal cramping over the past 2 days. She notes vaginal bleeding that she describes as thick and "clot-like." On exam, she has an open cervical os with evidence of fetal tissue. Which of the following is the most likely diagnosis? Complete abortion Incomplete abortion Inevitable abortion Missed abortion
Incomplete abortion A spontaneous abortion is defined as a loss of pregnancy before 20 weeks gestational age, or loss of a fetus that weighs less than 500 grams. Patients most commonly present with abdominal pain, vaginal bleeding, or both. The most common cause of spontaneous abortion is chromosomal abnormalities. Other risk factors include prior spontaneous abortions, infection, advanced maternal age, comorbid conditions, anatomic abnormalities, and tobacco use. There are six different types of spontaneous abortions: threatened, inevitable, incomplete, complete, missed, and septic abortions. An incomplete abortion is a type of spontaneous abortion diagnosed by the presence of an open cervical os with passage of fetal tissue, with products of conception still present in the uterus. Incomplete abortions most commonly occur between 6 and 14 weeks gestational age. Treatment includes evacuation of products of conception remaining in the uterus, either via medical treatment (misoprostol) or surgical treatment (dilation and curettage).
A 66-year-old woman presents with new-onset lower extremity edema and shortness of breath. She states this has progressively worsened over the past 2 months, during which time she has also had a 10-pound weight gain. She notes that earlier in the week, she had substernal chest pain that lasted for 10 hours before resolving on its own. She has an unknown medical history as she does not typically see a primary care clinician. Vital signs include a heart rate of 106 bpm, a respiratory rate of 22 breaths/minute, and an oxygen saturation of 92% on room air. Physical examination is remarkable for the patient sitting upright with rales noted in the lung bases, tachycardia, and 3+ pitting edema noted to the bilateral mid-thighs. What is the most likely pathophysiology for this patient's presentation? Decreased protein synthesis Decreased urinary output Increased capillary pressure Increased urinary protein excretion
Increased capillary pressure This patient presents with a concerning story for heart failure related to a recent ischemic event. Patients with heart failure can present with dyspnea, chest pain, jugular venous distention, orthopnea, paroxysmal nocturnal dyspnea, and edema. All of these symptoms are related to the heart's inability to either fill appropriately (diastolic dysfunction), pump effectively (systolic dysfunction), or a combination of both. This results in increased capillary pressures that cause pulmonary edema, venous distention, hepatomegaly, or peripheral edema. Treatment of edema is focused on determining the etiology and correcting the disorder if possible. In cases of heart failure, furosemide and other diuretics are the mainstay of therapy along with the prevention of further progression of the disease with beta-blockers and angiotensin-converting enzyme inhibitors
A 60-year-old man presents to urgent care with acute onset of right-sided weakness that began three hours ago while he was preparing breakfast. He decided to seek medical attention after experiencing double vision. He has no significant past medical history. His blood pressure is 170/89 mm Hg, heart rate is 90 beats/minute, respiratory rate is 14 breaths/minute, and O₂ saturation on room air is 97%. A noncontrast head CT is negative for intracranial hemorrhage and has findings of an acute infarction. Which of the following is the best first step in the management of this patient? Clopidogrel plus aspirin Endovascular thrombectomy Intravenous alteplase Intravenous nitroprusside
Intravenous alteplase Patients must be at least 18 years of age, and onset of symptoms must be within 4.5 hours of beginning treatment. If the exact time of stroke onset is not known, it is defined as the last time the patient was known to be normal. Ischemic Stroke Anterior cerebral artery: frontal lobe dysfunction, apraxia, contralateral paralysis (lower > upper) Middle cerebral artery: contralateral paralysis (upper > lower), aphasia Posterior cerebral artery and VBI: LOC, nausea or vomiting, CN dysfunction, ataxia, visual agnosia Rule out hypoglycemia CT reveals loss of gray-white interface, acute hypodensity Thrombolytics
A 45-year-old woman presents to the emergency department with eye pain and blurry vision after a cleaning solution accidentally splashed into her eyes. She was using a rust remover agent that contains hydrofluoric acid. On exam, she has difficulty opening her left eye due to intense pain. What is the most important management step to perform next to treat this patient? Instill tetracaine anesthetic drops and begin copious saline irrigation of both eyes Instill tetracaine anesthetic drops and begin topical erythromycin therapy Provide intravenous opiate pain medicine and instill calcium gluconate gel into the eye Test the pH of the eye and consult ophthalmology for emergent evaluation
Instill tetracaine anesthetic drops and begin copious saline irrigation of both eyes For any chemical injury, it is important to remove the toxic agent or remove the patient from the toxic environment. Alkaline substances usually cause severe damage and liquefactive necrosis. Acid injuries can cause significant pain, but the injuries are usually less severe and they are self-limited. The initial management is to instill tetracaine or proparacaine anesthetic drops and begin copious saline or water irrigation to both eyes. It is important to ensure that the patient has not suffered a penetrating eye injury before beginning irrigation. A good rule of thumb is to assume that a patient needs approximately 30 minutes of irrigation, but acid injuries generally do not require irrigation longer than 15 minutes.
A 76-year-old man presents to the emergency department with acute onset of left sided hemiparesis. Symptoms started abruptly 45 minutes prior to arrival. He has a history hypertension and type II diabetes mellitus. Stat imaging suggests an acute ischemic stroke. Which of the following is considered an absolute contraindication for thrombolytic therapy? Age greater than 75 years Blood glucose of 325 mg/dL Gastrointestinal or genitourinary bleeding within the past three months Ischemic stroke within the past three months
Ischemic stroke within the past three months There are strict protocols regarding the use of t-PA in patients with ischemic stroke due to the risk of hemorrhage. Eligibility for t-PA includes an age greater than 18 years, clinical diagnosis of ischemic stroke causing neurologic deficit and time of onset less than 4.5 hours. Absolute contraindications include intracranial hemorrhage on CT, clinical presentation that suggests subarachnoid hemorrhage, neurosurgery, head trauma or stroke within the past three months, uncontrolled hypertension (greater than 185 mmHg systolic or greater than 110 mmHg diastolic), history of intracranial hemorrhage, known arteriovenous malformation, neoplasm, or aneurysm, active internal bleeding, suspected or confirmed endocarditis, known bleeding diathesis, recent arterial puncture at a noncompressible site, or an abnormal glucose (less than 50 mg/dL).
A 61-year-old woman with a history of hypertension presents to the ED with fatigue and dyspnea on exertion, which has progressed over the past week. Her shortness of breath is worse with lying down. She has been sleeping with several pillows to prop up her head. She also reports bilateral leg swelling, which is new. On lung auscultation, rales are heard at the bases. Which of the following findings are most likely to be seen on her chest radiograph? A right middle lobe infiltrate Hyperinflated lungs Kerley B lines Westermark sign
Kerley B lines Heart Failure Most common cause: ischemic heart disease Systolic dysfunction (HFrEF) ↓ EF Cardiac output (CO) dependent on afterload Diastolic dysfunction (HFpEF) Impaired myocardial relaxation → abnormal LV . filling CO dependent on preload Left heart failure ↑ LVEDV + ↑ LVEDP → blood backup into lungs . → pulmonary edema Dyspnea, orthopnea, paroxysmal nocturnal dyspnea Crackles Right heart failure Most common cause: left heart failure JVD, peripheral edema, ascites S3 BNP: distinguishes between CHF and dyspnea of pulmonary etiology Most useful study: echocardiogram
A 20-year-old man presents to the ED complaining of a headache after a skiing injury. His friend witnessed the fall and reports that the patient was "out for a few minutes" but confirms that he appears back to his normal self now, other than amnesia about the circumstances of the fall and two episodes of vomiting. Physical exam reveals only crepitus and swelling over the left parietal skull. What do you most likely expect to find on the patient's computed tomography of the head? Crescent-shaped hyperdensity crossing suture lines Crescent-shaped hypodensity not crossing suture lines Lens-shaped hyperdensity crossing suture lines Lens-shaped hyperdensity not crossing suture lines
Lens-shaped hyperdensity not crossing suture lines Epidural Hematoma History of a head injury with a loss of consciousness followed by a lucid interval CT will show a biconvex opacity Most common artery ruptured is the middle meningeal artery Treatment is emergent evacuation
A 56-year-old man with a history of chronic alcohol use disorder presents with confusion. His wife notes that he has been progressively more confused in the last few days and sleeping much more than usual. There has been no fever, headache, cough, vomiting, or abdominal pain but she noticed that his stool was almost black when she helped him in the bathroom earlier today. On examination, he is lethargic and confused but able to answer a few simple questions. There is mild scleral icterus. The abdomen is soft without tenderness, guarding, or rebound. On rectal exam, there is dark, tarry stool in the rectal vault that is hemoccult positive. There are no focal neurologic deficits, although when asked to hold his arms up to test his strength, asterixis is noted. In addition to management of his gastrointestinal hemorrhage, which of the following is the best choice for treatment of his altered mental status? Ceftriaxone, vancomycin, and ampicillin Lactulose and rifaximin Metronidazole Thiamine
Lactulose and rifaximin Hepatic Encephalopathy History of infection, dehydration, medications, GI bleed, hypoxia, hypoglycemia Presents with confusion PE will show scleral icterus and asterixis Most commonly caused by toxin accumulation due to liver dysfunction Treatment is lactulose, rifaximin CSF glutamine correlates with severity
A 38-year-old woman presents to the urgent care clinic after she inverted her right ankle while stepping off a curb. She can walk on the affected foot. Physical exam is remarkable for tenderness to palpation of the right lateral malleolus. Pain worsens with ankle inversion. There are no bony deformities. What is the most likely diagnosis? Lateral ankle sprain Navicular stress fracture Nondisplaced fracture of the fifth metatarsal shaft Osteochondral lesion
Lateral ankle sprain Lateral ankle sprain is the most common ankle injury and occurs when the plantar-flexed foot is inverted, damaging the lateral ligament complex of the ankle. There are three grades of ankle sprains. A grade 1 sprain occurs when the ligament is stretched with associated microscopic tears. There is mild swelling and tenderness, and no joint instability. Patients can bear weight and ambulate with minimal pain. A grade 2 sprain involves an incomplete tear of a ligament. Patients report moderate pain, swelling, and ecchymosis. Exam reveals moderate joint instability with some restricted range of motion and loss of function. Ambulation and weight-bearing cause pain. A ligament is completely torn in a grade 3 sprain. he following are special tests to evaluate ankle injury: squeeze test, which elicits pain near the anterior tibiofibular ligament when a syndesmotic sprain has occurred; anterior drawer test, which detects anterior displacement of the talus on the tibia; external rotation test, which is positive if there is pain anterior to the lateral malleolus and proximal to the joint when the foot is externally rotated relative to the tibia; and talar tilt test, in which the degree of gentle inversion of the injured ankle is observed. The injured ankle should be compared to the unaffected side when performing each of these tests.
Which of the following is seen in the ECG above? Left anterior hemiblock Left bundle branch block Nonspecific ventricular conduction delay Right bundle branch block
Left bundle branch block A bundle branch block is caused by the blockage of conduction through the right or left bundle branches. This causes a delay in the depolarization of the affected ventricle. Instead of depolarizing simultaneously, one bundle branch fires after the other. On the ECG, this causes a wide QRS. When a bundle branch block is present, the QRS will be at least 0.12 seconds wide (3 small boxes on the ECG). When the left bundle branch is blocked, this will be seen best in the left-sided leads V5 and V6. The sequential depolarization of the right and then the left ventricle will be seen as two distinct R waves: R and R′. The criteria for left bundle branch block are (1) QRS duration ≥ 0.13 s in women or ≥ 0.14 s in men, (2) lead V1 QS or rS, (3) mid-QRS notching or slurring in two of the leads I, aVL, V1, V2, V5, or V6.
Which of the following deficits is the lesion above most likely to produce? Isolated left lower extremity weakness and numbness Isolated right facial droop with slurred speech Left hemiparesis and sensory loss with left facial droop Right hemiparesis without sensory loss
Left hemiparesis and sensory loss with left facial droop The middle cerebral artery (MCA) is the most commonly affected vessel in ischemic stroke. It is the largest branch of the internal carotid artery and supplies the majority of the cerebral cortex, including a portion of the frontal lobe and the lateral portions of the parietal and temporal lobes. Symptoms vary with the location of the lesion and whether the dominant lobe is affected. Typical symptoms include contralateral hemiparesis, facial weakness, and sensory loss. These symptoms will usually affect the upper extremity and face more than the lower extremity, which distinguishes MCA strokes from those caused by the anterior cerebral artery. Involvement of the dominant lobe results in aphasia, which may be expressive, receptive, or both. Management of MCA strokes is dependent on the timing of presentation in relation to symptom onset, with the administration of tPA significantly improving outcome in eligible patients.
A 65-year-old man with a history of hypertension presents with gradual right testicular pain over the past 3 days. He is monogamous with his wife of 30 years. His vital signs are within normal limits. On exam, he has tenderness and firmness at the posterolateral aspect of the right testicle. His pain is somewhat relieved with elevation of the right testicle. A urinalysis shows pyuria and positive leukocyte esterase. Which of the following is the most appropriate treatment for the suspected diagnosis? Ceftriaxone and doxycycline Cephalexin Doxycycline Levofloxacin
Levofloxacin This patient's presentation is consistent with epididymitis, most often caused by bacterial infection. Patients present with gradual-onset testicular, lower abdominal, inguinal, or scrotal pain. On exam, patients may have tenderness to palpation of the epididymis, located at the posterolateral pole of the testicle. Relief of pain with elevation of the affected testicle is known as Prehn sign. Urinalysis is useful in diagnosis and reveals pyuria in about 50% of patients with epididymitis or epididymo-orchitis. Ultrasound may be used as a diagnostic adjunct, revealing hyperemia, or increased blood flow, to the epididymis and testicle (if epididymo-orchitis). Patients who are not sexually active or do not have high-risk sexual behavior, such as this patient who has been monogamous with his wife for many years, should be treated with a fluoroquinolone such as levofloxacin or ofloxacin to cover for Escherichia coli and Klebsiella species
A 42-year-old man presents with pain in his foot after falling while descending the stairs. He is unable to bear weight on the foot. Pulses and sensation are intact. He has pain with palpation of the midfoot and a small area of ecchymosis on the plantar surface of the foot. His X-ray is shown above. What is the likely diagnosis? Jones fracture Lisfranc injury Navicular fracture Second metatarsal fracture
Lisfranc injury The Lisfranc joint (tarsometatarsal joint complex) is comprised of the articulations of the bases of the first three metatarsals with the cuneiforms and the fourth and fifth metatarsals with the cuboid, joined together with multiple ligaments. Lisfranc Injuries History of a motor vehicle collision, fall, or playing sports Severe foot pain and inability to bear weight PE will show tenderness of the tarsometatarsal joint Post-traumatic arthritis is the most common complication
Which of the following drugs can cause hypothyroidism in either therapeutic usage or overdose? 3,4-methylenedioxymethamphetamine Gamma-hydroxybutyrate Linezolid Lithium Valproic acid
Lithium Lithium can cause a variety of endocrine abnormalities, either in therapeutic usage or overdose. Subclinical or overt hypothyroidism is relatively common.
A 57-year-old man with a history of depression presents to the emergency department with a tremor, nausea, and vomiting. He states that he drinks 5 to 10 alcoholic beverages per day and smokes cigarettes. His blood pressure is 157/108 mm Hg, pulse is 115/min, respiratory rate is 26/min, and oxygen saturation is 99% on room air. Physical exam is notable for a tremor both at rest and with action and sweaty skin. Which of the following is the best treatment for this patient? Lactulose and rifaximin Levodopa and carbidopa Lorazepam Propranolol
Lorazepam This patient is presenting with signs and symptoms suggestive of alcohol withdrawal, which should promptly be managed with phenobarbital or a benzodiazepine. Alcohol acts on the GABA receptors of the central nervous system and activates them. Chronic alcohol use leads to the central nervous system adapting by upregulating NMDA channels (stimulatory) and downregulating GABA channels (inhibitory). When the patient suddenly stops consuming alcohol, their central nervous system has an increased NMDA to GABA ratio, and the patient can have withdrawal seizures. Other symptoms of alcohol withdrawal include hypertension, tachycardia, tremulousness, irritability, and hallucinations.
A 47-year-old man presents to the emergency department with a nonhealing ulcer on his heel. Past medical history is significant for uncontrolled diabetes. The patient states that the ulcer has been present for the past couple weeks but is now worsening in appearance. His vital signs are unremarkable. Physical examination demonstrates a left heel ulcer with purulent discharge and surrounding erythema. You are concerned for osteomyelitis of the calcaneus. Which of the following is the most sensitive test for suspected osteomyelitis? CT scan Elevated erythrocyte sedimentation rate Magnetic resonance imaging X-ray
Magnetic resonance imaging Osteomyelitis can occur as a result of hematogenous seeding, contiguous spread from adjacent structures or tissues, or direct inoculation secondary to trauma or surgery. Contiguous osteomyelitis is more common among all other age groups due to trauma, surgery, decubitus ulcers, or infected total joint arthropathies. Contiguous osteomyelitis can be either polymicrobial or monomicrobial. Staphylococcus aureus, coagulase-negative staphylococci, and aerobic gram-negative bacilli are the organisms most commonly responsible for osteomyelitis. Clinical features of acute osteomyelitis include the gradual onset of symptoms over several days. Pain with or without movement at the site of infection may be present with associated local tenderness, warmth, erythema, and swelling. Systemic symptoms of fevers and rigors may also be present. Definitive diagnosis is established with a culture of bacteria from a bone biopsy along with histological findings. However, this may not be required if radiographic studies along with positive blood cultures are present. Conventional X-ray imaging is a reasonable initial image modality in patients with suspected osteomyelitis. Findings, however, may not be present until two weeks of symptoms have been present. Magnetic resonance imaging (MRI) is the imaging modality with the greatest sensitivity for the diagnosis of osteomyelitis and has a high negative predictive value. Treatment of osteomyelitis consists of debridement and antibiotics with a prolonged duration of therapy.
A 40-year-old man presents after an episode of bloody emesis. He reports that this occurred after binge drinking and vomiting numerous times. He says the vomiting has currently stopped and he is feeling better. Physical exam is unremarkable, including vitals signs that are within normal limits. Which of the following is the most likely diagnosis? Boerhaave syndrome Diverticulosis Mallory-Weiss syndrome Peptic ulcer disease
Mallory-Weiss syndrome Mallory-Weiss Syndrome History of drinking alcohol and forceful vomiting Hematemesis Diagnosis is made by upper endoscopy Caused by an incomplete tear in the esophagus mucosa and proximal stomach
A healthy 20-year-old woman presents to the emergency department for evaluation of a hand laceration caused by her pet cat's teeth two hours prior to arrival. Examination reveals a 2 cm laceration to the dorsum of her hand, involving the skin and subcutaneous tissues without significant bleeding or surrounding erythema; her hand is neurovascularly intact. After assessment for tetanus and rabies immunization and copious irrigation of the wound, which of the following would be appropriate management of the laceration? Intravenous ampicillin-sulbactam and emergent hand surgery consult No wound closure and oral amoxicillin-clavulanic acid No wound closure and topical mupirocin ointment Primary closure and follow-up with a hand surgeon in 48 hours
No wound closure and oral amoxicillin-clavulanic acid Cat Bite Most commonly caused by Pasteurella multocida Treatment is irrigate, leave the wound open, amoxicillin-clavulanate Complications: osteomyelitis, tenosynovitis
A 25-year-old woman presents to the emergency department with a painful unilateral loss of vision. She states it started this evening and has persisted. The patient has no known medical history and is not currently taking any medications. Her vital signs and an initial ECG are within normal limits. When the patient's neck is flexed, an electrical pain is felt down her neck and back. An afferent pupillary defect is present in the affected eye. Which of the following is the best initial treatment for this patient? Ceftriaxone, vancomycin, and dexamethasone Methylprednisolone Plasmapheresis Riluzole
Methylprednisolone Multiple sclerosis is an autoimmune demyelinating disorder that is more common in Caucasian women. Demyelination can cause seemingly unrelated symptoms including paresthesias, weakness, urinary or bowel incontinence, and loss of pain or temperature sensation. Ophthalmoplegia can be seen in this condition, which presents with an inability to adduct the affected eye. Other unique symptoms include optic neuritis, which can present with a painful unilateral loss of vision and Lhermitte sign that presents with electrical pain that radiates down the neck and spine when the head is flexed. The diagnosis of multiple sclerosis can be supported with MRI (preferred) or lumbar puncture demonstrating oligoclonal bands, and the best initial treatment in an acute exacerbation is a steroid such as high-dose methylprednisolone.
A 24-year-old woman presents to the emergency department with tremors, anxiety, and palpitations for 3 days. She has been evaluated for these symptoms before, but no diagnosis was made aside from a presumptive psychiatric disorder. However, her symptoms are more severe now. Vital signs are HR of 126 bpm, BP of 157/89 mm Hg, RR of 20 bpm, and T of 37.9°C. She has exophthalmos on the exam, so you check a thyroid-stimulating hormone level and find it to be < 0.05 mU/L. What is the most appropriate treatment to initiate at this time? Hydrocortisone Iodine Methimazole Metoprolol
Metoprolol The most common cause of hyperthyroidism is the autoimmune disorder Graves disease. patients may report anxiety, emotional lability, tremors, palpitations, heat intolerance, and weight loss. Physical exam findings can include hyperactivity, rapid speech, lid retraction, and proximal muscle weakness. Once patients develop exophthalmos and have tachycardia, the diagnosis becomes more obvious, and a patient could be on the precipice of thyroid storm. Patients with suspected hyperthyroidism should have a thyroid-stimulating hormone test performed. Patients with the illness will have extremely low values along with elevations of free serum thyroxine (T4) and triiodothyronine (T3). Treatment of Graves hyperthyroidism consists of reducing symptoms, preventing complications of the illness (thyroid storm), and decreasing thyroid hormone synthesis. Treatment with a beta-blocker relieves palpitations, tachycardia, tremulousness, anxiety, and heat intolerance associated with hyperthyroidism. Additionally, beta-blockers prevent cardiac dysrhythmias and progressive cardiac disease. Once a beta-blocker has been initiated, other treatments can be started, with the most common choice of therapy a thionamide, such as methimazole.
An 88-year-old woman with a history of hypertension presents to the emergency department with palpitations and shortness of breath for one week. She has intermittent mild chest pain but denies any of those symptoms while sitting in the emergency department. She takes metoprolol for her hypertension and is compliant with her daily dose. Vital signs show HR 146 bpm, BP 126/68 mm Hg, RR 12/min, and T 37.4°C. She has an unremarkable physical examination aside from tachycardia. An electrocardiogram is obtained and is shown above. What is the most appropriate treatment at this time? Adenosine intravenously Amiodarone intravenously Intravenous fluid administration Metoprolol intravenously
Metoprolol intravenously To answer this question correctly, the clinician must recognize that this electrocardiogram demonstrates a narrow complex irregular tachycardia that is consistent with atrial fibrillation with rapid ventricular response. However, multiple studies have not shown a mortality benefit to rhythm control versus rate control, thus, a rate control treatment strategy is preferred. The patient in this question has mild symptoms, but none currently, and she is not hypotensive. She is an appropriate candidate for pharmacologic treatment. There are several treatment options to control the ventricular rate in atrial fibrillation, but beta-blockers and calcium channel blockers remain superior to other options like digoxin. Calcium channel blockers have been found to be slightly more effective in treating acute atrial fibrillation with rapid ventricular response, but both calcium channel blockers and beta-blockers are considered effective in the acute setting. Since this patient is already taking metoprolol for other reasons, it might be wise for the clinician to use intravenous metoprolol at this time.
A 19-year-old woman presents with a 5-day history of vaginal discharge. She reports no fever, chills, or abdominal pain. She has had two sexual partners in the last month. Urine pregnancy test is negative. Exam demonstrates normal vital signs and a nontender, nondistended abdomen. Vaginal exam is significant for a copious, frothy, yellow-grey discharge. Vaginal pH is 5.2. The cervix appears inflamed. Mobile flagellates are seen on microscopy. Which of the following is the best first-line treatment option for this condition? Ceftriaxone 500 mg intramuscularly once Doxycycline 100 mg orally twice daily for 7 days Fluconazole 150 mg orally once Metronidazole 500 mg orally twice daily for 7 days
Metronidazole 500 mg orally twice daily for 7 days Trichomoniasis Sx: malodorous vaginal discharge, pruritus, dysuria in sexually active person PE: frothy, green or yellow discharge, petechiae on cervical mucosa: strawberry cervix Dx: vaginal pH > 4.5Wet mount: flagellated, motile, pear-shaped organismNucleic acid amplification and rapid antigen tests are more sensitive Causal organism: Trichomonas vaginalis Tx: oral metronidazole
A 71-year-old woman presents with exertional dyspnea. A murmur is heard on cardiac auscultation. She reports a history of acute rheumatic fever as a child. Which of the following murmurs suggests a diagnosis of mitral stenosis? Continuous, machine-like murmur Harsh systolic murmur radiating to the carotids High-pitched apical holosystolic murmur Mid-diastolic rumbling murmur
Mid-diastolic rumbling murmur In mitral stenosis, damage to the mitral valve apparatus results in impaired diastolic filling of the left ventricle. Rheumatic heart disease, although decreasing in frequency, is still the most common cause of mitral valve stenosis worldwide. In the acute setting, patients may have valvular insufficiency and heart failure. Patients may be asymptomatic for decades, but as mitral valve obstruction progresses, left atrial pressure increases and leads to left atrial enlargement, often with associated atrial fibrillation. Pulmonary congestion and pulmonary hypertension follow. Clinically, patients with mitral stenosis present with dyspnea on exertion, orthopnea, and other sequelae of heart failure. The classic murmur of mitral stenosis is a mid-diastolic rumbling murmur. A loud opening snap after the first heart sound may also be heard. Mitral stenosis is usually managed with anticoagulation once left atrial diameter exceeds 55 mm or atrial fibrillation is present, due to the increased risk of systemic embolization.
A 59-year-old man presents to the emergency department after he fainted while at work. The patient is a construction worker and reports he hydrated poorly today while working in the humid weather. He is given 1 liter of Ringer lactate solution and states that he feels much better. His rhythm strip is shown above. Which of the following is the most likely diagnosis? Complete heart block Mobitz type I Mobitz type II Wolff-Parkinson-White syndrome
Mobitz type I Mobitz type I heart block is typically asymptomatic but can present with weakness, fatigue, and syncope. An ECG will demonstrate a PR interval that gradually prolongs until a QRS complex is dropped. A patient with a new-onset Mobitz type I rhythm should be worked up for an acute ischemic event, including troponins to rule out a new myocardial infarction, and electrolytes to rule out an electrolyte abnormality. A stable patient with Mobitz type I heart block requires no acute interventions and should be managed outpatient by their primary care physician and cardiologist once acute causes are ruled out. Mobitz type I heart block carries a low risk of progressing to complete heart block. This is in contrast to Mobitz type II, an unstable rhythm that carries a high risk of progressing to complete heart block and requires pacemaker placement. Unstable patients with Mobitz type I heart block can initially be treated with atropine and transcutaneous pacing
A 3-year-old boy presents with his mother for a pruritic facial rash that has been present for one week. Mom reports that the rash started as small vesicles that ruptured and formed a crust. Examination is significant for the skin findings shown above. Which of the following is the most appropriate treatment for this patient? Bacitracin Emollients Mupirocin Triamcinolone
Mupirocin Impetigo child < 6 years (can affect any age) Nonpainful, pruritic lesions on the face PE: honey-colored, weeping lesions Staphylococcus aureus, Streptococcus pyogenes Tx: -Limited number of lesions: topical mupirocin -Numerous lesions or involvement of more than one area: oral antibiotics (cephalexin or dicloxacillin) Complications: poststreptococcal glomerulonephritis
A 38-year-old man presents to the emergency department with a new-onset rash. He has a past medical history significant for epilepsy and hypertension, for which he currently takes levetiracetam and lisinopril. The patient was feeling well until two days ago when he started developing fevers, chills, malaise, and body aches. Yesterday, he noticed a rash had started on his face and trunk. He states the rash has since spread to all four extremities. A review of systems is positive for recently completing a course of azithromycin for walking pneumonia. Vital signs are remarkable for a temperature of 40°C, HR 138, BP 90/60, RR 29, and pulse oximetry 98%. Physical examination is remarkable for a toxic-appearing man with blisters and erythematous macules with a purple core covering the trunk, face, and extremities. Mucositis is noted on examination as well as a positive Nikolsky sign. Which of the following is most related to his likely diagnosis of Stevens-Johnson syndrome? Azithromycin Levetiracetam Lisinopril Mycoplasma infection
Mycoplasma infection Stevens-Johnson syndrome (SJS) is a severe mucocutaneous reaction characterized by extensive necrosis and detachment of the epidermis. It is more common in women than men and occurs in any age group. Drugs are the most common cause and trigger of SJS or toxic epidermal necrolysis (TEN). Drugs commonly known to cause SJS (remembered by the mnemonic PEC SLAPP) include penicillin, ethosuximide, carbamazepine, sulfa medications, lamotrigine, allopurinol, phenytoin, and phenobarbital. The next most common cause after medications is Mycoplasma pneumoniae infections. SJS is the more common and less severe of two with less than 10% of TBSA involvement. Patients with greater than 30% of TBSA involvement are diagnosed with TEN. The clinical presentation includes a prodromal phase, cutaneous lesions, and mucosal lesions. The prodromal phase is characterized by a high fever and influenza-like symptoms, which precede the development of mucocutaneous lesions by one to three days. The cutaneous lesions are described as erythematous macules with tender, purpuric centers. These lesions typically start on the face and thorax and spread to the extremities. Nikolsky sign may also be present. Mucosal lesions occur in the majority of the cases of both SJS and TEN. Treatment involves removal of the causative drug, and supportive care, including wound care, fluids and nutrition, pain control, and prevention and treatment of infections.
A 55-year-old man with a history of hyperlipidemia, diabetes mellitus, hypertension, and uric acid crystal arthropathy presents with right great toe pain, redness, and swelling. His examination reveals a right metatarsophalangeal tophus. Which of the following regimens represents appropriate medical therapy for this patient? Aspirin Naproxen and topical ice Prednisone and colchicine Triamcinolone injection
Naproxen and topical ice Crystal-induced arthropathy includes gout and pseudogout. Diagnosis can only be made with arthrocentesis of the affected joint and synovial fluid analysis with polarizing light microscopy. Uric acid crystals are negatively birefringent (i.e., appear blue when light is perpendicular to the crystal), and calcium pyrophosphate crystals are positively birefringent (i.e., appear yellow when light is parallel to the crystals). First-line therapy includes a five to seven day course of nonsteroidal anti-inflammatory drugs (NSAIDs) such as indomethacin or naproxen. Therefore, naproxen and topical ice is the most appropriate therapy for this patient. Additionally, prednisone is considered a first-line therapy but must be used with caution in diabetics and patients with hypertension and congestive heart failure. Allopurinol and probenecid are not useful for acute gout flares. Rather, they are used for long-term therapy and prophylaxis to reduce and prevent uric acid crystal formation.
A 1-year-old boy presents to the ED with shortness of breath and cough. Vital signs are BP 90/60 mm Hg, HR 110 beats per minute, RR 40 breaths per minute, oxygen saturation of 91% on room air, and T 97.5°F. Physical exam reveals intercostal retractions, rhinorrhea, and wheezing. Which of the following is the most appropriate next step in management? Administer nebulized albuterol Administer racemic epinephrine Endotracheal intubation Nasal suctioning
Nasal suctioning Nasal suctioning is the first step in management for a child presenting with bronchiolitis. Bronchiolitis is a viral infection of the lower airways most commonly caused by respiratory syncytial virus respiratory. Inflammation of the lower respiratory tracts leads to edema, bronchospasm, and increased mucus production within the bronchioles. The resulting increase in airway resistance leads to respiratory distress and increase work of breathing. Since infants are obligate nasal breathers, and the nasal passages account for 50% of airway resistance, nasal suctioning is essential in helping decrease work of breathing. In addition to nasal suctioning, supplemental oxygen should be administered for oxygen saturation below 90%.
A 14-month-old girl without significant past medical history presents with cough and difficulty breathing. Her parents note that she developed a low-grade fever and nasal congestion two days ago. Today she has had an increased cough and mild difficulty breathing but seems to be feeding well and is still active. She has not had similar symptoms previously. On examination, she has a temperature of 38.2 ºC, heart rate 120 beats per minute, respiratory rate 42 breaths per minute and oxygen saturation 97% on room air. She is well-appearing with rhinorrhea and crusting at the nares. She has scattered expiratory wheezes with fine crackles noted on auscultation. What is the next step in management of this patient? Dexamethasone Nasal suctioning Nebulized albuterol Nebulized hypertonic saline
Nasal suctioning This patient has signs and symptoms consistent with bronchiolitis, a lower respiratory tract infection most commonly due to infection with respiratory syncytial virus. It primarily affects young children under two years of age. There is often a one to three day prodrome of mild upper respiratory symptoms such as low-grade fever and rhinorrhea before the onset of cough and respiratory difficulty. Characteristic exam findings include nasal drainage, tachypnea, cough, intercostal and subcostal retractions, wheezing and fine or coarse crackles. The patient noted above has mild disease given her well appearance, minimal wheezing, normal oxygen saturation and lack of retractions or accessory muscle use. Patients with mild disease benefit most from supportive care and anticipatory guidance. This includes nasal suctioning, ensuring adequate hydration and education of the parents.
A 58-year-old man presents to the emergency department with abdominal pain and distension. He states his symptoms started last night and are accompanied by nausea and vomiting. Abdominal exam is notable for distension as well as generalized discomfort and tenderness. Radiography is performed, as seen above. Which of the following is the best treatment for this patient? Ciprofloxacin and metronidazole Endoscopic sphincterotomy Heparin Nasogastric tube placement
Nasogastric tube placement This patient is presenting with symptoms suggestive of a small bowel obstruction. Small bowel obstructions present with abdominal distension, absent or high pitched bowel sounds, nausea and vomiting, and an absence of passage of stool or flatus. The pathophysiology is most commonly related to adhesion formation, which entraps the bowel and causes the aforementioned symptoms. Previous surgery or trauma can lead to the formation of adhesions, thus, a history of abdominal surgery is a key risk factor. Other possible etiologies of small bowel obstructions include malignancy or tumor and hernias. The diagnosis can be supported with radiography, which will demonstrate dilated bowel with air fluid levels or the "stack of coins" sign. The treatment of a small bowel obstruction typically involves keeping the patient nothing by mouth, placing a nasogastric tube to decompress the gastrointestinal tract (though this may not always be needed), administering antiemetics, and fluid resuscitation. The obstruction may resolve spontaneously, however, any patient presenting with unstable vital signs, fever, or peritoneal signs require an emergency surgery evaluation.
Which of the following is considered a definitive airway? High flow nasal cannula Intubating laryngeal mask airway Nasotracheal tube Oropharyngeal airway
Nasotracheal tube
An 8-year-old boy with moderate persistent asthma presents to the emergency department with difficulty breathing. His parent is giving 4 puffs of his home albuterol every 2 hours without effect. He is awake and alert but seems breathless, as he speaks in incomplete sentences. He has moderate accessory muscle use and tachypnea. He has a prolonged expiratory phase and diffuse expiratory wheezes on auscultation. Which of the following is the most appropriate next step? Intramuscular ketamine Intravenous magnesium Nebulized albuterol Subcutaneous terbutaline
Nebulized albuterol The cornerstone for treatment of asthma remains a short-acting beta-agonist, such as albuterol, along with corticosteroids.
A 25-year-old healthy man presents to the emergency department with several days of sore throat associated with fever and voice change. Examination is significant for a temperature of 101.5°F (38.6°C), HR 110 bpm, SpO2 99% on room air, right tonsillar erythema and swelling with uvula deviation to the left, no pooling of oral secretions, and tender anterior cervical lymphadenopathy. Which of the following is the most appropriate management of this patient's condition? Clindamycin orally Needle aspiration Otolaryngology consult Penicillin intramuscular injection
Needle aspiration This patient's presentation is consistent with a peritonsillar abscess. Patients typically present with a sore throat, fever, odynophagia, or dysphagia, and examination will reveal unilateral swelling and displacement of the tonsil and uvula deviation. A peritonsillar abscess can be differentiated from peritonsillar cellulitis by the presence of tonsillar swelling and uvula deviation. Treatment for peritonsillar abscess consists of drainage, and some prefer the initial method is needle aspiration. Needle aspiration is usually less painful, but can have a higher recurrence rate than the use of incision and drainage.
A 30-year-old man presents to the emergency department after a motor vehicle collision, reporting chest and back pain. His vital signs are significant for a blood pressure of 85/50 mm Hg, heart rate of 115 bpm, and oxygen saturation of 92% on room air. Exam reveals ecchymosis of the left anterior chest wall, no crepitus, diminished breath sounds on the left hemithorax, and a Glasgow Coma Scale score of 15. Which of the following is the best next step in management? Bedside chest X-ray Bilateral tube thoracostomy in the fourth midaxillary intercostal space Needle thoracostomy in the left second midclavicular intercostal space Rapid sequence intubation
Needle thoracostomy in the left second midclavicular intercostal space This patient's presentation is consistent with tension pneumothorax, a clinical diagnosis that requires immediate intervention. These patients will present with diminished breath sounds, distended neck veins, hypotension, and tracheal deviation. Diagnosis should be made clinically in patients with hypotension and diminished breath sounds unilaterally. In most cases, this will follow chest trauma or positive pressure ventilation. Treatment requires needle thoracostomy of the affected side at the second intercostal space in adults. This should be performed prior to imaging studies, as delays in treatment allow for worsening obstructive shock and can ultimately lead to cardiac arrest. Needle decompression converts this to an open pneumothorax and will need to be followed by tube thoracostomy once the patient is stabilized. Patients with a persistent pneumothorax following tube thoracostomy should be evaluated for a possible bronchial injury.
A 65-year-old man presents in respiratory distress. He has a known history of hypertension, heart failure, diabetes mellitus, and hyperlipidemia. He arrives unable to speak in more than two-word phrases secondary to severe distress and tachypnea. Vital signs are remarkable for BP 220/130 mm Hg, HR 105 bpm, RR 40/min, and pulse oximetry 75%. Accessory muscle use, intercostal retractions, and diffuse rales are noted on examination. In addition to placing the patient on BPAP, which of the following should be administered next for blood pressure control? Esmolol Fenoldopam Nicardipine Nitroglycerin
Nitroglycerin In patients presenting with an acute hypertensive emergency with acute pulmonary edema, the mainstay of therapy is vasodilators, predominantly nitrates. Hypertensive Emergency Hypertension with acute end-organ system injury Encephalopathy, cardiac ischemia, kidney ischemia Objectives: gradually reduce MAP by approximately 10-20% in the first hour and by a further 5-15% over the next 23 hours Reduction of MAP > 25% may cause end-organ ischemia IV antihypertensives ( such as labetalol or nicardipine)
A 64-year-old man with a history of coronary artery disease, hypertension and hyperlipidemia reports substernal chest pressure. The pain is worse when he walks up stairs and improves with rest. An electrocardiogram is performed and shows ST depression with T wave inversion in the inferior leads. Troponin is positive. Which of the following is the most likely diagnosis? Inferior ST-elevation myocardial infarction Non-ST-elevation myocardial infarction Pericarditis Unstable angina
Non-ST-elevation myocardial infarction Acute coronary syndrome refers to a spectrum of disease when cardiac ischemia results from an imbalance between myocardial oxygen demand and delivery, most often as a result of atherosclerotic plaque disruption and thrombosis in a coronary artery, reducing or blocking coronary blood flow. n non-ST segment elevation myocardial infarction, cardiac biomarkers are elevated, indicating infarction; however, ECG changes do not include ST elevation. Instead ST segment depression or T wave inversion may be present, indicating ischemia.
A 62-year-old man with a history of myocardial infarction three months ago, diabetes, hypertension, and chronic obstructive pulmonary disease presents to the emergency department by ambulance for shortness of breath. The patient reports worsening dyspnea over 24 hours with mild chest discomfort. He denies fever. He reports swelling in his legs over several weeks. He is sometimes compliant with his medications. Paramedics found him with an oxygen saturation of 88% on room air when they arrived at his home. Current vital signs show a HR of 117 bpm, BP of 201/112 mm Hg, RR of 30/min, and oxygen saturation of 100% on nonrebreather mask. Diffuse crackles and mild wheezing are heard diffusely on lung examination. The patient has a protuberant abdomen and 2+ pitting edema of both legs. While awaiting further diagnostic tests, what is the most appropriate treatment of this patient? Noninvasive positive pressure ventilation and intravenous antibiotics with piperacillin-tazobactam and vancomycin Noninvasive positive pressure ventilation and intravenous heparin Noninvasive positive pressure ventilation, nebulized albuterol and ipratropium, and intravenous methylprednisolone Noninvasive positive pressure ventilation, nitroglycerin drip, and intravenous furosemide
Noninvasive positive pressure ventilation, nitroglycerin drip, and intravenous furosemide
A 40-year-old woman presents to the ED complaining of neck pain. For the past three days, she has had a fever, myalgias, and increasing fatigue. Vital signs are BP 130/90 mm Hg, HR 100 beats per minute, RR 14 breaths per minute, oxygen saturation of 97% on room air, and T 100.4°F. On physical exam, she has tenderness over her anterior, midline neck. Which of the following is the most appropriate treatment? Levothyroxine Methimazole Nonsteroidal anti-inflammatory drugs Propranolol
Nonsteroidal anti-inflammatory drugs The treatment for most cases subacute thyroiditis is nonsteroidal anti-inflammatory drugs (NSAIDs). Subacute thyroiditis is suspected to be caused by a viral infection or a post-viral inflammatory process. Common symptoms include fever, myalgias, fatigue, and malaise. Patients will complain of anterior neck pain, which is often localized over the area of the thyroid but can radiate to the jaw, throat, upper neck, or ears. The hallmark physical exam finding is tenderness over the thyroid gland. Subacute (de Quervain) thyroiditis Preceded by viral URI Anterior neck pain Suppressed TSH, elevated ESR Low uptake on thyroid scan Tx: high-dose ASA or NSAID
A 65-year-old man presents for urinary incontinence. He states that he has had difficulty with urinating and started urinating on himself a few months ago. The problem has been worsening and now he is embarrassed to go out in public for fear of having an incident. He describes difficulty initiating micturition and fully emptying his bladder, and additionally seems to constantly leak a small amount of urine. He reports no fevers, weakness, paresthesias, fecal incontinence, constipation, hematuria, or dysuria. Vital signs and physical exam are unremarkable. Which of the following type of incontinence is this patient likely experiencing? Functional incontinence Overflow incontinence Stress incontinence Urge incontinence
Overflow incontinence Overflow Incontinence Sx: difficulty urinating, incomplete emptying, fullness, pressure, dribbling Causes: detrusor underactivity, bladder outlet obstruction Treatment: alpha-blockers
A 60-year-old woman presents to the emergency department with chest pain. ECG shows ST-segment elevation in leads V2-V4. Vitals are BP 80/40 and HR 65. In addition to dobutamine, the most appropriate next step is administration of which of the following? Epinephrine Milrinone Norepinephrine Phenylephrine
Norepinephrine For patients in cardiogenic shock with a systolic blood pressure less than 90, the addition of norepinephrine will help to mitigate the vasodilatory effects of dobutamine thereby preventing further hemodynamic instability. Patients with ST-segment elevation myocardial infarction complicated by cardiogenic shock require emergent hemodynamic support as a temporizing measure before revascularization. Dobutamine is a beta-agonist that increases myocardial contractility through stimulation of the beta-1 receptors. However, it can lead to vasodilation through activation of beta-2 receptors which can worsen blood pressure. Norepinephrine is a vasopressor with beta-1 and alpha-1 activity. Through stimulation of the beta-1 receptors, norepinephrine improves myocardial contractility.
A 43-year-old woman presents to the emergency department with acute-onset headache and right eye pain. On examination, you notice she has a cloudy cornea with a fixed midposition pupil. She is stating that, in addition to pain, she is also starting to see halos form around lights. Which of the following mechanisms is the main cause of this condition? A sudden increase in blood pressure Accumulation of fluid in the retrobulbar space Obstruction of aqueous humor outflow Tissue damage to the lens
Obstruction of aqueous humor outflow The patient is presenting with acute angle-closure glaucoma, a vision-threatening emergency. The main cause of acute angle-closure glaucoma is obstruction of aqueous humor outflow, usually by the lens or the iris obstructing the trabecular meshwork, which prevents flow of aqueous humor from the posterior chamber to the anterior chamber. Patients usually present with eye pain, a fixed midposition pupil, conjunctival injection, a hazy-appearing cornea, and elevated intraocular pressures. This increase in pressure will eventually cause corneal edema, which often leads to a hazy appearance on physical exam and can sometimes cause visual disturbances in patients. Initial management of acute angle-closure glaucoma is aimed at rapidly decreasing intraocular pressure by blocking the production of aqueous humor (topical beta-blocker, topical alpha-1 agonist, carbonic anhydrase inhibitors), facilitating drainage of aqueous humor (topical pilocarpine), and reducing the volume of vitreous humor (mannitol)
A 55-year-old man presents to the emergency department with a cough and altered mental status. Past medical history is unknown. Vital signs are remarkable for temperature 39°C, HR 130 bpm, RR 30/min, BP 70/30 mm Hg, and pulse oximetry 89%. On exam, the patient is toxic appearing, tachycardic, and tachypneic, with rales noted bilaterally. Capillary refill is greater than 5 seconds and the mucosal membranes are dry. The patient receives a 30 mL/kg fluid bolus but remains hypotensive. A bedside ultrasound is performed of the IVC and is shown above. What is the next step in management? Additional fluid bolus Administer hydrocortisone IV Obtain central venous access and start norepinephrine Obtain central venous access and start vasopressin
Obtain central venous access and start norepinephrine The patient above is suffering from septic shock, a form of distributive shock. In this form of shock, there is relative intravascular volume depletion due to marked systemic vasodilation. Management of the shock and hypotension secondary to sepsis initially focuses on fluid resuscitation, with the recommended amount being 30 mL/kg of normal saline. The patient above shows a plethoric IVC, suggesting he would not benefit from any additional fluids. Therefore, the next step in the setting of septic shock would be the use of vasopressor therapy, with the current first-line recommendation being norepinephrine.
A 10-year-old boy presents to the pediatrician with his mother. She states the patient has become difficult to deal with over the past several years. He refuses to follow instructions, both in school and at home, and has been found trying to skip class multiple times by hiding in the bathroom. She reports he is verbally aggressive when told something he does not like. The patient has several friends but is hostile to peers outside his group of friends. Initially, she thought this was just due to the move they made 2 years ago, but his behaviors have persisted. Which of the following is the most likely diagnosis? Adjustment disorder Antisocial personality disorder Conduct disorder Oppositional defiant disorder
Oppositional defiant disorder Oppositional Defiant Disorder (ODD) At least 6 months of symptoms in a child Angry, argumentative, defiant behavior Vindictiveness Evaluate for other comorbidities Treat with psychotherapy
A 27-year-old man presents to the emergency department in January with a nonproductive cough, fever, headache, myalgias, weakness, nausea, and vomiting for the past three days. His temperature is 102°F (38.9°C), blood pressure is 127/68 mm Hg, pulse is 110/min, respiratory rate is 16/min, and oxygen saturation is 99% on room air. Physical exam is notable for a fatigued-appearing young man. Lungs are clear to auscultation. Which of the following is the best treatment for this patient? Azithromycin Moxifloxacin Oral rehydration and rest Oseltamivir
Oral rehydration and rest Influenza Patient presents with sudden onset of fever, headache, cough, myalgia, sore throat, fatigue Diagnosis is made clinically, can be confirmed with reverse transcription polymerase chain reaction (RT-PCR) or viral culture Treatment is mainly supportive or oseltamivir for patients at increased risk for severe disease Start oseltamivir as early as possible, after 48 hours it may not confer any benefit Most common cause of viral pneumonia in adults New vaccine needed yearly Monitor patients for postinfluenza pneumonia
A healthy 30-year-old woman presents to the emergency department for frequency and burning on urination. She reports no vaginal discharge, fever, nausea, or back pain, and on exam, she has normal vital signs and does not have vaginal discharge, cervical motion tenderness, or costovertebral angle tenderness. Her urine pregnancy test is negative, and her urinalysis is significant for 8 WBC/hpf and positive for bacteria and nitrites. Which of the following is the most appropriate treatment for this patient's condition? Oral azithromycin plus intramuscular ceftriaxone in the emergency department Oral ciprofloxacin for 7 days Oral phenazopyridine for 3 days Oral trimethoprim-sulfamethoxazole for 5 days
Oral trimethoprim-sulfamethoxazole for 5 days This patient's presentation and lab findings are consistent with uncomplicated acute cystitis. Symptoms of acute cystitis include frequency, urgency, or hesitancy with urination. On exam, suprapubic tenderness may be noted, but patients should not have systemic symptoms, such as fever, vomiting, or costovertebral angle tenderness. In the majority of patients, Escherichia coli is the causative bacteria. Patients with uncomplicated acute cystitis may be treated with oral trimethoprim-sulfamethoxazole for 5 days
A 70-year-old man with atrial fibrillation on warfarin daily and dementia presents to the emergency department after being referred from a coagulation clinic for elevated international normalized ratio. He is asymptomatic and reports no bleeding, bruising, or melena but states he may have unintentionally taken extra doses of warfarin. His vital signs are significant for an irregular heart rate of 88 bpm and a blood pressure of 150/80 mm Hg. His laboratory values are significant for an international normalized ratio of 11, and his hemoglobin is unchanged from baseline. Which of the following is the best next step in the management of this patient? No treatment and repeat labs in 24-48 hours Oral vitamin K, hold next warfarin dose, and repeat labs in 24-48 hours Reversal with four-factor prothrombin complex concentrate Reversal with protamine sulfate
Oral vitamin K, hold next warfarin dose, and repeat labs in 24-48 hours This patient is presenting with asymptomatic supratherapeutic international normalized ratio (INR) due to warfarin toxicity. By interfering with the vitamin K cycle, warfarin prevents the activation of clotting factors II, VII, IX, and X. The effect of warfarin is monitored with prothrombin time and INR, with a goal INR commonly between 2-3.5, depending on the condition being treated. Patients on warfarin who present with elevated INR and no significant bleeding should be treated based on the INR level. Patients with an INR greater than 10 should receive oral vitamin K, hold the next warfarin dose, and repeat labs in 24-48 hours. Patients without bleeding and an INR less than 10 should lower or omit their upcoming warfarin doses and have the INR rechecked to ensure it is therapeutic.
An 18-year-old man presents to the emergency department with ear pain. He states that pain started after he went swimming in a local pond several days ago. He also reports purulent drainage from his ear. Physical examination reveals an erythematous, edematous external auditory canal. The tympanic membrane is partially visible, and tenderness with tragal pressure is also present. Which of the following is the best treatment for this patient? Oral amoxicillin Oral ciprofloxacin Otic acetic acid with hydrocortisone Otic ciprofloxacin with hydrocortisone
Otic ciprofloxacin with hydrocortisone This patient has acute otitis externa, which most commonly presents with ear pain, pruritus, purulent discharge, and hearing loss. Physical exam findings include an edematous, erythematous external auditory canal with debris present. Tenderness with tragal pressure or when the auricle is manipulated or pulled may also be present. Risk factors for development of otitis externa include trauma and swimming or other water exposure. The most common pathogens responsible for causing otitis externa are Pseudomonas aeruginosa and Staphylococcus aureus. For mild otitis externa, which is characterized by minor discomfort and pruritus with minimal canal edema, otic acetic acid with hydrocortisone may be used. Moderate disease is characterized by an intermediate degree of pain and pruritus, along with a partially occluded ear canal. Otic ciprofloxacin with hydrocortisone would be appropriate treatment for moderate infection.
A 27-year-old woman presents with acute onset of right pelvic pain and nausea that started six hours ago. She reports two prior episodes that were similar, but symptoms with those episodes resolved within one hour. On examination, she appears uncomfortable and is mildly tachycardic. She has tenderness to palpation of the right adnexa. A transvaginal ultrasound is ordered. What is the most common ultrasound finding in ovarian torsion? Complete arterial obstruction Heterogeneous appearance of the ovarian stroma Ovarian enlargement Whirlpool sign
Ovarian enlargement Ovarian torsion is a gynecological emergency and is caused by the twisting of the ovary and fallopian tube on the vascular pedicle. Patients present with acute onset of unilateral pelvic pain often accompanied by nausea and vomiting. Ultrasound is the test of choice for diagnosing ovarian torsion. The most common finding is ovarian enlargement due to venous and lymphatic engorgement.
A 25-year-old man with a medical history of asthma, seasonal allergies, and tobacco use presents to the emergency department with a severe periorbital headache. It started suddenly this evening and has not been improving. Physical exam is notable for an uncomfortable young man with right-sided lacrimation and rhinorrhea. Which of the following is the best initial treatment for this patient? Ibuprofen, diphenhydramine, and prochlorperazine Oxygen Sumatriptan Verapamil
Oxygen Cluster headaches typically present in young men with severe unilateral headaches that are associated with a sudden onset of unilateral symptoms including lacrimation and rhinorrhea. Other autonomic symptoms that can be seen include sweating, miosis, and ptosis. The treatment of an acute episode is oxygen, which can be administered and resolve symptoms quickly with minimal side effects. Other possible acute interventions include triptans.
A 55-year-old man with a history of tobacco abuse, hypertension, and diabetes mellitus controlled with metformin presents to the ED with severe, sudden-onset retrosternal chest pressure that began while he was moving furniture in his apartment. Paramedics gave him sublingual nitroglycerin that reduced the pain somewhat. He appears diaphoretic and reports nausea. His ECG does not show ST segment elevation or depression or T wave abnormalities. His serum troponin I level is 0.84 ng/mL. He is given chewable aspirin 325 mg. Which of the following is the best next step in management? CT coronary angiography Nuclear medicine testing P2Y12 antagonist Tenecteplase
P2Y12 antagonist Acute Coronary Syndrome: Management Reperfusion PCI - increased survival, decreased ICH and recurrent MIPCI center: < 90 minutes "door to device" timeNon-PCI center: transfer for PCI if "door to device" time can be <120 minutes Thrombolysis - if PCI is not available or "door to device" time > 120 minutesAdminister within 30 minutes, can be given up to 12 hours of symptom onset Contraindications: ICH, intracranial malignancy, stroke within 3 months, aortic dissection Medical Therapy Oxygen - indicated for O2 sat < 90%, dyspnea, heart failure Nitroglycerin - relieves ongoing chest pain, lowers BP Morphine - relieves pain, reduces work of breathing in setting of pulmonary edema only used if nitroglycerin fails to relieve the pain can be associated with negative outcome Beta Blockers - prevent recurrent ischemia and dysrhythmia Antiplatelet agents - reduce recurrent coronary artery thrombosis, stent thrombosis, and death Aspirin - given before PCI and continued indefinitely P2Y12 inhibitor (clopidogrel, prasugrel, ticagrelor) - continue for 1 year if stent placed GPIIb/IIIa antagonists for patients undergoing PCI Anticoagulation - if LV thrombus or Afib present and all patients receiving thrombolytic therapy ACE Inhibitors - reduce cardiovascular events, prevent LV remodeling High dose statin - lipid lowering, lowers risk of death, recurrent MI, and stroke
Abnormal Uterine Bleeding (AUB) causes (PALM & COEIN)
PALM: Structural Causes Polyps Adenomyosis Leiomyoma Malignancy & Hyperplasia COEIN Coagulopathy Ovulatory dysfunction Endometrial Iatrogenic Not yet Classified
A patient presents to the emergency department for a painful, swollen finger. Which of the following features, if present, would be most consistent with a diagnosis of flexor tenosynovitis? Finger held in full extension Localized swelling along the flexor tendon Pain on passive extension of the affected finger Percussion tenderness isolated to the distal phalanx
Pain on passive extension of the affected finger Flexor tenosynovitis is a surgical emergency that requires rapid bedside diagnosis to expedite appropriate management. Penetrating trauma precedes most cases, although the injury itself may have been innocuous, leading to bacterial seeding of the deep tendon sheath. Skin flora, such as Staphylococcus aureus and Streptococcus spp, are most commonly identified. On examination, Kanavel signs may be identified that can help with the diagnosis of flexor tenosynovitis: pain on passive extension of the affected finger (often the first finding), finger held in partial flexion, tenderness of the full tendon sheath, and fusiform swelling of the digit. Kanavel signs have a sensitivity of approximately 90% in the diagnosis of flexor tenosynovitis. Patients with findings concerning for this diagnosis should rapidly be treated with intravenous antibiotics such as vancomycin plus extended-spectrum penicillins. They should have emergent hand surgery consultation, as this infection can rapidly spread through the fascial planes of the hand. Inappropriate management of flexor tenosynovitis can lead to loss of function in the affected hand.
A 75-year-old woman, discharged one day ago following admission for an inferior myocardial infarction, presents with acute dyspnea and is noted to have a new holosystolic murmur at the apex that radiates to her axilla. A chest radiograph is performed and shown above. Which of the following complications of her myocardial infarction is most likely? Papillary muscle rupture Pericarditis Right ventricular infarction Ventricular free wall rupture
Papillary muscle rupture The patient has developed papillary muscle rupture. The papillary muscles are fingerlike projections arising from the left ventricular wall, which anchor the leaflets of the mitral valve to the ventricle and prevent back flow of the leaflets during systole. Rupture of the papillary muscle, together with rupture of the ventricular free wall, and rupture of the interventricular septum, are the catastrophic mechanical complications that can occur with acute myocardial infarction (MI). Papillary muscle rupture is seen in approximately 1 percent of patients with acute MI. It typically occurs three to five days after infarction and is most common after inferior MI. Clinically, patients present with sudden development of mitral regurgitation, and the presentation can range from heart failure to cardiogenic shock. Patients may complain of dyspnea, and rales are typically present on lung auscultation, indicating pulmonary edema. The presence of a new holosystolic murmur, loudest at the apex and radiating to the axilla, indicates mitral regurgitation. Echocardiogram will demonstrate regurgitant mitral flow. Prompt recognition and treatment improves survival. Emergent surgery for mitral valve repair or replacement is often necessary.
A 26-year-old woman with a history of polycystic ovary syndrome presents with five days of diffuse headache that has not responded to over-the-counter medications. She also reports one day of intermittent blurring and "blacking out" of vision with positional changes. Temperature is 37°C (98.6°F), pulse rate is 78/min, respirations are 20/min, blood pressure is 135/90 mm Hg, and SaO₂ is 99%. The physical exam is notable for an obese, uncomfortable-appearing woman who is moving all extremities and answering you appropriately. Her visual fields are intact, but visual acuity is 20/40 OD and 20/70 OS, and she tells you that she has never had vision issues. What is an additional physical exam finding that is likely to be associated with the condition? Intraretinal hemorrhages and dilated tortuous retinal veins Mid-dilated poorly reactive pupils and elevated intraocular pressures Nuchal rigidity Papilledema
Papilledema Papilledema is the hallmark funduscopic finding of idiopathic intracranial hypertension and is typically bilateral and symmetric. Idiopathic intracranial hypertension primarily affects overweight women of childbearing age and has been associated with polycystic ovary syndrome. Symptoms include intermittent visual changes, headache (most common), and pulsatile tinnitus. Papilledema Patient presents with acute onset of intermittent headaches and blurred vision PE will show optic disk swelling Focus on finding intracranial pathology Most commonly caused secondary to increased intracranial pressure Treatment is the treatment of the underlying disorder
A 2-year-old boy presents to the ED at 3:00 AM for three days of cough and shortness of breath. Symptoms are worse at night. His symptoms were preceded by runny nose, low-grade fever, and nasal congestion. He appears calm but has stridor at rest, supraclavicular retractions, a hoarse voice, and drooling on exam. His lungs are clear to auscultation. This condition is most commonly caused by which of the following? Coxsackievirus Haemophilus influenzae Parainfluenza virus Respiratory syncytial virus
Parainfluenza virus Laryngotracheitis (Croup) Patient will be a nontoxic-appearing child, 6 months to 3 years old URI symptoms with barky seal-like cough, inspiratory stridor, low-grade fever X-ray will show steeple sign on PA view Most commonly caused by parainfluenza virus Treatment is steroids, aerosolized epinephrine
A 6-year-old girl with a history of sickle cell disease presents to the emergency department with a fever. Her parent reports that she has been less playful but is continuing to eat and drink well. Her vital signs on arrival are T 39.0°C, HR 130 bpm, RR 18/min, BP 94/60 mm Hg, and SpO2 98% on room air. Her physical examination shows normal tympanic membranes, clear lungs, no joint swelling, and no rashes. Her chest X-ray is shown above. Her laboratory workup is notable for a white blood cell count of 16,000/µL, a hemoglobin of 7.5 g/dL, and a reticulocyte count of 0.4%. Which of the following is the most likely cause of her symptoms? Acute chest syndrome Pain crisis Parvovirus B19 infection Splenic sequestration
Parvovirus B19 infection Aplastic crisis is an important complication of sickle cell disease in which the body is unable to make enough reticulocytes to compensate for hemolysis. The most common cause of aplastic crisis is infection, and human parvovirus B19 is the most frequently implicated. Aplastic crisis is seen more often in children than adults. Patients typically present with increasing fatigue and signs of infection, including fever and tachycardia. Labs will generally show anemia that is below the patient's baseline. The reticulocyte count is minimal, usually less than 0.5%. Aplastic crisis is usually self-limited and resolves in about 7-10 days. Care is supportive, with transfusions given as needed. All patients with sickle cell disease presenting with a fever should receive empiric antibiotics, even if a viral cause is found, and be worked up for acute chest crisis and sepsis, as they are at increased risk.
A 16-year-old boy presents to the emergency department with acute left knee pain. He was riding his bicycle when his back tire was hit by a car. He reached his left leg outward while attempting to prevent his bike from falling and felt a "pop" and then excruciating knee pain. The patient is lying in bed with his left knee slightly bent, but he is unable to move the leg or knee. There are no obvious bony deformities. You feel boggy swelling and tenderness over the patellar region. His lateral knee X-ray is shown above. What is the most likely diagnosis? Patellar fracture Patellar subluxation Patellar tendon rupture Quadriceps tendon rupture
Patellar tendon rupture The patella is a sesamoid bone embedded in the quadriceps tendon. Plain radiographs are the mainstay of diagnosis for patellar injuries. A complete tear of the patellar tendon is uncommon but most often occurs in patients under age 40. The injury typically occurs when the flexed knee sustains a force that suddenly increased the degree of flexion. The patient with a patellar tendon rupture often has significant knee swelling and tenderness and cannot fully extend the knee or lift a leg from a bed. Other associated injuries, as seen on radiograph, include a superior patellar dislocation. Initial care of this patient includes rest, ice, compression, elevation, analgesics, non-weight-bearing status with crutches, and prompt orthopedic referral.
An 80-year-old man with a history of diabetes mellitus, obstructive sleep apnea, peptic ulcer disease, and ischemic cardiomyopathy presents to the ED in cardiopulmonary arrest. EMS personnel are delivering chest compressions. The patient has an intraosseous line in the left shoulder through which he has received three doses of epinephrine 1 mg and amiodarone 300 mg. EMS placed a supraglottic airway device prior to arrival. The patient continues to be pulseless with ventricular tachycardia on the cardiac monitor. He is defibrillated at 300 J. What is the most important action to take after defibrillation? Administer amiodarone and epinephrine Obtain central venous access Perform chest compressions Secure a definitive airway
Perform chest compressions
A 48-year-old woman with a history of lupus and thromboembolic disease presents to the emergency department with a 6-day history of chest pain and shortness of breath. Vital signs are remarkable for BP 85/40 mm Hg, RR 30/min, and pulse oximetry 91%. JVD and tachycardia with clear breath sounds are noted on examination. ECG is performed and is demonstrated above. Which of the following is the best next step in management? Activate cardiac catheterization Administer alteplase Perform needle decompression Perform pericardiocentesis
Perform pericardiocentesis Pericardial tamponade occurs when fluid collects in the pericardial sac, restricting diastolic filling first and systolic function later. Nontraumatic causes of cardiac tamponade include malignancy, acute pericarditis, uremia, bacterial pericarditis, chronic pericarditis, spontaneous hemorrhage, systemic lupus, post-radiation, and myxedema. Physical examination may reveal a tachycardia, low systolic blood pressure, and a narrow pulse pressure. JVD, distant heart sounds, and hypotension may be present and is classically known as the Beck triad. Chest X-ray may show an enlarged cardiac silhouette, however, the cardiac silhouette may also be completely normal and is based on the rate of accumulation. ECG will typically show low voltage and may demonstrate the classic finding of electrical alternans. The initial treatment should involve volume expansion with a 500 to 1,000 mL fluid bolus. However, this is only a temporizing measure, and pericardiocentesis is necessary for definitive therapy. In the setting of hemodynamic instability, pericardiocentesis should be performed in the emergency department.
A 48-year-old woman presents to the ED with chest pain and nausea. She describes right subcostal postprandial cramping and two episodes of nonbloody, nonbilious emesis today. She has inspiratory arrest during deep palpation of her right upper quadrant. Which of the following sonographic findings is consistent with her most likely diagnosis? Fixed mural polyp Intraparenchymal abscess Pericholecystic fluid Wall thickness of 3 mm
Pericholecystic fluid The most common cause of acute cholecystitis is cholelithiasis, the presence of an obstructing gallstone within the gallbladder. Patients typically present with epigastric or right upper quadrant abdominal discomfort (biliary colic) that may be referred to the right shoulder. Symptoms classically peak during sleeping hours. Pain may be postprandial or periprandial. Murphy sign on physical examination is considered positive if the patient has inspiratory pause or arrest with deep palpation of the right upper quadrant. Computed tomography (CT) may demonstrate a thickened gallbladder wall (greater than 4 to 5 mm), pericholecystic fluid, or dilated intra- or extrahepatic biliary ducts.
Which of the following clinical features is most suggestive of appendicitis? Fever with abdominal pain Periumbilical pain that migrates to the right lower quadrant Rebound tenderness Vomiting followed by the onset of pain
Periumbilical pain that migrates to the right lower quadrant Appendicitis is the most common surgical emergency in children. Appendicitis usually begins with obstruction of the appendiceal lumen, most commonly due to a fecalith. High-risk clinical features that suggest appendicitis include a recent onset of abdominal pain that has migrated from the periumbilical region to the right lower quadrant over time, as well as fever, vomiting, and anorexia. Diagnosis is typically made with imaging. In children, ultrasound can be used to avoid the use of ionizing radiation. However, CT scanning has superior sensitivity and specificity. Treatment is surgical, with antibiotics being given before an appendectomy.
A 45-year-old woman who does not have housing presents with an intensely pruritic generalized rash that is worse at night. Excoriations and wavy scaly lines are present in some of the finger web spaces, and papules are present elsewhere except for the neck and face. Scabies mites are present on a wet mount of a skin scraping. Which of the following is first-line treatment? Ivermectin 200 mcg per kg orally on day 1, then repeat in 1 week Ivermectin 200 mcg per kg orally on days 1, 2, 8, 9, and 15 Permethrin 1% cream on day 1, then reapply in 2 weeks Permethrin 5% cream on day 1, then reapply in 1 week
Permethrin 5% cream on day 1, then reapply in 1 week Scabies is a common health issue. The arthropod mites Sarcoptes scabiei var. hominis mate on the skin surface, after which the female mite burrows under the skin, laying eggs for several weeks. The eggs hatch and the new mites breach the burrow and multiply on the skin surface. Transmission occurs via direct skin-to-skin contact. Risk factors include overcrowding, poor hygiene, being without housing, dementia, and sexual contact. The primary skin lesions are pruritic papules, pustules, vesicles, and nodules, and the pathognomonic finding is a burrow. Permethrin 5% cream on day 1, then reapply in 1 week is considered first-line treatment for scabies
Which of the following neurovascular structures is frequently injured with a tibiofemoral dislocation? Femoral artery Peroneal nerve Posterior tibial artery Posterior tibial nerve
Peroneal nerve Knee Dislocation History of violent trauma PE will show dimple sign with posterolateral dislocation Most common causesAnterior dislocation: hyperextension of the kneePosterior dislocation: a direct blow to the anterior tibia with the knee flexed Treatment is reduction and knee splint with 15-20 degrees of flexion Complications can involve popliteal artery and peroneal nerve
Which of the following matches the laboratory finding with the correct diagnosis? Auer rod - chronic lymphocytic leukemia Clue cell - acute myelogenous leukemia Philadelphia chromosome - chronic myelogenous leukemia Smudge cell - acute lymphocytic leukemia
Philadelphia chromosome - chronic myelogenous leukemia
A 21-year-old man presents with a stab wound to the right chest. His vitals are HR 157 bpm, BP 81/43 mm Hg, RR 28/min, and SpO2 91%. The patient is intubated and packed red blood cells are started. Physical examination reveals a bleeding wound to the right chest, a midline trachea, and decreased breath sounds on auscultation of the right hemithorax. Which of the following should be performed next? Application of a pelvic binder Placement of a right thoracostomy tube Right chest thoracotomy Transfer to operating room
Placement of a right thoracostomy tube This patient has a stab wound to the right chest that may be causing hemorrhagic shock from a hemothorax or obstructive shock from a pneumothorax or pericardial tamponade. The presence of a midline trachea makes pneumothorax as the sole cause of hypotension unlikely. Hemothorax is more likely, and a chest thoracostomy tube can both be diagnostic and guide further management. f the chest tube puts out > 1,500 mL of blood immediately or > 200 mL/hour for 3 hours, the patient likely has a significant intrathoracic injury that will require urgent operating room thoracotomy. Removal of blood from the hemothorax will allow the right lung to expand and improve oxygenation and possibly blood pressure.
A 23-year-old G1P0 woman at 30 weeks gestation presents to the emergency department for vaginal bleeding. She has had no prior prenatal care. She reports that she is soaking through one pad every eight hours. She denies abdominal cramping or a gush of water and reports good fetal movement. Her vital signs are T 37.2°C, HR 100, BP 105/70 mmHg, and RR 18. Her abdomen is soft, nontender, and consistent with a 30 week pregnancy. Which of the following is the most likely cause of this patient's symptoms? Abruptio placentae Placenta previa Threatened abortion Velamentous cord insertion
Placenta previa Placenta Previa Painless vaginal bleeding - Most often during the third trimester Diagnosis is made by ultrasound (transvaginal > transabdominal) Do not do a digital vaginal exam Delivery: cesarean section at 36 0/7-37 6/7 gestation
A 44-year-old man presents to the emergency department with a severe headache and sweating. Although his symptoms are already beginning to improve, he is concerned because similar episodes have occurred in the past several weeks during which he feels anxious, sweaty, and experiences flushing. The patient is a current smoker and drinks alcohol every night. His temperature is 99.5°F (37.5°C), blood pressure is 167/94 mm Hg, pulse is 90/min, and respiratory rate is 15/min. Which of the following is the best initial diagnostic test for this patient? Head CT scan without contrast Plasma fractionated metanephrine levels Renal artery ultrasound with Doppler flow Serum renin and aldosterone levels
Plasma fractionated metanephrine levels Pheochromocytoma Catecholamine-secreting tumor located in the adrenal glands Sx: paroxysmal headaches, diaphoresis, palpitations, tremors, and vision changes PE: hypertension, orthostasis Dx: ↑ 24h urinary catecholamines and metanephrines, or ↑ plasma metanephrine levels, adrenal CT or MRI Tx:alpha-blocker (phenoxybenzamine) prior to beta-blockade to prevent unopposed alpha-agonismsurgical resection Associated with MEN2 (medullary thyroid cancer, pheochromocytoma, +/- primary hyperparathyroidism)
A 62-year-old man presents to the emergency department with a chief concern of prolonged generalized weakness, fatigue, and headache over the past month. Vital signs are within normal limits. Physical exam reveals plethoric facies and splenomegaly. His blood work shows WBC 18,000/μL, hemoglobin 19.8 g/dL, hematocrit 61%, platelets 517,000/μL, and lactate dehydrogenase 370 U/L. Which of the following is the most likely diagnosis? Chronic myelogenous leukemia Essential thrombocythemia Myelofibrosis Polycythemia vera
Polycythemia vera Polycythemia Vera Patient presents with headache, dizziness, pruritus after showering PE will show hypertension, splenomegaly Labs will show increased RBC mass, overproduction of all cell lines, increased Hgb Most commonly caused by mutation of the Janus kinase 2 gene (JAK2) Treatment is phlebotomy, hydroxyurea, aspirin
A 27-year-old man sustains a knee injury when he falls in a rock-climbing gym and lands on his leg, hyperextending it at the knee. He complains of severe pain in the knee and is unable to bear weight. There is no obvious deformity. The knee is grossly unstable in all directions of stress testing. X-rays are negative for a fracture. Which of the following is the most serious immediate complication of his injury? Anterior cruciate ligament injury Compartment syndrome Deep venous thrombosis Popliteal artery injury
Popliteal artery injury Vascular injury is a catastrophic consequence of knee dislocations, and popliteal artery injury is reported in 5-68% of all dislocations, depending on injury mechanism. When popliteal artery injury is missed or repair is delayed for more than eight hours, the amputation rate is greater than 90%. Hard signs of vascular injury include the absence of a pulse, limb ischemia, pulsatile bleeding, and bruit or thrill, particularly in penetrating trauma. Paresthesias should also raise concern for vascular injury.
A 36-year-old G1P0010 woman presents with five weeks of nightmares after she had a stillbirth. She reports vivid nightmares of the delivery that occur multiple times a week, and she has started avoiding the kitchen where she last felt the baby's kicks. She feels she could have prevented it "if I had just eaten more organic vegetables." She has not been able to return to work due to fears that her coworkers might blame her for the miscarriage. Her husband reports she has been very isolated since the incident and rarely sleeps more than a few hours at night. Which of the following disorders is likely causing her symptoms? Acute stress disorder Adjustment disorder Post-traumatic stress disorder Primary insomnia disorder
Post-traumatic stress disorder This patient presents with five weeks of nightmares, guilt, and isolation after a traumatic incident, consistent with post-traumatic stress disorder (PTSD). Diagnosis is made clinically, using the Diagnostic and Statistical Manual of Mental Disorders criteria. Patients must meet certain criteria, such as personal exposure to a traumatic stressor (e.g., miscarriage), that lasts for at least one month and causes functional impairment (e.g., occupational) accompanied by intrusive symptoms (e.g., nightmares), avoidance (e.g., avoiding specific locations), negative thoughts (e.g., blame, isolation), and altered arousal (e.g., insomnia). Therapy is generally used to treat PTSD, and both cognitive behavioral therapy and eye movement desensitization and reprocessing are established treatments.
Where are the majority of anal fissures located? Anterior midline External sphincter muscle Lateral position Posterior midline
Posterior midline Anal Fissure Patient presents with rectal pain and bleeding that occurs with or shortly after defecation PE will show fissure located in the posterior midline Diagnosis is made by visual inspection Treatment is stool softeners, protective ointments, sitz baths If fissures are located laterally, search for pathologic etiologies
A 28-year-old man presents to the emergency department with lethargy and vomiting. He has a history of type I diabetes mellitus. Vital signs include heart rate 112 bpm and blood pressure 110/80 mm Hg. Laboratory analysis reveals sodium 135 mEq/L, potassium 4.0 mEq/L, chloride 100 mEq/L, bicarbonate 10 mEq/L, creatinine 1.4 mg/dL, glucose 558 mg/dL, pH 7.2. Normal saline and insulin have been ordered. Which of the following should be administered next? 5% dextrose with 0.45% saline Magnesium Potassium chloride Sodium bicarbonate
Potassium chloride The diagnosis of diabetic ketoacidosis can be established with the presence of hyperglycemia, ketosis and acidemia. The goals of treatment are aimed at insulin therapy, fluid resuscitation and electrolyte replacement. Potassium levels are often initially high or normal due to severe acidemia. The potassium levels often decrease significantly as the acidemia is corrected along with the administration of insulin. Potassium should be administered with intravenous fluids when the potassium is less than or equal to 5.0 mEq/mL.
A 29-year-old woman presents with palpitations. She has no known medical problems. Her family history is unknown because she was adopted at birth. Vital signs are remarkable for HR 180 bpm, RR 17, BP 100/60 mm Hg, and oxygen saturation of 99%. Her ECG is shown above. What is the treatment of choice for this patient? Adenosine Amiodarone Diltiazem Procainamide
Procainamide Wolff-Parkinson-White syndrome is characterized by an accessory pathway through which a reentrant tachycardia can occur, resulting in a cardiac dysrhythmia. The location of the accessory pathway is most commonly the left lateral aspect of the atrioventricular (AV) ring, but it can be found anywhere along the AV ring or septum. Clinical manifestations include palpitations, lightheadedness, dizziness, chest pain, syncope, and sudden cardiac death. The majority of patients are asymptomatic and the pattern is noticed on ECG incidentally. The characteristic findings on ECG include a shortened PR interval with a slurring of the QR or R segment of the QRS complex better known as the "delta wave." The two major forms of atrioventricular reentrant tachycardia associated with Wolff-Parkinson-White are orthodromic and antidromic, and the latter is typically distinguished by a widened QRS complex. This is an important rhythm to recognize as procainamide and cardioversion are the only two appropriate treatments. Any other medications or AV-node blockers can result in deterioration and possibly death.
A 52-year-old man presents to the emergency department with the complaint of palpitations. He has a history of type II diabetes mellitus. On physical examination his blood pressure is 180/104 mm Hg, heart rate is 168 bpm and temperature is 102.8°F. You note bilateral exophthalmos and fine tremors in the extremities. An ECG reveals an irregularly irregular rhythm. Propranolol was ordered and administered first. Which of the following should be administered next? Esmolol Hydrocortisone Iodine Propylthiouracil
Propylthiouracil Thyroid Storm History of thyrotoxicosis Sx: tachycardia, hyperpyrexia, agitation, anxiety PE: goiter, lid lag, hand tremor, and warm, moist skin Labs: low TSH and high free T4 or T3 Most commonly caused by an acute event (infection, trauma) Tx:Beta-blocker (propranolol)Thioamide (propylthiouracil or methimazole)Iodine solutionGlucocorticoids
A 29-year-old woman presents to the emergency department after she coughed up blood-tinged sputum this morning. She is also experiencing some shortness of breath and chest pain with deep inspirations. The patient is otherwise healthy and is a foreign exchange student who just arrived from India. She takes oral contraceptive pills, smokes two packs of cigarettes per day, and drinks alcohol on the weekends. She has had recent multiple sick contacts this past week. Her temperature is 99.9°F, blood pressure is 127/68 mm Hg, pulse is 126/min, and respirations are 24/min. Physical exam is notable only for pleuritic chest pain during the pulmonary exam. Which of the following is the most likely diagnosis? Bacterial pharyngitis Pulmonary embolism Small cell lung cancer Tuberculosis
Pulmonary embolism In the emergency department, a pulmonary embolism is one of the most concerning diagnoses associated with hemoptysis. A pulmonary embolism can present with shortness of breath, fatigue, pleuritic chest pain, signs of a deep vein thrombosis (such as a tender and inflamed calf), as well as a history of risk factors for clotting including stasis (such as a long flight, malignancy, or oral contraceptive use), and endothelial dysfunction (typically associated with smoking). The most common ECG finding is sinus tachycardia though right heart strain, a new right bundle branch block, or the classic S1Q3T3 could also be found. The diagnosis can be confirmed with a computed tomography angiography, and a heparin drip should be started. In unstable patients, thrombolytics or a surgical thrombectomy should be considered.
A 45-year-old woman presents to the ED complaining of shortness of breath. Her ECG is shown above. Which of the following is most closely associated with the findings seen in this ECG? Chronic cor pulmonale Emphysema Pulmonary embolism Right ventricular hypertrophy
Pulmonary embolism This ECG shows an S wave in lead I, a Q wave in lead III and a flipped T wave in lead III. These findings are suggestive of pulmonary embolism. The most common ECG changes seen in pulmonary embolism are tachycardia, nonspecific ST changes, and T wave changes.
A 43-year-old man presents after being struck by a motor vehicle on his right side and falling onto his left arm. His trauma evaluation is negative except for pain in the mid-right arm. A radiograph is seen above. Secondary injury to which of the following is the most common complication? Axillary artery Brachial artery Median nerve Radial nerve
Radial nerve The most common complication from a midshaft humerus fracture is a radial nerve injury. Clinically, patients may exhibit radial nerve palsy with wrist drop.
A 33-year-old man presents to the emergency department with joint pain. He describes lower back and wrist pain that has progressed to pain in his left knee in the setting of feeling generally fatigued and weak. He also notes that he had mild dysuria last week. Physical exam is notable for mild conjunctival injection, pain with passive movement of the patient's left knee, and erythema and edema of both wrists. Which of the following is the most likely diagnosis? Gout Reactive arthritis Rheumatoid arthritis Septic arthritis
Reactive arthritis Reactive Arthritis Risk factors: male sex, HLA-B27+, history of recent GI or Chlamydia infection Sx: acute asymmetric arthritis PE: conjunctivitis, arthritis, urethritis, keratoderma blennorrhagicum (lesions on palms and soles) Labs: may show positive stool cultures or + Chlamydia NAAT Tx: NSAIDs Mnemonic: can't see (conjunctivitis, uveitis), can't pee (urethritis), can't climb a tree (arthritis)
A 57-year-old man presents with low back pain radiating down his right lower extremity. On physical exam, he has decreased sensation in the L5 distribution. What is the most likely cause of his symptoms? Injury secondary to trauma Metastatic cancer Repetitive flexion Repetitive torsion
Repetitive flexion The patient is presenting with radiculopathy at the L5 spinal level. The most common cause of radiculopathy is disk herniation. Most disk herniation occurs at the L4-5 or L5-S1 spinal levels. These spinal levels are most susceptible to herniation from repetitive flexion of the torso. Disk herniation results secondary to degenerative changes that lead to osteophyte formation. Osteophytes are bony overgrowths that mostly cause no harm but can cause direct impingement on spinal nerve roots or lead to indirect pain from misalignment of the spine. On exam, patients with disk herniation exhibit a positive straight leg raise test. Most patients have an improvement in symptoms with supportive care.
Which of the following moves the midsystolic click of mitral valve prolapse later into the systolic phase? Prone position Squatting Standing Valsalva maneuver
Squatting Mitral Valve Prolapse Patient presents with palpitations, dyspnea, nonexertional CP, and fatigueMost cases are asymptomatic PE will show midsystolic click and late systolic murmur ↑ preload → ↓ murmur ↓ preload → ↑ murmur Marfan syndrome, Ehlers-Danlos Diagnosis is made on clinical exam and confirmed by echo
A young man presents with numerous painless floaters in his left eye and shadowing in the periphery of his vision. The symptoms started after he went skydiving this morning. On the drive to the hospital, the lines of the road appeared to be curving when viewed in his left eye, even though he knew them to be straight. He has myopia, corrected with glasses. His corrected visual acuity is 20/20 in the right eye and 20/30 in the left eye, with decreased peripheral vision on the left. Ocular ultrasound is shown in the image above. Which of the following is the most likely diagnosis? Posterior uveitis Posterior vitreous detachment Retinal detachment Vitreous hemorrhage
Retinal detachment This patient presenting with painless floaters and unilateral visual shadowing after skydiving shows a retinal detachment on ocular ultrasound. Diagnosis can be made with indirect ophthalmoscopy or ultrasound. Ultrasound will show a discrete hyperechoic retinal line projecting out from the posterior globe. Direct fundoscopy has low sensitivity for detecting retinal detachment and should not be used to rule it out. Patients usually present with painless vision changes, including flashes of light, floaters, or the classic "curtain-like" loss of vision. Retinal detachment is an ophthalmologic emergency and early intervention can prevent worsening of the detachment.
A 32-month-old girl presents to the ED with her mother after an episode of choking and "turning blue." Her mother notes that she was playing with her brother's toys when she choked and turned blue. After coughing, she seems to be back to normal. On physical exam, you note wheezing on the right side. Which of the following will aid in confirming the diagnosis? Abdominal radiograph Direct laryngoscopy Lateral soft tissue neck radiograph Right lateral decubitus chest radiograph
Right lateral decubitus chest radiograph
Encapsulated Bacteria (SHiNE SKiS)
S. pneumo Hib N. meningitidis E. Coli; Salmonella Klebsiella Strep (Group B)
A 50-year-old man presents with acute onset chest pain. His ECG shows an inferior ST segment elevation myocardial infarction (STEMI). Which of the following ECG findings are suggestive of left circumflex occlusion rather than right coronary artery occlusion? Biphasic T wave in leads V2 and V3 Bradycardia ST segment elevation in lead III that is greater than in lead II ST segment elevation in leads V5 and V6
ST segment elevation in leads V5 and V6 The ECG pattern in an inferior STEMI will have ST segment elevation in leads II, III, and aVF with reciprocal depressions in leads V1-3. An inferior STEMI can result secondary to occlusion of the right coronary artery (most common) or secondary to occlusion of the left circumflex artery. Because the right coronary artery and the left circumflex artery supply different areas of the inferior wall, subtle changes on the ECG can help determine which artery is occluded. In addition to elevations in leads II, III, and aVF, a left circumflex artery occlusion may show additional ST segment elevation in leads V5 and V6.
A 68-year-old man presents to the emergency department with dizziness. The patient has a past medical history significant for hyperlipidemia and hypertension. He states the symptoms started yesterday. They have been intermittent, lasting anywhere between 20 and 40 minutes at a time before resolving on their own. He describes his dizziness as a "room-spinning sensation." The patient is also complaining of an annoying ringing sound that is associated with the dizziness. The spinning sensation is made worse with head movement as well as positional changes. What would one expect to find on physical examination? Conductive hearing loss Middle ear effusion Sensorineural hearing loss Vertical nystagmus
Sensorineural hearing loss Ménière disease is a vestibular disorder that is a result of excessive endolymphatic fluid pressure. This is characterized as a peripheral vertigo (originating in the peripheral nervous system) as opposed to the more serious central vertigo (originating in the central nervous system). Clinical features include episodic vertigo, sensorineural hearing loss, and tinnitus. A horizontal or rotational nystagmus may be noted on exam. The vertigo has been reported to last between 20 minutes and up to 24 hours with associated nausea and vomiting being common. The diagnosis is based upon clinical features. Treatment consists of lifestyle adjustments, medical treatments, and interventional treatments. Lifestyle adjustments include avoidance of alcohol, caffeine, stress, high salt intake, monosodium glutamate, nicotine, and allergies as these are commonly known triggers. Medical management includes treating acute episodes with vestibular suppressants and antiemetics such as meclizine, dimenhydrinate, scopolamine, promethazine, prochlorperazine, or lorazepam. Diuretics and betahistine are used to prevent attacks by reducing the degree of endolymphatic hydrops.
A 72-year-old woman presents to the ED complaining of right knee swelling, stiffness, pain, and subjective fever. Her symptoms have developed and worsened over several days. She has no recent history of trauma to the area. Her past medical history includes type II diabetes and previous right knee replacement due to osteoarthritis. She takes metformin and sitagliptin. On exam, her temperature is 101.8°F (38.8°C); the knee is swollen with a large effusion, warmth, decreased active range of motion, and diffuse tenderness but no cellulitis. Findings on examination of synovial fluid include: cloudy, watery synovial fluid; white blood cell count 105 x 109/L with 90% neutrophils, no red blood cells, and no crystals. Gram stain is pending. What diagnosis fits these results most accurately? Gouty arthritis Intraarticular fracture Osteoarthritis Septic arthritis
Septic arthritis Patients presenting with acute monoarticular inflammation and swelling along with at least one predisposing condition have about a 10% chance of having septic arthritis. These predisposing conditions include a history of surgical replacement of the joint, skin infection, other joint surgery, rheumatoid arthritis, age over 80 years, diabetes mellitus, and renal disease. Synovial fluid associated with septic arthritis tends to be cloudy, yellow to white, and thin due to the presence of cells in the fluid and watery due to breakdown of glycosaminoglycans. White blood cell counts of more than 50 x 109/L are typical but counts of more than 100 x 109/L have a likelihood ratio of 28, versus 7.7 for white cell counts between 50 x 109/L and 100 x 109/L. Cell count differential is usually over 90% neutrophils.
A 32-year-old man presents with right-sided elbow swelling and pain. He works as a plumber and denies any specific injuries. The elbow pain and swelling developed rapidly and became worse over the last 2 days despite applying ice and using over-the-counter anti-inflammatory medications. His vital signs are HR 104 bpm, BP 110/60 mm Hg, RR 18/min, and T 100.2°F. On physical exam, his right elbow is swollen and tender to touch, with redness and fluctuance isolated to the posterior tip of the elbow. He refuses full flexion of the elbow on exam due to pain but allows passive pronation and supination. What is the most likely diagnosis? Lateral epicondylitis Medial epicondylitis Olecranon bursitis Septic bursitis
Septic bursitis Septic bursitis refers to a bacterial infection that seeds in one of the bursas. The most common mechanisms of infection involve percutaneous inoculation of skin organisms from trauma, prolonged pressure on the elbow such as from leaning on a hard surface, or from an overlying cellulitis. Staphylococcus aureus is responsible for the majority of cases, accounting for up to 80% of them. Patients present with fluctuance, erythema, warmth, and tenderness of the overlying area. More specific features include the rapid onset of pain and swelling, as well as fevers and limited joint mobility. Diagnosis is confirmed with synovial fluid analysis. A yellow or serosanguineous fluid usually indicates nonseptic bursitis, while a purulent cloudy fluid may indicate infection or crystal formation.
A 37-year-old woman with a history of hypertension, depression, and intravenous heroin use presents with sudden-onset abdominal pain and vomiting. Symptoms began one hour prior to arrival. Her vital signs are temperature 98.9°F, HR 126 beats/minute, RR 32 breaths/minute, BP 89/50 mm Hg, and oxygen saturation 98% on room air. She appears distressed and is writhing on the bed. Her abdomen is soft without ecchymosis or guarding. Which of the following studies is most useful to aid in the diagnosis of her condition? Kidney, ureter, bladder series Serum D-dimer Serum lactate Upright chest X-ray
Serum lactate Acute mesenteric ischemia is a small bowel vascular catastrophe that needs early diagnosis and management to prevent mortality. The four types of acute mesenteric ischemia include arterial embolism (most common), arterial thrombosis, venous thrombosis, and venous embolism. Arterial emboli most commonly occur from cardiac thrombi. Risk factors for the development of cardiac thrombi include myocardial ischemia or infarction, cardiomyopathies, ventricular aneurysms, endocarditis, and atrial dysrhythmias. Patients typically present with acute severe abdominal pain out of proportion to physical exam findings. Laboratory testing is of limited utility but may aid somewhat in diagnosis. An elevated serum lactate level, though nonspecific, is useful when the diagnosis is considered. A serum lactate is readily available, quickly results, and can be repeated to check serial levels to determine if ongoing ischemia is occurring. Mesenteric angiography is the gold standard imaging modality for diagnosis, but CT angiography has largely replaced this due to its better availability to emergency providers.
92-year-old man from a nursing home presents with constipation. On physical examination, his abdomen is tympanitic, distended, and tender to palpation. An X-ray demonstrates a single, large distended loop of bowel as shown above. What is the most appropriate next step? Computed tomography scan of the abdomen and pelvis Exploratory laparotomy Nasogastric tube decompression Sigmoidoscopy
Sigmoidoscopy Sigmoid Volvulus Sigmoid volvulus occurs when a redundant piece of sigmoid colon attached to the mesentery twists on itself. Risk factors: advanced age, bedbound, chronic constipation Gradual-onset diffuse abdominal pain, distention, nausea, vomiting. The classic triad of symptoms is abdominal pain, distention, and constipation. AXR: U-shaped dilated loop of colon Sigmoidoscopy to reduce volvulus Surgery to prevent recurrence
An 18-year-old woman presents with a diffuse erythematous rash that started yesterday. She was seen recently at a local urgent care three days ago for a sore throat, fatigue, and low-grade fever. She was told her strep screen was negative but because she was found to have "pus on my tonsils," she was started on amoxicillin pending the results of her throat culture. On examination, she has posterior pharyngeal erythema with tonsillar exudates and a diffuse maculopapular rash. Vital signs are unremarkable for her age other than a temperature of 38.1°C. Which of the following is most likely to be found on exam? Anterior cervical lymphadenopathy Positive Nikolsky sign Splenomegaly Wheezing
Splenomegaly Infectious Mononucleosis Patient presents with low-grade fever, headache, malaise, severe fatigue PE will show mildly tender lymphadenopathy involving the posterior cervical chain, hepatosplenomegaly Diagnosis is made by heterophile antibody test (mononuclear spot test) or a generalized maculopapular rash following administration of amoxicillin Most commonly caused by Epstein-Barr virus Treatment is self-limiting, refrain from contact sports for 4 weeks postinfection
A 37-year-old man presents to urgent care with pain and swelling in his fingertip. A football struck the patient's right fourth finger while he was playing a game of football with friends the day before. On examination, the distal interphalangeal joint of the right fourth finger has full passive range of motion, but the patient is not able to actively extend the joint. An X-ray confirms a small avulsion fracture of the dorsum of the proximal fourth distal phalanx. Which of the following is the most appropriate treatment for this fracture? Buddy taping and early range of motion Immobilization of the joint in 30° flexion for six to eight weeks Referral for surgical pinning Splinting of the joint in extension for six to eight weeks
Splinting of the joint in extension for six to eight weeks Mallet Finger Patient will be a basketball or volleyball player History of forced hyperflexion of the DIP Inability to extend the DIP Treatment is volar splinting of DIP in extension If untreated, swan neck deformity can result
A 22-year-old man presents to the ED after a witnessed syncopal event while playing basketball. A bedside transthoracic echocardiogram demonstrates septal hypertrophy. His ECG demonstrates left ventricular hypertrophy and deep Q waves in leads I, aVL, V5, and V6. The murmur associated with this condition is most likely to be louder with which of the following maneuvers? Hand grip Passive leg raise Squatting Standing up
Standing up Hypertrophic Cardiomyopathy Autosomal dominant; 1st degree relatives May present as sudden death in a young athlete Sx: dyspnea on exertion (most common), syncope, orthopnea, chest pain, palpitations PE: harsh crescendo-decrescendo systolic murmur that increases in intensity with Valsalva maneuver and decreases with squatting Dx: repolarization changes on ECG, echocardiography (LVH with septal hypertrophy) Management includes refraining from vigorous physical activity, ICD for high risk patients Rx options for symptomatic patients: beta-blockers or calcium channel blockers
A 4-year-old boy presents to the emergency department with a limp and fever. His mother states that he began limping 2 days ago, and it has gotten progressively worse. He will no longer walk. His fever started today. His mother denies any known injuries. On physical examination, his temperature is 39°C with a heart rate of 130 beats per minute. His right hip is warm to the touch with overlying erythema present. He has limited range of motion of the right hip due to pain. Laboratory analysis reveals a leukocytosis with elevated erythrocyte sedimentation rate and C-reactive protein. X-ray of his right hip reveals periosteal thickening. An ultrasound of the right hip reveals a hip effusion. Which of the following is the most common cause of the patient's presentation? Bartonella henselae Pseudomonas aeruginosa Staphylococcus aureus Streptococcus pyogenes
Staphylococcus aureus Osteomyelitis X-ray will show periosteal elevation or bony erosions Diagnosis is made by bone scan or MRI Most common organism General: S. aureus Sickle cell: Salmonella, S. aureus Cat or dog bites: Pasteurella multocida Infants < 3 mo: Group B streptococcus (Streptococcus agalactiae) Treatment is long-term Abx Adults: contiguous spread Children: hematogenous spread
A 24-year-old man presents after being hit by a car while walking across the street. During the secondary survey, it is noted that the right lower leg hangs at an awkward angle, concerning for a distal tibia-fibula fracture and dislocation near the ankle. He has abrasions, cuts, and punctate lacerations throughout the lower leg, including near the ankle. The patient is neurovascularly intact distal to the area of concern and has no other apparent injuries. Tetanus is updated, and the X-ray technicians and orthopedic surgery are on their way. What is the most appropriate next step regarding his right lower leg while waiting? Obtain dorsalis pedis and posterior tibial Doppler pulses Perform a hematoma block in preparation for reduction Perform a joint challenge to assess for a traumatic arthrotomy Reduce the ankle Start antibiotics
Start antibiotics This patient presents with an apparent tibia-fibula fracture. When a fracture is associated with cuts or puncture holes near the skin, it is considered an open fracture until proven otherwise. Open fractures are at high risk for infection, and studies have shown that the risk increases greatly when antibiotics are delayed more than 3 hours from the time of injury. For this patient, antibiotics should be started immediately. For antibiotic choice, one should start with a first-generation cephalosporin like cefazolin, but the coverage needs to be escalated if the wound is particularly large (> 10 cm), considered very contaminated, associated with fresh or saltwater, or deemed a "farming injury."
A 40-year-old man presents to the emergency department with sudden-onset back pain after lifting a box. The pain radiates down to the mid-thigh and is worse with bending and walking. Physical exam reveals left para-lumbar muscular tenderness without spasm. Which of the following exam maneuver(s) has the highest sensitivity and specificity for sciatica, respectively? Crossed straight leg alone Crossed straight leg raise, straight leg raise Straight leg raise alone Straight leg raise, crossed straight leg raise
Straight leg raise, crossed straight leg raise Sciatica is a lumbosacral radiculopathy that is caused by compression of the spinal nerve root(s). Symptoms typically develop suddenly during physical activity and consist of back pain radiating down the leg, though the pain may radiate bilaterally. Weakness and numbness in the affected limb may also be present. The straight leg raise (SLR) is a good screening test for sciatica with 92% sensitivity and 28% specificity. The patient's affected leg is extended and passively elevated. Pain elicited in a sciatic nerve distribution past the knee indicates a positive test. Pain elicited in the back alone is not considered positive. The crossed straight leg raise compliments the straight leg raise well in that it has a sensitivity of 28% and specificity of 90%. The crossed straight leg raise is performed by passively elevating the extended unaffected leg.
A 29-year-old man presents to the emergency department with acute altered mental status, headache, and neck stiffness. Intravenous corticosteroids, ceftriaxone, and vancomycin are given. A noncontrast head CT scan shows no evidence of mass effect. Subsequently a lumbar puncture is performed and a cerebrospinal fluid sample is obtained. It has a cloudy appearance, opening pressure of 27 cm H₂O, white blood cell count of 3,013 cells/µL with a predominance of polymorphonucleocytes, glucose level of 30 mg/dL, and protein level of 62 mg/dL. A point of care blood glucose is 110 mg/dL. Which of the following is the most likely causative organism? Cryptococcus neoformans Herpes simplex virus Mycobacterium tuberculosis Streptococcus pneumoniae
Streptococcus pneumoniae
Which of the following organisms is the most common bacterial cause of acute otitis media in children between 1 month and 10 years of age? Moraxella catarrhalis Staphylococcus aureus Streptococcus pneumoniae Streptococcus pyogenes
Streptococcus pneumoniae Streptococcus pneumoniae is present in about 40% of cases. Patient will be an infant or young child Ear pain, fever, URI symptoms PE will show TM erythema and decreased mobility of TM Most common bacteria isolated: H. influenza (nontypable) (previously S. pneumoniae but has decreased post-PCV13 vaccination) Treatment is amoxicillin; consider amoxicillin-clavulanate in otitis-conjunctivitis syndrome or adults due to drug resistance Consider period of observation if ≥ 2 years and immunocompetent with mild symptoms
A 23-year-old man presents to the emergency department with a fever and mental status changes. According to his sibling, he was involved in a motor vehicle collision 4 weeks ago and underwent splenectomy because of uncontrolled bleeding in that area. He improved during his hospital stay and did not have other complications from the collision or injury. He has been doing well at home for over 1 week until today, when he developed a fever. Vital signs show HR 142 bpm, BP 89/51 mm Hg, RR 24/min, and T 39.4°C. The patient will open his eyes and attempt to interact, but he does not answer all questions appropriately. He does not have other focal examination findings or significant abdominal tenderness. You are concerned about underlying sepsis and initiate the appropriate workup and empiric antibiotic treatment. What organism is the most likely cause of this patient's infection? Escherichia coli Pseudomonas aeruginosa Staphylococcus aureus Streptococcus pneumoniae
Streptococcus pneumoniae Without a spleen, patients are at risk for severe sepsis from encapsulated organisms. Salvaging some splenic tissue can help the patient maintain immune competence to fight these infections. It is important to recognize that this particular patient is at the highest risk for sepsis from encapsulated organisms. The organisms that most commonly lead to sepsis in asplenic patients include Streptococcus pneumoniae, Neisseria meningitidis, and Haemophilus influenzae. Empiric antibiotic treatment should be initiated with ceftriaxone and vancomycin.
A 33-year-old woman with a history of peptic ulcer disease presents to the emergency department with a unilateral, pulsatile headache that is 8 out of 10 in severity. She has had similar headaches previously. This time, she was unable to control her symptoms at home with acetaminophen. Her neurological exam is within normal limits. Which of the following is the best treatment for this patient's headache? Ibuprofen Morphine Oxygen Sumatriptan
Sumatriptan Migraine Headache Risk factors: female sex, family history Gradual onset, unilateral > bilateral, throbbing, pulsating headache Without aura: most common, nausea or vomiting, photophobia, phonophobia Aura: scotoma, flashing lights, sounds Diagnosis is made clinically Treatment Abortive Rx: triptans, DHE, antiemetics, NSAIDs Prophylaxis: TCAs, beta-blockers, anticonvulsants (valproic acid, topiramate), CCBs Triptans, DHE: avoid in uncontrolled HTN or CV disease
A 3-year-old boy presents to the emergency department due to refusal to move his left arm. His symptoms started after his parent grabbed him by the arm to stop him from running into the street. On examination, he is holding the left elbow in flexion with pronation of the forearm. He resists any attempts to move the arm. There is no noted swelling or obvious deformity. What method can be used to reduce a radial head subluxation? Pronation of the forearm with extension of the elbow Pronation of the forearm with flexion of the elbow Supination of the forearm with extension of the elbow Supination of the forearm with flexion of the elbow
Supination of the forearm with flexion of the elbow Pulled Elbow (Radial Head Subluxation, Nursemaid Elbow) Patient will be a child 1-4 years old History of being pulled up by the wrist PE will show the affected arm held close to the body in a flexed and pronated position Most commonly caused by longitudinal traction on a pronated forearm while the elbow is extended Treatment: Hyperpronation method: apply pressure to the radial head and hyperpronating the forearm Supination-flexion method: supinate and fully flex the elbow while applying pressure to the radial head and pulling with gentle traction
A 22-year-old man with no significant past medical history presents to the emergency department with 1 day of atraumatic left-sided chest pain. He is on his college's track and field team. His vital signs are HR 109 bpm, BP 120/82 mm Hg, T 98.6°F, RR 22 breaths/min, and oxygen saturation 93% on room air. A chest radiograph is shown above. Which of the following is the most appropriate management of this patient's condition? Needle thoracostomy Pigtail catheter aspiration Supplemental oxygen, observation for 4 hours, repeat chest radiograph Tube thoracostomy
Supplemental oxygen, observation for 4 hours, repeat chest radiograph This patient is presenting with a spontaneous left apical pneumothorax. The pathophysiology of a pneumothorax is an accumulation of free air in the potential space between the visceral and parietal lung pleura. Pneumothoraces may be either primary or secondary, with primary pneumothoraces occurring in patients without underlying lung disease. Risk factors for the development of primary spontaneous pneumothorax include tobacco smoking, mitral valve prolapse, Marfan syndrome, and male sex. The incidence of primary spontaneous pneumothorax is almost three times higher in male than female patients. Patients present with sudden-onset ipsilateral pleuritic chest pain and dyspnea. Sinus tachycardia is the most common vital sign abnormality. On exam, patients may have decreased breath sounds of the affected lung. Diagnosis varies depending on the severity and hemodynamic stability of the patient. In hemodynamically stable patients, upright chest radiography and ultrasound are commonly used. The absence of lung markings is a diagnostic radiographic finding for pneumothorax. In patients in whom only a supine chest radiograph can be obtained, a profound lateral costophrenic angle, known as deep sulcus sign, may be seen on the affected side. A sonographic finding suggestive of pneumothorax in the right clinical context is the absence of lung sliding. Supplemental oxygen greater than 28% FiO2 (increases pleural air resorption), observation for 4 hours, and repeat chest radiograph is appropriate in patients with small, stable pneumothoraces.
A 75-year-old man presents to the emergency department with dyspnea. He has a history of COPD, hypertension, hyperlipidemia, and coronary artery disease. He has recently been fighting an upper respiratory infection and found he needed to use his albuterol inhaler more frequently. Today he started experiencing worsening shortness of breath and decided to come in for evaluation. Vital signs are remarkable for HR 120 bpm, BP 130/80 mm Hg, RR 28/minute, and oxygen saturation 87%. A rhythm strip is shown above. What is the best next step in management? Administer IV diltiazem Electrical cardioversion Initiate IV procainamide Supportive care
Supportive care Multifocal Atrial Tachycardia Associated with older patients and those with COPD Rate will be 100-200 beats/min PR interval will differ Notable feature: at least three different P wave forms Treatment is to treat the underlying cause, calcium channel blockers
An 8-year-old boy presents with his mother to the ED via EMS following six days of rhinorrhea, cough, and nasal congestion. He has been taking an over-the-counter medication for fever. Two days ago, he started having multiple bouts of vomiting, despite restricting his oral intake, and on the day of arrival, he became disoriented and confused. On exam, he has T 37.16°C, BP 110/70 mm Hg, HR 155 bpm, RR 28/min, and glucose 78 mg/dL. He is altered and stuporous, has a palpable liver edge but no jaundice or icterus, and has no meningeal signs or focal neurologic deficits. His labs reveal elevated levels of AST, ALT, lactate, and ammonia. Which of the following is the most appropriate treatment for this condition? High-dose barbiturates Hyperosmolar agents Hyperventilation Supportive care
Supportive care This patient has Reye syndrome, characterized by encephalopathy and fatty degeneration of the liver. It has been associated with aspirin exposure in the setting of a viral illness. Its major cause of morbidity and mortality is cerebral edema leading to increased intracranial pressure. There is no specific cure, and management is supportive. Reye Syndrome Patient will be a child History of viral illness treated with aspirin Encephalopathy, delirium, seizures, vomiting PE will show fatty liver Labs will show hypoglycemia, elevated liver enzymes, hyperammonemia, metabolic acidosis Treatment is supportive
A 32-year-old man presents with arm pain. He was practicing parkour, jumped off a ledge and caught onto an overhead bar but then slipped and fell on his wrist. His neurologic exam reveals weak finger abduction, weak thumb opposition, and decreased sensation to the ulnar aspect of the forearm. Which nerve is most likely damaged? C7 nerve root Median nerve T1 nerve root Ulnar nerve
T1 nerve root The brachial plexus consists of nerve roots C5 through T1, which merge and divide to form the median, ulnar, and radial nerves. A T1 nerve root injury is most likely in hyperextension, as it is the most inferior and subject to the greatest degree of pulling. The T1 nerve root's motor function includes finger abduction, finger adduction, thumb opposition, and wrist flexion.
A 62-year-old man presents to the emergency department with right-sided flank pain radiating into the right abdomen for one day that seems characteristic of a ureteral stone. He has a history of kidney stones and hypertension. After completing the workup, you diagnose the patient with a 6 mm distal ureteral stone with mild hydronephrosis on the right side. There are no other significant findings on laboratory evaluation, and he indicates he will see his urologist in a few days. What medication can help facilitate passage of this ureteral stone? Dexamethasone Tadalafil Tamsulosin Verapamil
Tamsulosin Kidney and ureteral stones are a common problem in the emergency department. Classic presentation of renal colic is with flank pain, abdominal pain, hematuria, and urinary urgency. Diagnosis of ureteral stone is confirmed with imaging, preferably with noncontrast computed tomography of the abdomen and pelvis. Almost all stones that are less than 5 mm will pass on their own within four weeks. Even 60% of stones 5 to 7 mm and 40% of stones greater than 7 mm will pass on their own within four weeks. Proximal ureteral stones are less likely to pass than distal ureteral stones. Emergency department treatment includes controlling pain and vomiting. The most widely studied and successful medication that is associated with higher rates of stone passage is tamsulosin, an alpha-blocker. It is particularly recommended to help facilitate stone passage in patients with larger ureteral stones. The vast majority of patients with ureteral stones, even larger stones, can be discharged from the emergency department with outpatient urology follow-up. Admission criteria include urosepsis, acute kidney injury, and uncontrolled pain or vomiting.
A 17-year-old boy presents after injuring his left ankle while playing soccer. Which of the following indicates the need for imaging of the ankle? Inability to walk prior to arrival, but ambulatory during evaluation Tenderness over the anterior edge of the distal medial malleolus Tenderness over the base of the fifth metatarsal Tenderness over the posterior edge of the distal lateral malleolus
Tenderness over the posterior edge of the distal lateral malleolus Ottawa Foot and Ankle Rules X-rays required if: Patient cannot bear weight for four steps Distal 6 cm of tibia or fibula is tender Medial or lateral malleolus is tender Fifth metatarsal is tender Navicular bone is tender
A 78-year-old woman is brought to the ED by her adult child with concern for altered mentation and somnolence over the past 3 days. The patient has a history of hypertension and elevated cholesterol, both well-controlled with medication. Her vitals are significant for a blood pressure of 148/72 mm Hg, heart rate of 82 bpm, oxygen saturation of 99% on room air, and a temperature of 100.6°F (38.1°C). Physical exam reveals a well-appearing older female patient who is sleeping but easily arousable. Laboratory values show a WBC of 8.2/µL with 2% bands, hemoglobin of 13.6 g/dL, platelets of 120,000/µL, sodium of 136 mEq/L, potassium of 3.4 mEq/L, blood urea nitrogen of 12 mg/dL, and creatinine of 0.8 mg/dL. Urinalysis is positive for leukocyte esterase, nitrites, and 15 WBC/hpf. Which of the following statements best describes the diagnosis and next step of management? The patient has cystitis; start oral levofloxacin and discharge The patient has cystitis; start oral trimethoprim-sulfamethoxazole and discharge The patient has pyelonephritis; start parenteral ceftriaxone and admit The patient has pyelonephritis; start parenteral trimethoprim-sulfamethoxazole and admit
The patient has pyelonephritis; start parenteral ceftriaxone and admit Acute Pyelonephritis Sx: fever, dysuria, and flank pain PE: CVA tenderness Labs: UA + leukocyte esterase, nitrites, microscopy +WBCs, Gram stain, urine culture and susceptibility testing Most commonly caused by Escherichia coli Treatment depends on infection severity and community/host risk factors for resistant pathogens, options include fluoroquinolones, 3rd/4th gen cephalosporins, TMP-SMX. Critical illness or risk for multidrug resistant organisms: consider coverage for MRSA, VRE
A 72-year-old man with a history of hypertension, emphysema, and depression presents to the emergency department after a house fire. He is awake and alert but screaming in pain due to multiple burns across his body. His exam reveals a 100 kg man with superficial burns of his entire chest, superficial partial thickness burns of his entire left arm and half of his right arm, and deep partial thickness burns of the anterior left lower extremity. Along with admission to a burn unit and wound care, using the Parkland formula, what fluid resuscitation should be started in the emergency department? 1,800 mL of lactated Ringer over the first 8 hours 4,500 mL of lactated Ringer over the first 8 hours 6,300 mL of lactated Ringer over the first 16 hours 6,300 mL of lactated Ringer over the first 8 hours
Thermal Burn Fire: obtain CO levels Industrial fire: suspect CN toxicity Superficial: similar to sunburn Superficial partial: red, painful, blisters Deep partial: white, leathery, painless Full-thickness: charred, insensate ABCs and consider intubating early Rule of 9s Parkland formula: 4 mL × weight (kg) × % total BSA burned 50% given in first 8 hours, remainder over 16 hours Target urine output: 0.5-1 mL/kg/h in adults, 1-2 mL/kg/h in children Consider escharotomy for circumferential and full-thickness burns
A 45-year-old man presents to the ED due to trouble walking and disorientation. The patient was brought in by his wife, who reports he was walking with a wide-based, slow gait for the past week and became confused today. She reports he has no significant medical conditions and no known allergies. On a physical exam, the patient is confused and disoriented. He is not able to walk without help, and you note horizontal nystagmus. The patient's wife is concerned and confides in you that her husband has been drinking more alcohol since he lost his job 6 months ago. CT scan of head shows symmetric low-density abnormalities in the midbrain regions. Which of the following is the most appropriate therapy for the suspected diagnosis? Ceftriaxone Diazepam Haloperidol Thiamine
Thiamine Wernicke encephalopathy is an acute neuropsychiatric emergency. It is characterized by oculomotor dysfunction, mental status change, and gait disturbances and is due to a depletion of intracellular thiamine. This depletion occurs in the setting of poor nutrition from poor dietary intake, malabsorption, increased loss of thiamine by dialysis, or increased metabolic requirement during systemic illness. Most cases occur due to chronic heavy alcohol use, but it can also be associated with anorexia nervosa, hyperemesis during pregnancy, prolonged poor nutrition, systemic malignancy, IV feeding or dialysis without supplementation, and acquired immunodeficiency syndrome. Wernicke encephalopathy is a primarily clinical diagnosis that should be a differential diagnosis in all patients presenting with acute delirium and ataxia. The Caine criteria are more sensitive than the classic triad of symptoms. Patients are diagnosed with Wernicke encephalopathy if they have two or more of the following: dietary deficiency, oculomotor abnormalities, cerebellar dysfunction, altered mental status, or mild memory impairment. Prompt treatment with IV thiamine. The recommended dose is thiamine 500 mg IV over 30 minutes three times daily for 2 days and 250 mg IM or IV once daily for 5 more days. It should be given in combination with other B vitamins and should be given before glucose, as the administration of glucose can worsen Wernicke encephalopathy.
A 9-year-old girl previously in a normal state of health presents to the emergency department due to epistaxis for the past 4 hours. Her parents have applied pressure to her nose and have sprayed oxymetazoline intranasally with intermittent improvement. However, she continues to rebleed. Physical exam reveals a well-appearing girl with oozing epistaxis from the left nare. She also has petechiae scattered throughout her back and on both anterior thighs. Her parents have no medical problems, and the patient does not have a history of malignancy or easy bleeding or bruising. Which of the following lab findings is most consistent with the diagnosis? Elevated serum D-dimer Elevated serum uric acid Reduced factor VIII activity level Thrombocytopenia
Thrombocytopenia Primary Immune Thrombocytopenia - Pediatric 2-6 years old Antiplatelet autoantibodies H/o recent viral infection Non-blanching petechiae/purpura, gingival bleeding Labs: platelets < 100,000/µL, normal WBC, normal hematocrit Tx: activity restriction, observation, glucocorticoids and IVIG or IV anti-D if severe
A 51-year-old woman with no significant medical history presents to the ED with fever and shortness of breath. Over the last 3 months, she has noted weight loss, decreased appetite, diarrhea, palpitations, dyspnea on exertion, and generalized anxiety. Her only travel history was a mission trip to the Dominican Republic 6 months ago. She is fully immunized. Her BP is 156/89 mm Hg, HR is 131 bpm, RR is 28/minute, T is 102.3°F, and oxygen saturation is 94% on room air. She appears thin and diaphoretic and has jugular venous distension to the mandible, bibasilar crackles, warm extremities, and bounding peripheral pulses. Her sclera can be seen above and below her iris bilaterally. Her ECG shows sinus tachycardia, and her chest X-ray shows pulmonary edema. Which of the following is the most likely diagnosis? Bacterial endocarditis Chagas disease Cor pulmonale Thyroid storm
Thyroid storm Thyroid storm can imitate many other conditions that present more commonly in the emergency department. These include hyperthermia, hypertension, tachycardia (often out of proportion to temperature), anxiety, tremors, diaphoresis, hair thinning, diarrhea, exophthalmos, and an anterior neck mass. Patients can also develop high-output heart failure and present with dyspnea on exertion, elevated jugular venous pressure with jugular venous distension, and pulmonary edema. The hyperadrenergic effects of thyroid storm (i.e., tachycardia, hypertension) should be treated with a beta-blocker (e.g., propranolol). Antithyroid medications like propylthiouracil or methimazole should be administered to block new thyroid hormone synthesis. Then, iodine should be administered. Glucocorticoids are also given to block peripheral conversion of thyroxine (T4) to triiodothyronine (T3), which is the active form of thyroid hormone.
A 32-year-old man presents to the emergency department with diarrhea for 4 days. It began after he returned from a multi-day hunting and camping trip. His stools are malodorous and greasy, and he notices a sulfuric smell when he belches. He also reports abdominal discomfort. You diagnose giardiasis. Which of the following is the first-line treatment for the patient's condition? Albendazole 400 mg orally daily for 5 days Ciprofloxacin 500 mg orally twice daily for 3 days Rifaximin 200 mg orally three times daily for 5 days Tinidazole 2 g orally as a single dose
Tinidazole 2 g orally as a single dose Tinidazole 2 grams orally as a single dose is a primary treatment for Giardia infection. Patients should be counseled to avoid lactose-containing foods. Treatment is not usually recommended in asymptomatic individuals, though is advised if the patient has close contact with a pregnant woman, immunocompromised individual, or a child in a day care setting who may transmit the infection. Giardia lamblia is a protozoan parasite that commonly causes diarrheal illnesses among international and backcountry travelers as well as in day care centers. Immunocompromised individuals, international adoptees, and young children are at high risk of infection. Transmission occurs via three main routes: waterborne, foodborne, or fecal-oral transmission. Symptoms of acute giardiasis include: diarrhea, malaise, foul-smelling and fatty stools, abdominal bloating and cramps, flatulence, nausea, weight loss, vomiting, fever, and occasionally constipation and urticaria. Symptoms of acute giardiasis typically resolve within 5 to 7 days of treatment.
A 25-year-old man presents to the emergency department with five days of watery, foul-smelling diarrhea. He recently returned from a trip where he hiked and camped the Inca Trail to Machu Picchu. He would not always purify his water since he was high in the mountains and assumed it was clean. He reports abdominal bloating and cramping with flatulence and smelly diarrhea. What is the best treatment for this patient? Azithromycin 1 g orally for one dose Ciprofloxacin 500 mg orally twice a day for three days Tinidazole 2 g orally for one dose Trimethoprim-sulfamethoxazole DS one tab orally twice a day for ten days
Tinidazole 2 g orally for one dose The patient in this question, who just returned from an international camping trip where he did not adequately purify his water, most likely is returning with Giardia lamblia infection. Giardia is a flagellated protozoan parasitic infection that is waterborne and foodborne and is frequently found in international travelers. Symptoms generally include diarrhea, abdominal bloating and cramping, steatorrhea, flatulence and foul-smelling, frothy stool.
A 56-year-old man presents with penile and prepuce redness, swelling, and discharge as pictured above. The symptoms have been ongoing for several days. He reports worsening redness and pain and notes a mild odor that has not been present previously. What is the most appropriate treatment for this patient? Oral fluconazole Topical clotrimazole cream Topical metronidazole cream Topical mupirocin cream
Topical clotrimazole cream The patient in this question is presenting with balanitis and balanoposthitis, an inflammation of the glans penis and prepuce. Balanitis has a wide range of causes but is most related to inadequate hygiene in uncircumcised men. When the foreskin is not routinely cleaned, buildup of sweat, debris, exfoliated skin, and bacteria or fungus can occur leading to inflammation. Candidal infection is the most common cause and in this situation, empiric treatment with topical clotrimazole cream is warranted.
A 50-year-old man with a history of schizophrenia, seizure disorder, chronic back pain, and hypertension presents to the emergency room with altered mental status. His temperature is 40.0°C (104°F), blood pressure is 174/89 mm Hg, pulse is 112/min, respirations are 22/min, and oxygen saturation is 96% on room air. On physical exam, he is agitated, diaphoretic, hypersalivating, and has mydriasis. Lower extremity hyperreflexia, rigidity, and clonus are noted. Review of his chart indicates that he was started on a cough medication 1 day ago. Which of the patient's current medications is the most likely cause of this patient's suspected diagnosis? Haloperidol Lisinopril Phenytoin Tramadol
Tramadol Serotonin syndrome is a potentially life-threatening adverse reaction to serotonergic drugs and is one of many causes of drug fevers. The most common class of drugs associated with serotonin syndrome is antidepressants such as selective serotonin reuptake inhibitors (SSRI), monoamine oxidase inhibitors (MAOI), serotonin and norepinephrine reuptake inhibitors (SNRI), tricyclic antidepressants (TCA), and atypical antidepressants such as trazodone and bupropion. Several other drugs are associated with serotonin syndrome such as tramadol, lithium, linezolid, triptans, meperidine, and dextromethorphan. Symptoms typically begin 2-24 hours within dosage of a serotonergic drug, or the addition of a new serotonergic drug. This patient was taking tramadol for chronic back pain and was recently prescribed a cough medication, most likely dextromethorphan, which triggered serotonin syndrome. Patients with serotonin syndrome present with a triad of cognitive, autonomic, and neuromuscular related symptoms. Cognitive symptoms may manifest as altered mental status, agitation, or pressure speech. Autonomic effects cause vital sign abnormalities such as hyperthermia, hypertension, and tachycardia and symptoms such as diarrhea, mydriasis, and diaphoresis. Neuromuscular symptoms include tremor, hyperreflexia, rigidity, and myoclonus. Treatment is immediate cessation of offending medications as well as supportive management. Benzodiazepines and cyproheptadine have activity as serotonin receptor antagonists and are often used for symptomatic management, although they have not been consistently shown to have a significant effect
A 27-year-old man with a past medical history of sickle cell disease presents to the emergency department with shortness of breath and chest discomfort with exertion. The patient is requesting hydromorphone. His temperature is 100°F (37.8°C), blood pressure is 102/68 mm Hg, pulse is 120/min, respiratory rate is 18/min, and oxygen saturation is 99% on room air. Physical exam is notable for an uncomfortable-appearing, pale young man who vomits during his exam. His lungs are clear to auscultation. Laboratory values are notable for a hemoglobin of 5.5 mg/dL and a reticulocyte count of 0.5%. An initial ECG is notable for sinus tachycardia and his chest X-ray is shown above. Which of the following is the best next step in management? Administer hydromorphone Discharge the patient with addiction medicine follow-up Perform an exchange transfusion Transfuse packed red blood cells
Transfuse packed red blood cells This patient is presenting with a history of sickle cell disease and exertional dyspnea in the setting of severe anemia without a reticulocytosis, suggesting a diagnosis of an aplastic crisis. Sickle cell disease typically presents with recurrent episodes of pain (termed pain crises). In a patient who presents with a pain crisis, the best initial step in management is to administer oxygen, IV fluids, and analgesics in addition to a workup for an underlying infectious etiology and broad-spectrum antibiotics if the patient has a fever (as sickle cell patients are particularly susceptible to infection secondary to splenic infarction). If a patient with sickle cell disease presents with severe anemia with a decreased reticulocyte count (keep in mind that the reticulocyte count should be elevated in anemia), then the most likely diagnosis is an aplastic crisis, most commonly secondary to infection (e.g. parvovirus B19 infection). These patients should immediately be transfused with packed red blood cells, in particular, if they are experiencing signs of decreased end-organ perfusion such as demand ischemia or exertional dyspnea.
A 32-year-old man with a history of injection drug use presents to the ED with fever and chills. On cardiac auscultation a new heart murmur is heard and the provider suspects infective endocarditis. Which of the following heart valves is most likely to be involved? Aortic valve Mitral valve Pulmonary valve Tricuspid valve
Tricuspid valve Bacterial Endocarditis Risk factors: injection drug use, valvular heart disease Sx: fever, rash, cough, and myalgias PE: fever, Roth spots, Osler nodes, murmur, Janeway lesions, anemia, nailbed hemorrhages, emboli (FROM JANE) Diagnosis is made by echocardiography and Duke criteria Most commonly caused by:IVDA: Staphylococcus aureus, tricuspidNative valve: Staphylococcus aureus, viridans streptococci (most common in previously diseased), mitral Tx: antibiotics GI malignancy: Streptococcus bovis Dental prophylaxis in some cases
A 34-year-old woman with a history of sickle cell anemia and depression presents to the emergency department with an episode of dysarthria. According to her husband, approximately 30 minutes ago, she was slurring her speech and having difficulty getting words out. She seemed confused at the time and was drooling from the right side of her mouth. He attempted to shake her and speak to her, but she would not stand up or respond clearly. While he was finding his phone to call 911, her symptoms resolved. She was confused as to what had happened but was then speaking clearly. She continues to be at her baseline now but complains of a mild headache. Vital signs indicate a BP of 168/92 mm Hg, HR of 88 bpm, RR 16 breaths/minute, and T of 37.3°C. Her exam is unremarkable. Laboratory results show anemia with a hemoglobin of 7.9 g/dL but otherwise are unremarkable. CT imaging of the brain is normal. What is the most likely diagnosis in this patient? Bell palsy Complicated migraine headache Seizure Transient ischemic attack
Transient Ischemic Attack Transient Ischemic Attack Transient episode of neurological dysfunction without acute infarction 10% of TIA patients will have a stroke within 90 days Low-risk TIA (ABCD2 score < 4) or moderate to major ischemic stroke (National Institutes of Health Stroke Scale (NIHSS) > 3)Treatment with aspirin alone High-risk TIA (ABCD2 score ≥ 4) or minor ischemic stroke (NIHSS score ≤ 3)Begin with dual antiplatelet therapy (DAPT) for 21 days using aspirin plus clopidogrel ABCD2 score: predicts likelihood of subsequent stroke within 2 days
A 47-year-old man presents to the clinic with foot pain. He says the pain is worse in the morning with his first steps and gradually improves with activity. Physical examination reveals tenderness of the medial calcaneal tubercle and several areas of point tenderness upon dorsiflexing the patient's toes. Which of the following is the most appropriate next step in management? Corticosteroid injections Opioid therapy Topical lidocaine cream Trial of NSAIDs
Trial of NSAIDs Plantar Fasciitis Sx: heel and foot pain when first stepping out of bed or after period of inactivity, improves with walking or stretching the calf PE: tenderness over the sole of the foot near the calcaneus Tx: stretching plantar fascia and calf, heel inserts
A 28-year-old woman presents with a two-week history of dyspnea, nonproductive cough, fever, and fatigue. She is HIV-positive and noncompliant with her medications. She has no known drug allergies. Laboratory results reveal an arterial blood gas with a PaO2 of 72 mm Hg and an elevated LDH level. Her chest X-ray is significant for bilateral diffuse interstitial infiltrates. Which one of the following treatments should be initiated? Clindamycin and steroids Pentamidine Trimethoprim-sulfamethoxazole Trimethoprim-sulfamethoxazole and steroids
Trimethoprim-sulfamethoxazole Trimethoprim-sulfamethoxazole is the preferred treatment for Pneumocystis jirovecii pneumonia, formerly known as Pneumocystis carinii pneumonia, or PCP. Pneumocystis pneumonia is the most common opportunistic infection seen in HIV patients. Patients present with dyspnea, nonproductive cough, and fever, which typically manifest over a two- to three-week period. Chest X-ray most commonly demonstrates bilateral diffuse interstitial or alveolar infiltrates. LDH levels are frequently elevated, and a higher degree of elevation portends a worse prognosis. The diagnosis is confirmed by induced sputum analysis. Trimethoprim-sulfamethoxazole and steroids (D) is an incorrect choice because steroids are not indicated with a PaO2 of > 70 mm Hg
A 45-year-old man with HIV-AIDS and a CD4 count of 150 cells/mm3 presents with several days of fever and dry cough. His oxygen saturation is 85% on room air. Lung auscultation is notable for decreased breath sounds bilaterally and crackles at the bases. A chest X-ray reveals bilateral interstitial infiltrates. Which of the following medication regimens is most appropriate? Amphotericin B Azithromycin and ceftriaxone Oseltamivir Trimethoprim/sulfamethoxazole and prednisone
Trimethoprim/sulfamethoxazole and prednisone This patient has clinical and radiographic evidence for pneumonia. His low CD4 count puts him at risk of opportunistic infections, such as Pneumocystis jiroveci pneumonia (PJP). Signs and symptoms of PJP pneumonia include fatigue, fever, cough, and hypoxia. Chest radiographs typically show diffuse interstitial infiltrates. Serum lactate dehydrogenase levels are often elevated. Initial therapy for PJP includes trimethoprim/sulfamethoxazole. In severe cases, corticosteroids are also given. The indications for corticosteroid use include an arterial oxygen partial pressure of < 70 mm Hg (equivalent to an oxygen saturation of <~93%) and an alveolar-arterial gradient > 35 mm Hg.
Which of the following lumbar puncture results would you expect to find with the head CT seen above? Tube 1: WBC 5/microL, RBC 4,000/microL; tube 2: protein 70 mg/dL, glucose 60 mg/dL; tube 3: RBC 50/microL Tube 1: WBC 7/microL, RBC 4,000/microL; tube 2: protein 75 mg/dL, glucose 60 mg/dL; tube 3: RBC 3,200/microL Tube 1: WBC 80/microL, RBC 3/microL; tube 2: protein 60 mg/dL, glucose 60 mg/dL; tube 3: RBC 0 /microL Tube 1: WBC 900/microL, RBC 10/microL; tube 2: protein 200 mg/dL, glucose 15 mg/dL; tube 3: RBC 1/microL
Tube 1: WBC 7/microL, RBC 4,000/microL; tube 2: protein 75 mg/dL, glucose 60 mg/dL; tube 3: RBC 3,200/microL The head CT scan shows a subarachnoid hemorrhage. Most subarachnoid hemorrhages are caused by a ruptured berry aneurysm in the circle of Willis. if the CT scan is negative, a lumbar puncture (LP) should be performed with comparison of red blood cell count in the cerebrospinal fluid (CSF) across tubes to differentiate subarachnoid hemorrhage from a traumatic lumbar puncture. If the first and last tubes both have a significant number of RBCs, a subarachnoid hemorrhage is most likely. A very large difference in RBC numbers (first tube >> last tube) indicates a traumatic tap. ubarachnoid hemorrhage can also be detected by identification of xanthochromia, a yellow appearance of the CSF due to blood breakdown and release of bilirubin.
A 55-year-old woman with a history of diabetes, hypertension, hyperlipidemia, and tobacco use presents with a chief complaint of chest pain. She states she has been experiencing chest pain for the past couple of weeks, mostly with exertion. This has progressively worsened to the point where she now can barely walk to her mailbox without pain. Typically, her symptoms would improve with rest or sitting. Today, she developed pain while walking, but this time it persisted. She describes the pain as squeezing, with radiation into her neck. Vital signs and physical examination are both unremarkable. Her ECG is shown above. All lab results, including two sets of troponins, are unremarkable. What is the most likely diagnosis? Non-ST elevation myocardial infarction ST elevation myocardial infarction Unstable angina Wellens syndrome
Unstable angina ACS includes unstable angina, non-ST elevation myocardial infarction (NSTEMI), and ST elevation myocardial infarction (STEMI). Unstable angina is a clinical diagnosis defined by chest pain, or an equivalent, from inadequate myocardial perfusion that is new or occurring with greater frequency, with less activity, or at rest. Unstable angina does not have pathologic ST segment elevation on ECG or cardiac biomarker elevation. Classic symptoms of ACS include pain that is described as crushing, squeezing, tightness, or pressure. This pain is typically brought on or made worse with exertion and relieved with rest. Radiation of symptoms into arms, neck, or jaw; diaphoresis; dyspnea; and nausea and vomiting are classic associated symptoms. However, patients present frequently with nonclassic symptoms of sharp or stabbing pain without any of the associated symptoms above, and this often contributes to delays in seeking care, evaluation, and diagnosis. Management of patients with unstable angina includes aspirin therapy, admission or observation, continuous cardiac monitoring, and most often cardiac stress testing.
A 75-year-old man presents to the Emergency Department with acute lower abdominal pain and urinary retention. He was well until he woke up this morning to urinate and was unable to void. He reports a steady pressure in the suprapubic region and appears uncomfortable. His vital signs are within normal limits for his age. He has suprapubic tenderness on examination without costovertebral angle tenderness. A bedside ultrasound is obtained and shown above. Which of the following is the most appropriate next best step in management? Abdominopelvic computed tomography Bilateral renal ultrasound Suprapubic catheter placement Urethral catheter placement
Urethral catheter placement This elderly man has acute urinary retention likely secondary to benign prostatic hyperplasia (BPH), which is the most common cause of acute urinary retention in men. Urethral catheter placement will relieve acute urinary retention, alleviate pain, and prevent hydronephrosis. Other obstructive causes of acute urinary retention in men include phimosis, paraphimosis, urethral strangulation, meatal stenosis or stricture, penile trauma and prostate cancer.
A 65-year-old woman presents with back pain. She states that she slipped and fell on her back 2 days ago and reports shooting pains down both of her legs. The pain has been significant and worsening over the last 2 days. She is also experiencing increasing weakness in her legs and requires assistance to get into the room today. She has a history of diabetes mellitus, hypertension, and chronic back pain. Vital signs are within normal limits. On examination, the patient has diminished deep tendon reflexes. There is diminished sensation in the bilateral lower extremities and a palpable bladder. Which one of the following findings on her history and physical exam is most sensitive for the diagnosis described above? Motor weakness Radiculopathy Sensory deficits Urinary retention
Urinary retention Cauda equina syndrome etiologies are most commonly herniated discs, bone fragments, hematomas, epidural abscesses, tumors, or vascular insufficiency. A history of a recent spinal procedure, trauma, anticoagulation, intravenous drug use, and malignancy are all important risk factors. Clinical features may include saddle anesthesia, motor and sensory deficits in the lower extremities, sciatica, decreased lower extremity reflexes, decreased anal sphincter tone, and bowel or bladder dysfunction. Urinary retention is the most consistent finding with a sensitivity of 90%. A palpable distended urinary bladder secondary to urinary retention may be appreciated on exam. Frequently, the clinical features are asymmetric. Patients with a suspicious history and physical concerning for cauda equina syndrome require emergency MRI. Treatment is surgical, and an emergent consultation with a spine surgeon is required.
An older man is brought from the nursing home for evaluation of altered mental status. On laboratory testing he is found to have acute kidney failure with a BUN of 75 mg/dL and creatinine of 5.0 mg/dL. Which of the following is most likely to be elicited on history or physical examination? Cystitis Diuretic use Urinary retention Vomiting
Urinary retention The patient has markedly elevated BUN and creatinine values at a ratio of 15:1. When evaluating acute renal failure, the ratio of BUN/Creatinine is helpful for the identification of the underlying cause. Ratios of <10:1 are suggestive of an intrinsic renal problem (tubular, interstitial, or glomerular injuries) whereas post-renal pathology due to obstruction of flow (neurogenic bladder, mass, stricture, prostate enlargement) will result in a ratio of 10-20:1. In this case, acute urinary retention suggests an outflow obstruction either mechanically or due to medication side effects. Prerenal causes of renal failure will often lead to the BUN/Creatinine ratio of > 20.
A 27-year-old man presents to the emergency department with a rash. He states that he noticed these lesions, seen above, as he was going to bed and they felt warm and itchy. Physical exam is otherwise unremarkable. The patient has a past medical history of asthma. Which of the following is the most likely diagnosis? Angioedema Atopic dermatitis Contact dermatitis Urticaria
Urticaria Urticaria presents with pruritus and an erythematous plaque with central pallor and can be accompanied with deeper dermal swelling (angioedema). The pathophysiology is related to cutaneous mast cells, basophils, and the release of chemical mediators, including histamine. The treatment of urticaria only requires antihistamines, however, a short course of oral steroids can be given for more severe symptoms.
What is the most common cause of postpartum hemorrhage? Coagulopathies Lower genital tract lacerations Retained placental fragments Uterine atony
Uterine atony Postpartum hemorrhage usually occurs within the first 24 hours and is referred to as primary postpartum hemorrhage. Excessive blood loss in the postpartum period is defined as a cumulative blood loss of > 1,000 mL or bleeding associated with signs and symptoms of hypovolemia within 24 hours of delivery. The most common cause of postpartum hemorrhage is uterine atony. Aggressive fluid and blood resuscitation is the initial step in management while identifying and treating the underlying cause. Specific treatment for uterine atony includes uterine massage and oxytocin.
A 68-year-old man presents to the emergency department complaining of diarrhea and dehydration. He reports that he had a dental procedure two weeks prior for which he took antibiotics. A week later, he began having crampy abdominal pain and foul-smelling diarrhea six to seven times a day that is watery and contains mucus. His blood pressure is 102/68 mm Hg, heart rate is 103 bpm, temperature is 100.8°F, and oxygen saturation is 98% on room air. On examination, he has diffuse mild crampy abdominal pain. His lab work is notable for white blood cells of 13,000/mcL and positive fecal leukocytes. What is the most appropriate oral treatment? Ciprofloxacin Clindamycin Paromomycin Vancomycin
Vancomycin Clostridioides difficile Colitis (Pseudomembranous Colitis) Risk factors: history of recent antibiotic use (clinda most common), hospitalization, age > 65 Sx: loose stools (≥3 over 24h), abdominal pain Diagnosis made with two-step testing to distinguish from colonization: sensitive test initially (NAAT or GDH), followed by specific test: toxin EIA Treatment in adults: -Nonsevere or severe: oral vancomycin or oral fidaxomicin -Fulminant: oral vancomycin with parenteral metronidazole Treatment in children: -Mild: discontinuation of any nonvital antibiotic therapy and administration of fluid and electrolyte replacement -Nonsevere: oral vancomycin or oral metronidazole -Severe or fulminant: oral vancomycin Patients with at least two Clostridioides difficile infection recurrences treated with appropriate antibiotic therapy: guidelines recommend use of fecal microbiota transplantation
A 27-year-old Japanese woman with a history of Raynaud disease presents to the emergency room for chest pain. This is the third time this week she has presented for chest pain. Her prior visits have all been at night and she tends to arrive pain-free after receiving nitroglycerin from the paramedics. Her ECGs after the nitroglycerin have been normal. Upon arrival to the emergency room tonight, she is again pain-free with a normal ECG after nitroglycerin. While sitting down and awaiting her discharge paperwork, she begins having sharp chest pain. An ECG performed while having symptoms shows ST segment elevations in contiguous leads. Nitroglycerin is given and she rapidly becomes asymptomatic. Her repeat ECG is now normal. A repeat set of blood work, including cardiac enzymes, is normal. What is the most likely diagnosis? Non-ST elevation myocardial infarction Stable angina Unstable angina Vasospastic angina
Vasospastic angina Vasospastic Angina (Prinzmetal Angina) History of HTN, smoking, DM, obesity, or cocaine use Squeezing, pressure-like chest discomfort at rest ECG will show transient ST segment elevations Cardiac enzymes will be normal Diagnosis is made by cardiac stress test Most commonly caused by coronary artery spasm Treatment is calcium channel blockers and nitrates
Which of the following dysrhythmias can be triggered by premature ventricular contractions? Atrial fibrillation Atrial flutter Supraventricular tachycardia Ventricular tachycardia
Ventricular tachycardia
A 44-year-old woman presents to the emergency department with hyperglycemia and back pain. She has truncal obesity with relatively thin extremities as well as a protrusion over the nape of her neck. She has midline lower spine tenderness, and an X-ray demonstrates a compression fracture of the 12th thoracic vertebral body. The presence of which of the following additional findings supports your diagnosis? Bitot spots Dry, flaky skin Violaceous striae Xanthelasma
Violaceous striae Cushing disease is a specific type of Cushing syndrome (a condition of excess glucocorticoids) that results from a hypersecretory pituitary microadenoma. Clinical manifestations of cortisol excess may be present in nearly all organ systems. Classic findings include central obesity, thin and weak extremities due to proximal myopathy, rounded or "moon" facies, a fat pad on the back of the neck (buffalo hump), supraclavicular fat pads, purple or violaceous striae (stretch marks), facial plethora, amenorrhea in women, depression, psychosis, hypertension, osteopenia, osteoporosis, thin skin, easy bruising, poor wound healing, and hirsutism. Laboratory analysis may reveal hypokalemia, hyperglycemia, and elevated serum and urine cortisol level. Treatment involves gradual reduction of corticosteroids (in the case of exogenous use) or surgical removal of an adenoma.
What class of medications is recommended for treatment of performance-only social anxiety disorder? Monoamine oxidase inhibitors Selective serotonin reuptake inhibitors α2-adrenergic receptor agonists β-adrenergic receptor blockers
β-adrenergic receptor blockers Social anxiety disorder is defined as a phobia or intense fear of social situations and situations in which individuals are potentially under the scrutiny of others. Performance-only social anxiety disorder, on the other hand, refers to intense fear surrounding performance situations in which the patient may be scrutinized by others. This subgroup is characterized differently than classic social anxiety disorder including having a positive response to treatment with β-adrenergic receptor blockers.
A 70-year-old woman with hypertension and COPD presents with shortness of breath. On evaluation, she appears tachypneic with increased work of breathing. Her breath sounds are notable for diffuse wheezing. She reports no fevers, productive cough, headache, nausea, or vomiting. She is started on ipratropium and albuterol. Which of the following would be expected on an initial ABG? pH 7.25, PaCO₂ 80 mm Hg, HCO₃ 30 mmol/L pH 7.30, PaCO₂ 75 mm Hg, HCO₃ 14 mmol/L pH 7.37, PaCO₂ 90 mm Hg, HCO₃ 10 mmol/L pH 7.43, PaCO₂ 60 mm Hg, HCO₃ 28 mmol/L
pH 7.25, PaCO₂ 80 mm Hg, HCO₃ 30 mmol/L This patient is presenting with an acute exacerbation of COPD. The key to answering this question is understanding acid-base physiology in COPD exacerbations, which represents acute-on-chronic hypercapnia. This ultimately will lead to an acute rise in PaCO₂ on top of a chronic respiratory acidosis with metabolic compensation. in acute respiratory acidosis, we expect the bicarbonate to increase by 1 for each 10 mm Hg of PaCO₂, while in chronic we expect the bicarbonate to increase by 4 for each 10 mm Hg of PaCO₂. If we apply this to the above vignette, an acute PaCO₂ of 80 mm Hg should lead to a bicarbonate of 28 mmol/L, but it is 30 mmol/L, which indicates that there is some degree of renal compensation. Conversely, if this patient chronically lived at a PaCO₂ of 80 mm Hg, we would expect the bicarbonate to be 40 mmol/L. Therefore, an ABG showing a pH 7.25, PaCO₂ 80 mm Hg, and HCO₃ 30 mmol/L is most consistent with a patient with an acute-on-chronic COPD exacerbation.